Sei sulla pagina 1di 209

TABLE OF CONTENTS

PART I - DISPLACEMENT AND DRAFT

CHAPTER 1 - Fundamental Principles and Calculations

Force, Mass, Weight and Moment 3


Center of Gravity 3
Moment 6
Density 8
Archimedes' Principle 12
Principle of Flotation 15
Center of Buoyancy 16
Reserve Buoyancy 17

CHAPTER 2 -Displacement Calculation


Displacement 21
Light Displacement 22
Deadweight 23
What's the Connection? 23
The Estimated Weights Remaining On Board 24
Displacement -Deadweight -Cargo Calculation 24
Estimated ROB Calculation 25
Displacement Considerations 25

CHAPTER 3 -Calculating the Ship's Draft


Draft 31
Effects of Water Density on Draft and Displacement 32
Finding the Draft due to Change in Water Density when the
Displacement is Constant:
Fresh Water Allowance 35
Dock Water Allowance 36

i
Change of Draft Due to Density Formula 36
Using Hydrostatic Tables to find Change of Draft 36

Finding the Displacement Due to Change in Water Density


when the Draft is Constant 40

New Displacement? 40

CHAPTER 4 -The Hydrostatic Table


Using the Hydrostatic Table 45
Taking Values From The Hydrostatic Table
Taking The Value Of Displacement 46
Taking The Value Of Draft 46
Center Of Flotation (CF Or F) 48
Tons Per Centimeter Immersion (TPC) 49
Moment To Change Trim (MTC) 52
Center Of Buoyancy 54
Height of Metacenter (KM) 56
Interpolating Values from Hydrostatic Tables 57
Longitudinal Distances 58
The True Mean Draft 61

CHAPTER 5 -Quantity of Cargo


The Amount of Cargo the Ship Can Carry 65
Deadweight Limited by Draft 65
Draft Limit Imposed By The Load Line Rules 66
o The Load Line 66
o Finding the Amount of Cargo the Ship Can
Load When Limited by Load Line Regulations 70
o The Zone Allowance 74
Draft Limit due to Depths of Water 80
o At Departure Load Port 80
o At Destination 82
When Amount of Cargo Is Limited By Volume 87
Volume Of Cargo Spaces 88
Stowage Factor (SF) 88
Distribution Of Cargo Between Holds 91

ii
PART II: TRIM CALCULATIONS

CHAPTER 6 -Center of Gravity

Center of Gravity (G) 97


Longitudinal Center of Gravity (LCG) 97
The Use of Moments in Finding the Location of G 98
Movement of Center of Gravity 102
Finding the LCG from TRIM 107
Vertical Center of Gravity 108
Light Ship KG 108
The Vertical Movement of G 109

CHAPTER 7 -Longitudinal Stability and Trim

Longitudinal Stability 113


Trim 114
Trimming Moment 115
Calculating Trim 117
When Involving Large Changes in Mean Draft 117
Trim and Stability Calculation Sheet 118
When Handling Single / Several Moderate Weights 130
Shifting Weights 131
Loading or discharging weights 135
The Use of Trimming Table 139
Finding Required Trim to Arrive at Even Keel 141
Trimming Pours 144
Trim Considerations
Distribution of Trim Between Forward and Aft Draft 147
Change of Trim Due To Change in Density 149

iii
CHAPTER 8 -Transverse Stability

Transverse Stability 153


Behavior of Ship When Influenced by Internal Forces 153
Behavior of Ship When Influenced by External Forces 155
Transverse Shift of B 155
The Righting Lever (GZ) 156
Equilibrium of a Floating Ship 157

The Initial Stability 159


The Metacenter 159
The Metacentric Height 160
Light Ship KG 161
Final KG 161
Free Surface Correction 162
Assessing the Initial Stability 163
Stability at Large Angles of Inclination 164
The Statical Stability Curve 164
The Assumed Righting Lever 167
Dynamic Stability 169
Calculating the Initial Stability 173
Large Angle Stability Calculation 176
Stability Considerations 188
Grain Loading Calculations 195

iv
CHAPTER 1

FUNDAMENTAL PRINCIPLES AND CALCULATIONS

IN THIS CHAPTER

Force, Mass, Weight and Moment

Center of Gravity

Converting Mass to Weight and Vice Versa Moment

Density

Archimedes' Principle

Principle of Floatation

Center of Buoyancy

Reserve Buoyancy

1
2
FORCE, MASS, WEIGHT and MOMENT

Force - push or pull that tends to produce motion or a change in motion.

1. Force is a quantity measured using the standard metric unit known as the Newton. It is abbreviated
as N.

2. One Newton is the amount of force required to give a l-kg mass an acceleration of 1 m/s/s. To say
"10.0 N" means 10.0 Newton of force. One thousand Newton (1000 N) may be expressed as 1kN.

3. Units of force are expressed in Newton, tons, kilos, pounds, etc.

Types of Forces -there are several types of forces, some of which are listed below:

Contact Forces Action at a Distance Forces


Frictional Force Gravitational Force
Tensional Force Electrical Force
Normal Force Magnetic Force

Gravitational Force (also known as Weight) -The


force of gravity is the force at which the earth, moon,
acceleration or other massively large object attracts
another object due to gravity towards itself (= 9.81
2
m/s on Earth). By definition, this is the weight of the
object. All objects upon earth experience a force of
gravity, which is directed "downward" towards the
center of the earth. The force of gravity on earth is
always equal to the weight of the object as found by
the equation:

Force gravity = mass x acceleration due to gravity

Force gravity = mass x 9.81 m/s2

Center of Gravity

To simplify the study of the effects of gravitational


force, gravity = weight forces or weight, the total
or the resultant force of gravity acting on all points of
the body is considered. The resultant force is
considered at a central point called the center of
gravity. Center of gravity is the point about which the
body will balance. Through this point, the force of
gravity is considered to act vertically downwards,
with a force equal to the weight of the body.

In the study of ship's trim and stability, one of the


primary concerns is how to determine the ship's center
of gravity. The center of gravity of the ship or of any
object can be found by the use of moments.
For definition and calculation of moment go to page 9.

3
Mass -The amount of matter a body contains. Figure 1.2 - Consider the description of mass
and weight of the metal cylinder illustrated
1. Mass is different from weight, which is a below.
measure of the attraction of the earth for a
given mass.

2. The standard metric unit of measure of


mass is kilogram, but mass can also be
expressed in tons, pounds, long tons etc.

3. A kilogram (1,000 grams) was originally


defined as the mass of 1 cubic decimeter of
pure water at the temperature of its
maximum density (4.00 C/39.2 F).

At present, the standard kilogram is equal to


a block of metal alloyplatinum-iridium Mass is measured by a balance scale while
referred to as the "International Mass, weight is measured by a spring scale. A
measured by a balance scale with the balance scale will give the mass of an object
Prototype Kilogram". regardless of gravity, because the mass of
the cylinder is compared to a standard mass.

Weight -The gravitational force exerted on an


The spring scale measures the force of
object and measured in terms of Newton (N). gravity acting on an object. The scale is
graduated representing the stretch of the
Mass and weight are connected by the formula: spring caused by the force of gravity on
the metal cylinder. The scale calibrated
2 in Newton tells the weight of the object.
WEIGHT = mass x 9.81 m / sec

In this example, the force of gravity


Note: acting on the object is 1 kg x
9.81m/sec2. This is also the weight of
In non-technical terms, weight may be the object which, when expressed in
expressed in kilograms, tons, pounds, long tons, Newton, is 9.8 N.
etc.
In laymans terms, the weight of the
cylinder can be simply expressed as
1kg. The acceleration due to gravity
can be ignored since this is common
to all objects.

Volume the space that an object occupies,


expressed in cubic meters, cubic feet, etc.

Volume represents the space enclosed by a compartment, such as tanks, holds etc., in which case it may
be called capacity, referring to the amount of objects that the enclosure can accommodate at a time.

It also represents that part of the hull below the water line called the under water volume or "volume of
displacement".

4
CONVERTING MASS TO WEIGHT AND VICE VERSA

Example 1.1 - Converting mass to weight.

Find the weight of a body with a mass of 75 kilograms at a place where the acceleration due to gravity is
2
9.81 m/sec :

Weight = mass x acceleration due to gravity

2
Weight = 75 kg x 9.81 m/sec

= 735.75 kg-m/sec2

Ans. Weight = 735. 75 N or 0.73 kN

Example 1.2 - Converting weight to mass.

The lashing is capable of withstanding an ultimate load of not less than 133 kN. Find the equivalent load in
2
tons, assuming acceleration due to gravity (g) is 9.81 m/sec

1 kN = 1000 N kN = t-m/sec2

Weight
Mass = g

133 t-m / sec2


mass = 9.81 m / sec2 = 13.56 tons

Example 1.3 The calculated shear force in one of the stations in a ship is 635 tons. How much is this force
in terms of Newton?

Force in Newton = Force in tons x 9.8066 m/sec2.

= 635 x 9.8066

= 6227.2 N or 6.2 kN

NOTE:
To estimate the equivalent of N to kilograms or kN to tons, divide N or kN by 10. Ten is the rounded-
off value of acceleration due to gravity.

5
MOMENT

The moment of a force (or mass) about a point is the product of force and the distance from the point at
which it is acting. The force acting on a pivoted beam imparts a turning movement to the beam whose focal
point is the fulcrum or point of support. The turning effect on this point is expressed as moment.

Moment = force x distance

Turning effect will depend upon:

1. The magnitude (strength) of the force (such as the weight W 1 and W 2 in figure 1.2).

2. The perpendicular distance (length of the lever) between the lines of action and the point about which
the moment is taken (such as the distance d 1 and d 2 in figure 1.2).

Figure 1.4 Moment is the effect of a force applied at a distance from a point, which tends to produce a
turning effect.

W1 W2

In figure 1.5, a beam is shown supported at a fulcrum F. Two weights, W1 and W2 are acting upon the
beam and producing moments about the point F.

In the case of W 1 In the case of W 2

Moment 1 = W1 x d 1 Moment 2 = W 2 x d2
= 5 kg x 9 m = 10 kg x 6 m
= 45 kg-m = 60 kg-m

6
W 1 produces 45 kg-m moments, while W 2 produces 60 kg-m moments. The resultant of these moments
creates 15 kg-m turning effect acting at the fulcrum, which will tilt the beam down in the direction of W 2 .

In the figure 1.6 below, the W2 is moved away from the fulcrum such that:

W1 W2

in the case of W 1 , in case of W 2 ,

moment 1 = w1 x d 1 moment 2 = W2 x d2
= 10 kg x 6 m = 5kg x 12 m
= 60kg m = 60 kg m

Both weights on either side of the fulcrum F produce 60 kg-m moments each. The resultant of these
moments is zero kg-m, since one moment exactly balances the other. This causes the beam to rest in
equilibrium.

The use of moments in stability

Calculations in stability are mostly expressed in moment of force, such as the righting moment, upsetting
moment, heeling moment, trimming moment, etc. All these are internal and external forces acting on the
ship, expressed as moments of force.

The principle of moment can be used to locate the center of gravity, buoyancy, and flotation.

Fundamentally, moment is equal to the weight or force times its distance to the point where the moment
is to be taken. This turning effect is called the moment of force (often abbreviated as "moment"), and the
distance is called the moment arm (or lever arm) of the force.

NOTE The effect of a force applied at some distance from a point is called the moment of
force (at the point considered).

The turning effect of the force or "moment" is dependant on the distance of the force
from the point. The point is often times called pivot point, fulcrum, point of support,
or just a point used as reference for the moment.

Units of moments are expressed as Newton-meter, ton-meter, foot-ton, etc.

7
DENSITY

Density is the ratio of the mass of a body to its volume. In other words, density is mass per unit volume of a
body. Density gives you the idea of how tightly packed the matter is in an object.

Densities of solid objects

When discussing density what always comes to the mind of the


ship officer is the density of water. Density does not only apply to
water, it applies to all objects. As defined, density is the mass of
the object compared to a unit volume. Comparing the same
volume of metal with the same volume of wood, metal is heavier
than wood. You can say that matter is more tightly packed in
metals than in wood. Simply, metal is denser than wood.

The density of each object can be found by dividing its mass by


its volume.
mass
Density =
volume

Density and Stowage Factor

Stowage factor describes the amount of space required to store


objects. It is the volume of space occupied by objects (such as
cargoes) when stowed. Stowage factors are always approximate
depending on the shape of cargo holds and condition of cargo
upon loading.

Stowage factors (SF) are commonly expressed in units of cubic


meter per metric ton, cubic feet per long ton, cubic feet per metric
ton, etc.

3
SF = volume / weight --> m /MT
SF = volume / weight --> ft3/L T

Where volume = the space occupied when the weights are stowed.
3 3
Stowage factor conversion from ft / LT to m / MT and vice versa
3 3
ft /LT + 35.88 = m /MT
3 3
ft /LT + 35.31467 = m /MT
ft3/LT x 1.06047 = 3
ft /LT

8
Densities of water

Salt water -Dissolved solids are ever present in seawater. In seawater the vast majority of dissolved solids
are salts, the most abundant being sodium chloride. Seawater is denser than fresh water because of
dissolved salts in it. The total amount of salts present in different locations of the ocean vary due to addition
of fresh water. Seawater is about 2.5 percent denser than fresh water due to dissolved salts in it. Seawater
density is 1025 kg/m3.

Fresh water -Water free of dissolved solids such as salts is called fresh water. At 40 C the density of pure
water is 1000 kg/m3. The physical properties of water are used as standards of measurements such as in
the metric system: the gram.

Brackish water -is a mixture of salt and fresh water, where the content of fresh water varies. The density of
brackish water varies between 1.001 and 1.025 t/m3.

mass
Density =
volume

Density of FW = 1000 kg per cubic meter = 1000 kg/m3 or


3
= 1.000 ton/m

3
Density of SW = 1025 kg per cubic meter = 1025 kg/m or
3
= 1.025 ton/m

Specific Gravity

Specific gravity is defined as the ratio of the weight of the body to the density of fresh water. Specific gravity
(SG) is also referred to as relative density (RD). The SG of a substance is derived as:

SG = Density of substance
Density of fresh water

The SG of fresh water is thus found as: The SG of salt water can also be found as:

SG of FW = Density of substance SG of SW = Density of substances


Density of fresh Density of fresh
SG of FW = 1000 kg/m3 SG of SW = 1025 kg/m3
3 3
1000 kg/m 1000 kg/m

SG of FW = 1.000

9
DENSITY is the measure of how tightly packed matter is.

Figure 1.8 -Hydrometer


Hydrometer -an instrument used to measure the
density of liquids. It is based on the hydrostatic
principle of the Greek mathematician and
inventor Archimedes, which states that the
weight loss of a body in a liquid equals the
weight of the liquid displaced when immersed;
the graduated stem rises vertically to give a
scale reading. The scale is derived from the
weight of the hydrometer divided by the density
which gives the volume that must be
submerged. Marking this volume of the
hydrometer makes the scale which would
indicate the density of the water.

The hydrometer floats deeper in fresh water than in salt water since the same weight requires
greater amount of fresh water than salt water.
The same rule also applies to ships. A ship with a given displacement floats deeper in fresh
water than in salt water.

The use of density


Density is used in various calculations, such as in finding the mass and volume of liquids. In your study, you
will use density in correcting the ship's draft or displacement. It is essential that you keep in mind the
relationships of density, mass and volume.

Density = mass / volume

Mass = volume x density

Volume = mass / Density

One relationship is that the volume of water will be different for each density of water having the same
weight. Consider the following examples and notice the relationship of mass, density and volume.

Example 1.4 -Finding the volume of mass when density is given:

3
1.4.1 How much space does water occupy if it weighs 46,547 tons at density of 1.025 ton/m ?

Sol.:
Volume = mass
Density 1.025 ton/m3
3
45.412 m
mass = 46,547 tons /
3
density = 1.025 ton/m
3
volume = 45,412 m

10
1.4.2 How much space must the above weight of water occupy if its density is 1.000 ton/m3?

volume = mass
density
mass = 46,547 tons /

density = 1.000 ton / m3


3
volume = 46,547 m

3
1.4.3 How much space must the above weight of water occupy if its density is 1.015 tons/m ?

volume = mass
density
Mass = 46,547 tons /
3
Density = 1.015 ton/ m
3
Volume = 45,859 m

Notice that the volume of water is different for each density of water having the same weight. Which density
of water requires a greater amount of water for the same given weight?

Mass = 46,547 tons Mass = 46,547 tons Mass = 46,547 tons


3
Density = 1.025 ton/ m Density = 1.000 ton/ m3 density = 1.015 ton/ m3

Volume = 45,412 m3 Volume = 46,547 m3 Volume = 45,859 m3

Try pondering what the relationship of weight is when the volume of water is kept constant but has different
densities.

Example 1.5 Find the mass when volume and density is given.
3
1.5.1 At draft mark of 10.0m even keel, the underwater volume of a floating ship is 40,608 m ,

- Find the mass of seawater displaced.


3
- Find the mass of water displaced if the density is 1.008 ton/m

11
Sol.
3
mass = volume x density 40,608 m
1.025 tons/m3
SW displaced;
3
u. volume = 40, 608 m
3
x density = 1.025 ton/ m
mass = 41,623 tons

weight of SW = 41,623 tons

BW displaced; 40,608 m3
3
3
1.008 tons/m
u. volume = 40, 608 m
x density = 1.008 tons/ m3
mass = 40,933 tons

weight of BW = 40,933 tons

The terms "SW Displacement" and "BW Displacement are used only to describe the displacements
that the vessel must assume to be able to float at the same draft in SW and in BW water being
considered. Remember that it is based on the condition "to float at the same draft in different
densities.

Example 1.6 -Finding the equivalent mass of the same volume of water at different densities
3
1.6.1 The weight of displaced water at density of 1.025 tons/m 43,919 tons. Keeping the volume of liquid
the same, what is the weight of the water displaced if the density is 1.012?
3
volume = 42,848 m
Sol. x density = 1.012 tons/ m3
volume = mass mass 2 = 42,340 tons
density weight of water displaced = 42,340 tons
Mass 1 = 43,919 tons /
Density = 1.025 ton/ m3 Considering mass = displacement, the solution can be
volume = 42,848 m3 shortened to:
mass = volume x density
Displ2 = Displ1 x density2 / density1

ARCHIMEDES' PRINCIPLE

Archimedes' Principle states that when a body is wholly or partially immersed in liquid, it appears to suffer a
loss of weight equal to the weight of the liquid displaced.

To create the apparent loss of weight, there must be a force acting vertically upwards on the body. This
force is called the force of buoyancy and it is equal to the weight of liquid displaced by the body.

12
3
Figure 1.9 -Consider a metal block weighing 2.5 tons with a volume of 1.0 m , immersed in water with a
3
density of 1.000 t/m .

The apparent weight of the block which when immersed in water is reduced by an amount equal to the
weight of water displaced.

13
Figure 1.10 -Take the case of a wooden block. It weighs 0.6 ton and its volume is one cubic meter.

When the block is immersed in water, the block floats.

This is so, because the weight of water displaced is equal to the weight of the block. The force of buoyancy
(from the water displaced) is sufficient to support the block to float.

The other way of looking at this is by comparing the density of the material to the density of water. In the
earlier illustration, the metal block is denser than water therefore it sinks; in the later, the wooden block is
less dense than the water, therefore, it floats.

14
PRINCIPLE OF FLOTATION

The Archimedes Principle also applies to ships. Although ships are made of steel, they are hollow inside
which enables them to displace enough water to support its weight.

From Archimedes principle, when a body is floating in liquid, the weight of liquid displaced is equal to the
weight of the body.

Weight of body = weight of liquid displaced

A body released into the water will submerge and displace water. By how much the body is submerged or
how much water is displaced depends upon the bodys volume and weight. The body will be immersed until
the weight of the water displaced equals its own weight. That means the force of buoyancy or the up thrust
acting on the immersed portion of the body is already equal to the weight of the body. If the body is unable
to displace water equal to its own weight then it will continue to submerge and eventually sink.

Figure1.11 A floating ship displaces water equal to its own weight.

Example 1.7

A block-shaped object measures 8 meters long, 2 meters wide and 1.5 meters tall, and weighs 25 tons.
3
Determine whether the object will float or not, if immersed onto water with a density of 1.000 t/m

Sol.:

Mass of water that must be displaced = 25 tons and

Mass of water the object can displace = 8m x 2m x 1.5m x 1.025 t/m

Mass of water the object can displace = 24.6 tons

Answer The object is unable to displace an equal amount of water therefore it will sink.

15
BUOYANCY (B)

The upward thrust exerted by water on the submerged


portion of a body is called force of buoyancy. The
force of buoyancy is equal to the weight of the liquid
displaced by the body. If a body is unable to displace
the amount of liquid equal its own weight, then the body
will sink. The force of buoyancy is still there but not
enough to counter the force of gravity and keep the body
afloat.

For a floating ship, the force of buoyancy is equal to the


weight of the water displaced by the submerged part of
the ship. The direction of buoyancy force is upwards
that in effect, it counters the downward force of the
ships weight.

Center of Buoyancy

The force of buoyancy acting on the body makes the


body seem to suffer a loss of mass, and it appears that
the body is lighter than when it is suspended in air. The
resultant of this force is considered to act vertically
upwards through a point, which is the center of the
submerged part of the body known as the center of
buoyancy. Being the center of the submerged portion,
the location of the net upward force of center of
buoyancy is dependent on draft. As the draft changes so
does the location of center of buoyancy.

Force of Buoyancy = Weight of the ship

The location of center of buoyancy is described by its


vertical distance or longitudinal distance from a
reference line. Vertically, the location of center of
buoyancy is measured from the keel and referred to as
KB. Longitudinally, the location of center of buoyancy is
measured from either amidships or from aft
perpendicular. This is referred to as LCB.

KB is used to calculate the transverse stability of the


ship while LCB is used to calculate the ship's trim.

The KB and LCB of the ship at each draft are listed in


the ship's hydrostatic table. The values are the
measurements when the ship is upright and in an even
keel.

Being the center of the submerged portion, the location of center of buoyancy is dependent on draft.
As the draft changes, so does the location of the center of buoyancy.

16
RESERVE BUOYANCY

Reserve buoyancy is the remaining volume of enclosed


spaces above the waterline. It provides the required
buoyancy to accommodate weights added to the ship,
when compartments are flooded, or the extra buoyancy
required in stormy weather conditions.

When several compartments are flooded and the water


has free communication with the sea, some buoyancy is
lost. When the reserve buoyancy in freeboard is
consumed and the water displaced is no longer equal to
the weight of the ship, the ship sinks.

Example

Consider the box-shaped container: length = 2.0 m,


height = 0.8 m, breadth = 1.2 m

The combined weight of the container and the object


inside it is equal to one ton. The density of water is
3
1.025 t/m .

Find the volume of water displaced.

Find the draft at which the box will float.

Find the volume of reserve buoyancy.

17
SUMMARY

1. For a ship to float, it must displace a mass of water equal to its own mass. It can also be said:

Weight of a floating ship = weight of water displaced

2. When the mass of a ship changes, the mass of water displaced changes by equal amount.

3. For a floating body, the immersed or underwater volume of the ship is equal to the volume of displaced
water.

Underwater volume = volume of water displaced

EXERCISE 1 - Density, mass, and volume


3 3
1.1 Determine the mass of water with a density of 1.022 t/m when it occupies 195 m of space in a tank.

1.2 A ship displaces 7546 MT. Calculate the volume of water displaced when floating in SW and when in
FW.

1.3 The underwater volume of the ship up to a draft of 5.5 m is 66S1. Find the ship's displacement, when
floating in SW and in FW.

1.4 A piece of wood measures 6 m long, 2 m wide, and 1 m tall. The wood weighs 8 tons. Determine
whether the wood will float in SW or not. If it does float, determine the volume of water displaced.

Write your solutions here:

18
CHAPTER 2
DISPLACEMENT CALCULATIONS

IN THIS CHAPTER

Displacement

Light displacement

Deadweight

Whats the connection?

The estimated Weights Remaining On Board

Displacement deadweight cargo calculations

Estimated ROB calculation

Displacement considerations

19
20
Displacement
Describing displacement two ways.

First, by way of displaced water. The displacement of a floating ship is equal to that amount of water which
the submerged portion of the ship displaces. By Archimedes Principle the weight of the water displaced is
equal to the weight of the ship.

The weight of water is the volume of water displaced times the density of water. The quantity of water
3
displaced expressed in cubic meters (m ) is equal to the volume of the submerged portion of a floating ship.
The ship's displacement can be calculated by:

Displacement = volume of water displaced x density

Secondly, by way of accounting the weights. Displacement is the weight of the ship, denoting the weight of
the hull and machineries, including everything and everyone onboard.

The total weight of the ship or its displacement is the sum of:

1. Non-removable weightrepresenting the weight of hull and machinery, which is equal to the light
displacement of the ship or often times called lightship.
2. Removable weightsuch as fuel oils, diesel oils, lube oils, fresh water, ballast, constant, cargoes and
any other weights onboard. These are called the deadweight.

Figure 2.1

Displacement = Light Displacement + Deadweight

Displacement = underwater volume of the ship x density

The displacement by the first and the second descriptions must be equal.

21
Meaning of "displacement" when used formally.

Load Displacementis the mass of the ship floating even keel at her summer load line in salt water.

Light Displacementis the mass of the ship when floating with no cargo fuel, stores or any other
weights not forming part of the hull or machinery or fixed equipment of the ship. In some loading
manuals, light displacement is also referred to as lightship and lightweight.

Volume of Displacementis the underwater volume of the ship.

The submerged or under water volume of the ship is called the volume of displacement and can be
calculated for a series of vertical distances from keel. For ship-shaped objects, the underwater volume can
be found by Simpson's Rules. After finding the volume, this is multiplied by density to find the ship's
3
displacement for this draft. In the preparation of a hydrostatic table, the density used is 1.025 t/m . The
displacement of the ship as calculated (by naval architects) is then tabulated against the draft. In practice,
the water level is read as draft and the displacement determined from the tables. Displacement is commonly
represented by the symbol . The underwater volume can be also compared to the volume of rectangular
block having the same length, breadth and draft of the ship. The ratio of the underwater volume of the ship
to the volume of the block is termed the block coefficient (Cb).

Cb = underwater volume / (length x breadth x draft)

When the Cb is known, the underwater volume of the ship can be determined by the formula:

Underwater volume = length x breadth x draft x Cb.

The purpose of Cb is to measure the ship's underwater "fineness" and it is often used in the application of
regulations, such as those governing its freeboard or strength.

LIGHT DISPLACEMENT

Light displacement is the mass of the ship when floating with no cargo, fuel stores or any other weights not
forming part of the hull or machinery or fixed equipment as taken by the ship builders at the final stage of
ship construction. The shipbuilders supply this information in the ship's loading manual. This includes the
position of center of gravity (LCG and KG) in lightship condition and the ship's constant.

The figures are from the results of the inclining tests usually conducted by shipbuilders. These values can
serve as the initial condition from which the displacement and the position of G (ex. Final LCG and
KG) for any loading condition may be calculated.

NOTE: The light displacement serves as the initial condition of the ship in finding the final
displacement. All weights on board when added to the light ship must be equal to the ship's final
displacement.

22
DEADWEIGHT

Deadweight (Dwt) -is the difference between light displacement and load displacement. The deadweight is
also defined as the total amount of weight that can be loaded to bring the ship down to her load
displacement. By accounting this must equal to the amount of cargo, fuel, fuel stores, ballast, etc. which
when added to her light displacement, will bring the ship to her summer mark.

The word deadweight in everyday usage may also mean as the sum of all weights presently onboard.

Deadweight = Displacement - Lightweight

The deadweight besides the cargo includes items called weights on board, deductibles, or non-cargo weight.
These are the bunkers, stores, ballast, and other items carried not forming part of the cargo. The method of
listing these items varies from ship to ship. What is important is to ensure that all the weights are listed and
taken into account when calculating the ship's deadweight.

Deadweight = Cargo weight + non cargo weights

By transposing the above formula, the weight of cargo can be calculated by;

Cargo weight = Deadweight - non cargo lights

Non-cargo weights can be categorized into;

Fuel Oils Ballasts


Diesel Oils Fresh Water Constant or
Lube Oils Stores Any other weights not forming part of the cargo

The non- cargo weights

In planning the loading, an estimated quantity of fuel oil, diesel oil, lube oil, fresh water, ballasts, stores and
constant is needed. Such quantities must be known at departure from loading port or arrival at discharging
port. The quantities can be taken by sounding the tanks or compartments. Then quantities remaining
onboard in the future are taken by applying the estimated consumptions.

WHAT'S THE CONNECTION?

The relationship between displacements, deadweights, non-cargo weights and cargo can be summarized
as:

Displacement Cargo weight


- Lightship + Non- cargo weight
Deadweight Deadweight
- Non- cargo weight + Lightship
Cargo weight Displacement

The described relationship is used in calculating the amount of cargo the ship is able to carry.

23
Example 2.1

The vessel MV Max Panama is to load to her summer load line. The non-cargo weights on board are listed
below. Find how much cargo she can load.

Fuel Oil = 1,290 MT Displacement = 78,849 MT


Diesel Oil = 95 MT Light displacement = 9,614 MT
Fresh Water = 268 MT Deadweight = 69,235 MT
Ballast = 0 MT Weights on board = 1,883 MT
Constant = 230 MT
Weights on board = 1,883 MT Max. cargo = 67,352 MT

Ans: Loadable cargo = 67,352 MT

The maximum amount of cargo in this case depends upon the limiting draft, which
is the load line. The conditions in the course of the voyage must be studied to
know the limiting draft. The amount of cargo cannot be determined until the
limiting draft is known.

Example 2.2 -Finding Displacement

The ship's light displacement is 8951 MT. She has fuel and lube oil remaining on board listed as follows. IFO
= 1013 MT, MDO = 132 MT and LO = 23 MT. She also has FW = 205 MT, and the sum of ballast water =
9675 MT. Her constant is 240 MT.

Find her displacement.

Displacement = light displacement + deadweight

IFO = 1,013 MT Lightship = 8,951 MT


MDO = 132 MT present deadweight = 11,288MT
LO = 23 MT present displacement = 20,239 MT
FW = 205 MT
Ballast = 9,675 MT
Constant = 240 MT
present deadweight = 11,288 MT

Ans. displacement = 20,239 MT

Note:
Present deadweight is the sum of all the weights presently onboard. For convenience, deadweight
can be grouped into non-cargo weights and cargo weights.

Example 2.3 -Finding Displacement

A ship's light displacement is 7,595 MT. Initially she has remaining on board IFO = 287MT, MDO = 35 MT,
LO = 21 MT, FW = 58 MT, BLST = 8085 MT. Her constant is taken as 230 MT. She then took bunkers and
FW listed as IFO = 910 MT, MDO = 60 MT, FW = 150 MT. After removing all her ballast she has also loaded
25,600 MT of cargo.

1. Find her displacement before bunkering and loading operations.


2. Find her displacement after loading operations.

24
Solution:

Initial Rob's (before bunkering/loading)

IFO = 287 MT Lightship = 7,595 MT


MDO = 35 MT Deadweight = 8,716 MT
LO = 21 MT Displacement = 16,311 MT
FW = 58 MT
BLST = 8,085 MT
CONST = 230 MT
deadweight = 8,716 MT

Final ROB's (after bunkering/loading/de-ballasting)

IFO = 1,197 MT Lightship = 7,595 MT


MDO = 95 MT Deadweight = 27,351 MT
LO = 21 MT Displacement = 34,946 MT
FW = 208 MT
BLST = 0 MT
CONST = 230 MT
CARGO = 25,600 MT
Deadweight = 27,351 MT

Ans. displacement before bunkering and loading = 16,311 MT


displacement after bunkering and loading = 34,946 MT

DISPLACEMENT CONSIDERATIONS

Remember that displacement is also equal to the volume of water displaced times the density of water. The
volume of water is equal to the under water volume of the ship. When the density of water changes, so will
the volume of water displaced. Any increase/decrease in density means an increase/decrease in volume of
water displaced, hence an increase/decrease in draft. For the draft to remain the same in water of different
densities, the displacement of the vessel must be changed.

Example 2.4 -Displacement to remain constant

The ship's displacement is 32,560 MT.

1. Find the volume of SW displaced.


3
2. Find the volume of BW displaced if the density of water is 1.011 ton/m .

Solution for no.1 Solution for no.2


u. volume = mass / density u. volume = mass / density
3
u. volume = 32,560 MT/ 1.025 ton/m . u. volume = 32,560 MT / 1.011 ton/m3
3 3
u. volume = 31,765.8 m u. volume = 32,205.7 m

Ans. 1. Volume of SW displaced = 31,765.8 m3


2. Volume of BW displaced = 32,205.7 m3

At the same displacement but at different densities, the volume of displacement changes hence the change
in draft.

25
Example 2.5 -Volume of displacement to remain constant.
3
When floating on certain draft, the ship's underwater volume is 29,405m .

1. Find her displacement if she is floating in SW.


3
2. Find her displacement if she is floating in brackish water density 1.014 ton/m .

Solution for no.1 Solution for no.2


Displ = u. volume x density Displ = u. volume x density
3 3 3 3
Displ = 29,405 m x 1.025 ton/m Displ = 29,405 m x 1.014 ton/m
Displ = 30,140.1 MT DispI = 29,816.7 MT

The ship must assume the displacement at each case in order to displace the same volume of water or
remain at the same draft.

Underwater volume of the ship is also referred to as the volume of displacement. Keeping the volume of
displacement the same means keeping the ship floating at the same draft.

NOTE: The formula in Examples 2.4 and 2.5 can be combined to get:

Displacement N = Displacement O x densityN


Density O
Review also Example 1.6 on page 15.

NOTE: When the density of water changes, so will the volume of water displaced. Any
increase/decrease in density means an increase/decrease in volume of water displaced, hence an
increase/decrease in draft. For the draft to remain the same in water of different densities, the
displacement of the vessel must be changed (either by loading or discharging).

26
EXERCISE 2 -Displacement and Deadweight

A ship's light displacement is 1771 MT. All the weights on board were accounted as follows: ballast is
1658 MT, fresh water is 76 MT, fuel oil is 206 MT, diesel oil 41 MT and constant, 48 tons. Determine the
ship's present deadweight and displacement.

A ship's light displacement is 1771 MT. She loaded 5108 MT of cargo. After loading, accounting of other
weights onboard are as follows; fresh water including drinking water is 152 MT, fuel oil is 412 MT, diesel
oil is 82 MT, constant is 48 tons. Determine the ship's present deadweight and displacement.

A ship's summer displacement is 7573 MT. Her lightship is 1771 MT. The ship has following weights
already onboard; fresh water 118 MT, fuel oil 376 MT, diesel oil is 68 MT and constant 48 tons.
Determine how much cargo must be loaded so as not to exceed her summer displacement.

------------------------------------------------------------------------------------------------------------------------------------------------
------------------------------------------------------------------------------------------------------------------------------------------------
------------------------------------------------------------------------------------------------------------------------------------------------
------------------------------------------------------------------------------------------------------------------------------------------------
------------------------------------------------------------------------------------------------------------------------------------------------
------------------------------------------------------------------------------------------------------------------------------------------------
------------------------------------------------------------------------------------------------------------------------------------------------
------------------------------------------------------------------------------------------------------------------------------------------------
------------------------------------------------------------------------------------------------------------------------------------------------
------------------------------------------------------------------------------------------------------------------------------------------------
------------------------------------------------------------------------------------------------------------------------------------------------
------------------------------------------------------------------------------------------------------------------------------------------------
------------------------------------------------------------------------------------------------------------------------------------------------
------------------------------------------------------------------------------------------------------------------------------------------------
------------------------------------------------------------------------------------------------------------------------------------------------
------------------------------------------------------------------------------------------------------------------------------------------------
------------------------------------------------------------------------------------------------------------------------------------------------
------------------------------------------------------------------------------------------------------------------------------------------------
------------------------------------------------------------------------------------------------------------------------------------------------
------------------------------------------------------------------------------------------------------------------------------------------------
------------------------------------------------------------------------------------------------------------------------------------------------
------------------------------------------------------------------------------------------------------------------------------------------------
------------------------------------------------------------------------------------------------------------------------------------------------
------------------------------------------------------------------------------------------------------------------------------------------------
------------------------------------------------------------------------------------------------------------------------------------------------
------------------------------------------------------------------------------------------------------------------------------------------------
------------------------------------------------------------------------------------------------------------------------------------------------
------------------------------------------------------------------------------------------------------------------------------------------------
------------------------------------------------------------------------------------------------------------------------------------------------
------------------------------------------------------------------------------------------------------------------------------------------------
------------------------------------------------------------------------------------------------------------------------------------------------
------------------------------------------------------------------------------------------------------------------------------------------------
------------------------------------------------------------------------------------------------------------------------------------------------
------------------------------------------------------------------------------------------------------------------------------------------------
------------------------------------------------------------------------------------------------------------------------------------------------
------------------------------------------------------------------------------------------------------------------------------------------------
------------------------------------------------------------------------------------------------------------------------------------------------
------------------------------------------------------------------------------------------------------------------------------------------------
------------------------------------------------------------------------------------------------------------------------------------------------
------------------------------------------------------------------------------------------------------------------------------------------------
------------------------------------------------------------------------------------------------------------------------------------------------

27
28
CHAPTER 3
CALCULATING THE SHIPS DRAFT
IN THIS CHAPTER

Draft

Effects of Water Density on Draft and Displacement

Finding the Displacement Due to Change in Water Density when the Draft is Constant

New Displacement?

Finding the Draft due to Change In Water Density when the

Displacement is Constant

29
30
DRAFT

Draft is the distance between the waterline and the keel. In contrast, the freeboard this is the distance
between the waterline and the upper edge of the uppermost continuous deck.

Figure 3.1 -Draft and Volume of Displacement

A floating ship displaces water equal to her own weight. Since mass is equal to volume times density, the
ship must float such that its underwater volume will displace an equal amount of water which when multiplied
by its density equals that of the ship. The ship, to provide the necessary volume of displaced water, will have
to immerse to a certain depth. This depth or distance from the bottom of the keel to the water surface is
called the draft.

The draft is directly related to the underwater volume of the ship, that every time weights are added, the
draft increases. That is because any increase in displacement entails an increase in the volume of water
displaced. When weights are discharged, the displacement is decreased and any decrease in displacement
requires a decrease in volume of water displaced, thus, a decrease in draft. It follows that the draft is directly
proportional to displacement.

Draft Marks

Numbers are permanently marked on both sides of the ship in the fore, amidships and aft to indicate the
draft at which the ship is floating. The markings may not be located at the perpendiculars. This introduces
differences in draft read at marks and the draft calculated at the perpendiculars. For calculations demanding
high degree of accuracy such as the draft survey, draft read at marks must be corrected to draft at
perpendiculars.

Figure 3.2 -The relative position of draft marks with the perpendiculars

Factors that may cause the draft of the ship to change

1. Loading and discharging of weights.


2. Moving in water of different densities.
3. Listing of the ship.
4. Lost buoyancy.

31
THE EFFECTS OF DENSITY ON DRAFT AND DISPLACEMENT

Remember that displacement is equal to the volume of water displaced times the density. Hence, changes
in water density affect the draft.

A ship keeping its displacement constant will float deeper in FW than in SW as the volume of displacement
varies with density. In effect;

1. If the desired displacement of a ship is not changed, when allowed to float from water of one density to
another, the drafts at which she will float will not be the same.

2. For the desired draft of a ship to remain the same when floating from water of one density to another,
then the displacements must be changed.

NOTE: In ports and canals with draft limit the ship must be at the correct displacement in order that
she float not exceeding the draft limit. If the ship must float to such a draft limit, this might affect the
amount of cargo she can carry. Draft is related to displacement and therefore, is the deadweight.

Change of draft due to change in water density (where the ship is floating).

While the ship's displacement remains the same, the volume of displacement varies with the density. This is
so because the ship must displace the same weight of water in each case. In order to displace the same
weight of water when the density of the water has changed, the volume of water displaced must increase or
decrease.

The ship being fixed in form can increase or decrease her underwater volume by sinking to a deeper draft or
rising to a lighter draft. It can be said that a ship floating on water of one density then to another density
while keeping her displacement the same, the ship's draft will change.

Figure 3.3 - Illustrating the effect of density to draft

From the Archimedes Principle

32
A floating ship displaces water the amount of which
equals the underwater volume of the ship. The
weight of the displaced water is equal to the weight
of the ship. The underwater volume of the ship
relates to its draft.

3.3.2 If the density of the water changes then the


equilibrium is lost. The amount of water displaced
is no longer equal to the weight of the ship.

To regain equilibrium when the displacement of the


ship is not changed then the amount of water
displaced must change to keep it equal to the
weight of the ship. To do this, the ship is immersed
to a new draft to increase (or decrease) the volume
of water displaced until both weights are again
equal, hence the change in draft.

33
Figure 3.4 -A ship will float to different drafts in waters of different densities when her displacement remains
the same.

Displ = 34,823 MT Displ = 34,823 MT

Volume of displacement = 34,823/ 1.025 Volume of displacement = 34,823/1.000


3
= 33973.6 m = 34,823 m3

The volume of displacement corresponds to The volume of displacement corresponds to draft 2 .


Draft 1 .

When the change of density of water is decreasing as from SW to FW, the volume of water displaced
increases, therefore, an increase in draft. When the change of density of water is increasing, as in from FW
to SW, then the volume of water displaced must decrease, hence the draft also decreases.

The amount by which the draft will either increase or decrease because of the change of water density
where the ship is to float is called change in draft due to density. The ship officer must know how much this
change of draft will be.

Given either the draft or displacement of the ship, the change in draft (either increase or decrease) can be
estimated by;

1. Using Fresh Water Allowance


2. Using Dock Water Allowance
3. Using the formula:
Change in draft due to density change = x (1.05 x dd )
dd x TPC X 100
4. Or, using the hydrostatic tables.

NOTE: To find the new draft, apply the change in draft to the old draft as:

If change of draft is sinkage = Initial draft + change of draft


New draft

If change of draft is rise = Initial draft - change of draft


New draft

34
FINDING THE NEW DRAFT DUE TO CHANGE IN DENSITY WHEN DISPLACEMENT IS CONSTANT

Fresh Water Allowance (FWA)


3 3
When a ship operates in water density of 1025 kg/m (salt water) to water density of 1000 kg/m (fresh
water) or vice versa, the change in draft is referred to as Fresh Water Allowance. It is found by the formula
FWA, applied as an increase in draft when calculating the draft from SW to FW and as a decrease in draft
when calculating the draft from FW to SW.

FW draft = SW draft + FWA and SW draft = FW draft -FWA

Fresh water allowance can be calculated for a ship at any draft at which she is floating. It is normally of
interest when the ship is at, or close to, her summer load line that it is desired to determine how much
deeper than the summer draft in salt water she may be immersed when operating in fresh water.

Example 3.1 -FWA calculation

The load displacement of MV Max Panama is 78,849 MT at a draft of 13.295 m when floating in SW.
3
1. Find the FWA and her draft when floating in dock water density of 1.000 t/m .

Sol.

FWA = Displ / 4 x TPC


FWA = 78,849 / 4 x 64.4
FWA = 306 mm or 0.306 m

SW mean draft = 13.295 m


FWA in meters = 0.306 m
FW mean draft = 13.601 m

Ans. draft at FW = 13.601 m

Example 3.2 -FWA Calculation

The load displacement of MV Nara is 30,884 MT, and draft at 9.55 when floating in SW.

1. Find the FWA and her draft when floating in FW.

FWA = Displ / 4 x TPC

FWA = 30,884 / 4 x 35.46

FWA = 218 mm or 0.218 m

SW mean draft = 9.55 m


+ FWA in meters = 0.218 m
FW mean draft = 9.768 m

Ans. draft at FW = 9.768 m

35
Dock Water Allowance (DWA)
3
The change in draft when a vessel is operating from salt water of density 1.025 t/m then to a water of
3 3
density between 1.025 t/m (SW) and 1000 t/m (FW), or vice versa, is commonly called the dock water
allowance.

Dock water allowance is found by simple proportion from FWA formula. Thus;

DWA = FWA (1025 dd) or DWA = FWA (1.025 dd)


25 0.025

Like FWA, the DWA is applied as an increase in draft when calculating drafts from SW to DW, and the
reverse when from DW to SW.

DW draft = SW draft + DWA, and SW = DW draft DWA

Example -3.3 - DWA calculation

The vessel MV Max Panama is fully loaded to her summer displacement. Find the mean draft if she is
3
floating in dock water density of 1.012 t/m .

DWA = FWA (1025 dd) SW mean draft = 13.295 m


25 + DWA in meters = 0.159 m
DWA = 306 (1025 -1012) Final mean draft = 13.454 m
25
DWA = 159 mm or 0.159 m Ans. draft at DW = 13.454 m

Example 3.4 -DWA calculation

The vessel MV Nara is fully loaded to her summer displacement. Find the mean draft if she is floating in
3
dock water density of 1.015 t/m .

DWA = FWA (1025 -dd) SW mean draft = 9.550 m


+ DWA in meters = 0.088 m
DWA = 219 (1.025 - 1.015) Final mean draft = 9.638 m

DWA = 87.6 mm or 0.088 m Ans. draft at DW = 9.638 m

Change of Draft due to Density formula

The same change of draft as Dock Water Allowance can be calculated by:

Change in draft due to density of water = x (1.025/dd 1) / (TPC x 100)


(change of draft in meters)

Or by the formula:

Change in draft due to water density = x (1,025 -dd ) / dd x TPC x 100

Where:
displacement = present displacement
dd = dock water or new density
TPC = in salt water

36
For calculator operation, the formula can be written as:

Change in draft due to water density = x (1.025 -dd) / (dd x TPC x 100)

Change in draft due to water density is applied to the Initial draft following the same rule as FWA and DWA.

Example 3.5 -change of draft calculation

The vessel MV Max Panama is fully loaded to her summer displacement. Find the mean draft if she is
3
floating in dock water density of 1.012 t/m . -

Cod = x (1.025 -dd ) Initial mean draft = 13.295 m


dd x TPC x 100 Cod in meters = 0.157 m
Cod = 78849 x (1.025 -1.012) Final mean draft = 13.452 m
1.012 x 64.4 x 100
Cod = 0.157 m Ans. draft at DW = 13.452 m

Example 3.6

The vessel MV Nara is fully loaded to her summer displacement. Find the mean draft if she is floating in
3
dock water density of 1.015 t/m .

Cod = x (1.025 x - dd) Initial mean draft = 9.550 m


dd x TPC x 100 Cod In meters = 0.086 m
Cod = 30884 x (1:025 -1.015) Final mean draft = 9.636 m
1.015 x 35.46 x 100
Cod = 0.086 m Ans. draft at DW = 9.636 m

Finding Draft Due to Change of Density by Using Hydrostatic Tables

The draft at BW density can be found by first dividing the SW displacement by the BW density to get the
3
volume of displacement (underwater volume of the ship), then multiplying it by density 1.025 t/m . This is
called Look up" displacement. This serves as the displacement to look-up in the hydrostatic table to find the
draft. The draft corresponding to this look-up displacement would be the draft at BW.

What had been done above was to convert the ships displacement to "volume of displacement". This is the
underwater volume required to float at a given density. There must be a draft representing this volume in the
hydrostatic table. But the hydrostatic table is tabulated for a ship in SW, so the volume of displacement must
be converted back to weight in SW and call it "look up" displacement. The "look up" displacement does not
represent the displacement of the ship but merely as an argument to look up for the draft at BW.

Example 3.7 - Find the draft of the same displacement in different water densities.

The vessel MV Max Panama's mean draft in SW is 13.29 m. Find the mean draft in dock water density of
3
1.013 t/m .

Draft Displacement
13.28 78753
13.29 78817
13.30 78882

37
Step 1. - Calculate the volume of water to displace at density 1.013.

volume = weight/density

volume of displacement = 78817/1.013


3
volume of displacement = 77805.5 m

volume of water displaced = underwater volume of ship

Step 2. -Look-up the draft that corresponds to this under water volume by multiplying the volume by 1.025
3
t/m .

weight = volume x density

look-up displacement = u. volume x density

look-up displacement = 77805.5 x 1.025

look-up displacement = 79750.6 MT Ans. = 13.435 m

This look-up displacement corresponds to the draft of 13.435 m. At this draft the under water volume is
3
77805.5 m which is equal to the volume of the actual displacement of 78817 MT when the density is 1.013
3
t/m .

The above calculation can be shortened to:

Look up displacement = present displ x 1.025 / dd

Look up displacement = 78817 x 1.025 / 1.013

Look up displacement = 79750.7 MT


3
From hydrostatic tables this corresponds to the draft 13.435 m and this will be the draft at density 1.013 t/m .

REMEMBER

It should be clearly understood that the actual displacement of the


ship remains as 78817 MT. The weight of the volume of displacement
is not her actual displacement at that draft. It was found only to
facilitate obtaining the new draft from the hydrostatic table, which is
tabulated for the vessel in SW.

Example 3.8

The MV Max Panama's present displacement is 77080 MT. Using the ship's hydrostatic tables find the draft
3
if she floats in water with density of 1.012 t/m .

Look up displacement = 77080 x1.025 / 1.012


= 78070 MT

From the Hydrostatic table, this corresponds to the hydrostatic draft of 13.174 m.

38
The Effects of Density to Displacement When the Draft is Held Constant

For a ship to float at the same draft on the water of one density then to another, the displacement
must be changed.

Draft is directly related to the underwater volume of the floating ship (volume of displacement) and for the
draft not to change, the underwater volume must also remain the same. The same underwater volume
multiplied by two different densities will produce two displacements. Therefore, to maintain the draft, the ship
must assume the displacement for each case.

Figure 3.5 -The effect of density to displacement

From the premise that;

A floating ship displaces water the amount of If the density of the water changes,
which is equal to the underwater volume of then the equilibrium is lost. The
the ship. The underwater volume of the ship amount of water displaced is no longer
relates to its draft. The weight of the equal to the weight of the ship.
displaced water is equal to the weight of the
ship.

As a rule:

To maintain the same


draft at different
densities, the ship
must assume the
displacement for each
Density.

To regain equilibrium and not change the


amount of water displaced, thereby keeping
the draft the same, the displacement of the
ship must be changed. The displacement is
changed by either loading or discharging
weight to that displacement.

39
The displacement of the ship on the draft she is floating can be taken directly from the hydrostatic tables.
3
Such displacement, however, was calculated with the density of 1.025 tons/m . If the water where the ship
operates is of another density, then the hydrostatic displacement must be corrected in order to float at that
same draft.

Displacement (corrected for density) can be found by:

Displacement = Tabulated displacement x (new density / 1.025)

The above formula is from the ratio:

New displacement = New density


Old displacement Old density

The displacement does not change on its own as compared to the effect of density on the ship's draft. The
displacement must be changed either by loading or discharging.

NOTE : Remember the formula:

Displacement N = Displacement O x density N


density O

Example 3.9 -Finding displacement In water of different density when draft is held constant.
3
Calculate the displacement of MV Nara corresponding to draft of 9.00 m at density of 1.013 t/m .

Step 1. From hydrostatic table take the hydrostatic displacement of MV Nara corresponding to draft 9.00 m.

MV Nara SW displacement at draft 9.00 m = 28953.02 MT

Step 2. Calculate new displacement


Displacement = Tabulated displacement x new density
1.025
Displacement = 28953.02 x 1.013
1.025

Displacement = 28614 MT

Ans. Displ = 28614 MT to float at draft 9.00 m on water density 1.013

Example 3.10

What should be the displacement of the vessel MV Max Panama in order to float even keel at a draft of 12.5
3
m on dock water density of 1.015 t/m .

Step 1 From the hydrostatic table look up SW displacement of MV Max Panama at draft 12.5 m.

MV Max Panama SW Displ at draft 12.5 m = 73747 MT

40
Step 2 - calculate new displacement.

Displ in DW = SW displacement x (new density / SW density)

Displ in DW = 73747 x (1.015 / 1.025)

Displ in DW = 73027 MT

Displ = 73027 MT to float 12.5 mat DW density 1.015

Alternative Solution to Example 3.9

Change in draft due to density change = x (1.025 dd)


dd x TPC x 100
Change in draft due to water density = 28953 x (1.025 1.013)
1.013 x 35.06 x 100

Change in draft due to water density = 0.098 m

Draft at 1.013 = 9.00 m


- Change of draft = 0.098 m
Draft at 1.025 = 8.902 m

Ans. Draft 8.902 m @ 1.025 = 28612 MT Displ

Alternative solution Example 3.9

Change in draft due to density change = x (1.025 dd)


Dd x TPC x 100
Change in draft due to water density = 28953 x (1.025 1.013)
1.013 x 35.06 x 100

Change in draft due to water density = 0.098 m

Weight = sinkage x TPC at 1.013

Weight = 9.8 x 35.06 x 1.013/1.025 New Displacement = 28953 -339.6

Weight = 339.6 MT = 28613.4 MT

41
42
CHAPTER 4
THE HYDROSTATIC TABLE

IN THIS CHAPTER

USING THE HYDROSTATIC TABLE

TAKING VALUES FROM THE HYDROSTATIC TABLE

- Taking The Value Of Displacement

- Taking The Value Of Draft

- Center Of Flotation (Cf or F)

- Tons Per Centimeter Immersion (Tpc)

- Moment To Change Trim (Mtc)

- Center Of Buoyancy

43
44
USING THE HYDROSTATIC TABLE

All ships are provided with stability information and loading manuals by the shipbuilders and stamped with a
seal of approval by a classification society or flag state national authority. Loading manuals, as they are
often called, contain, among others, the hydrostatic table.

The values for each item of the ship's hydrostatic data are provided for the full range of possible mean
drafts, all based on the underwater form of the ship. The shipbuilder prepares these tables after long and
arduous calculations. Furthermore, the shipbuilders assume the ship to be on even keel with a straight
waterplane, hence, the term "static. That is, the values cannot be computed for an infinite number of wave
profiles and trimmed waterplanes, such as a ship has under dynamic sea conditions.
3
The hydrostatic values are also based on water density of 1.025 t/m , where the ship is assumed to be
3
floating. When the ship is floating in water other than a density of 1.025 t/m , some values are affected and
will vary directly proportionally to the density of the water.

Figure 4.1 -Ship's parameters that are commonly found in the hydrostatic table

HYDROSTATIC TABLE
DRAFT DISPT TPC MTC LCF LCB TKM LKM KB
10.81 45355 46.5 574.3 1.18 -4.25 11.72 234.3 5.61
10.82 45401 46.5 574.6 1.19 -4.24 11.72 234.1 5.61

1. Draftis the mean draft at center of flotation. This is also called the hydrostatic draft. Hydrostatic tables
assume the ship to be in even keel. When in even keel, the forward draft, aft draft, the mean draft and
the hydrostatic draft are the same.

2. Displacementthe weight of the ship and all its contents or the weight of SW displaced at that draft, the
3
density of SW being 1.025 t/m .

3. TPCnumber of tons to cause the mean draft to change 1 centimeter. This is the ship's TPC in SW.

4. LCBlongitudinal position center of buoyancy

5. KBvertical position of center of buoyancy

6. LCFlocation of center of flotation or the "tipping center".

7. MTCthe moment required to change the trim of the ship by one centimeter.

8. TKMheight of transverse metacenter from the keel. Also referred to as KM

9. LKMheight of longitudinal metacenter from the keel. Also referred to as KML.

45
TAKING VALUES FROM THE HYDROSTATIC TABLE

TAKING THE VALUE OF DISPLACEMENT

Since a floating ship displaces water equal to its own mass, finding the mass of the ship can be found by
multiplying the underwater volume of the ship with the density of water in which it floats. The mass of the
floating body is generally called its displacement and is normally expressed in tons.

At design stage, the underwater volume of the ship at a particular draft is calculated and converted to
3
displacement based on the density of salt water at 1025 kg/m .

Figure 4.2

From Archimedes' Principle,

Under water volume of the ship = Volume of water displaced

= Volume of displacement

The displacement can be found by:

Displacement = underwater volume of the ship x density of water displaced, or

Displacement = volume of water displaced x density of water displaced.

Using the above formula the designer of the ship calculates the underwater volume of the ship for every
draft at even keel and converts it into displacement based on the density of salt water. These are then
presented in graphical or tabular form such as in the hydrostatic table.

Using draft as reference, the displacement of the ship can be found, or vice versa. From the hydrostatic
table, the displacement taken directly against the draft is the displacement of the ship at that draft in SW.
Conversely, when the displacement of the ship is known then her draft in SW can be taken directly from the
hydrostatic table.
3
When the draft is known but the water density is not 1025 kg/m , then, the displacement must be corrected
for that density. At this displacement (corrected for density) the ship floats at that same draft.

Displ = (Tab. Displ x dd) / 1.025

The draft listed in hydrostatic tables is the draft at center of flotation. This is also called the hydrostatic draft.
Hydrostatic draft is different from the mean draft, which is the average of forward and aft drafts. Hydrostatic
tables assume the ship to be in even keel that the forward draft, aft draft, the mean draft and the hydrostatic
draft are all the same. When the ship is trimmed, the mean draft may differ from the hydrostatic draft
depending on the position of the center of flotation. The mean draft must be corrected to the hydrostatic draft
before the displacement is taken. The hydrostatic draft is also referred to as the true mean draft.

46
TAKING THE VALUE OF DRAFT

When the displacement is known the draft at SW can be taken directly from the hydrostatic table. When the
density is other than SW then the tabulated draft must be corrected, for the change in draft due to change in
density.

The change in draft can be calculated by using any one of the following;

1. FWA and BWA


2. Change of draft
3. Look-up displacement
------------------------------------------------------------------------------------------------------------------------------------------------
------------------------------------------------------------------------------------------------------------------------------------------------
------------------------------------------------------------------------------------------------------------------------------------------------
------------------------------------------------------------------------------------------------------------------------------------------------
------------------------------------------------------------------------------------------------------------------------------------------------
------------------------------------------------------------------------------------------------------------------------------------------------
------------------------------------------------------------------------------------------------------------------------------------------------
------------------------------------------------------------------------------------------------------------------------------------------------
------------------------------------------------------------------------------------------------------------------------------------------------
------------------------------------------------------------------------------------------------------------------------------------------------
------------------------------------------------------------------------------------------------------------------------------------------------
------------------------------------------------------------------------------------------------------------------------------------------------
------------------------------------------------------------------------------------------------------------------------------------------------
------------------------------------------------------------------------------------------------------------------------------------------------
------------------------------------------------------------------------------------------------------------------------------------------------
------------------------------------------------------------------------------------------------------------------------------------------------
------------------------------------------------------------------------------------------------------------------------------------------------
------------------------------------------------------------------------------------------------------------------------------------------------
------------------------------------------------------------------------------------------------------------------------------------------------
------------------------------------------------------------------------------------------------------------------------------------------------
------------------------------------------------------------------------------------------------------------------------------------------------
------------------------------------------------------------------------------------------------------------------------------------------------
------------------------------------------------------------------------------------------------------------------------------------------------
------------------------------------------------------------------------------------------------------------------------------------------------
------------------------------------------------------------------------------------------------------------------------------------------------
------------------------------------------------------------------------------------------------------------------------------------------------
------------------------------------------------------------------------------------------------------------------------------------------------
------------------------------------------------------------------------------------------------------------------------------------------------
------------------------------------------------------------------------------------------------------------------------------------------------
------------------------------------------------------------------------------------------------------------------------------------------------
------------------------------------------------------------------------------------------------------------------------------------------------
------------------------------------------------------------------------------------------------------------------------------------------------
------------------------------------------------------------------------------------------------------------------------------------------------
------------------------------------------------------------------------------------------------------------------------------------------------
------------------------------------------------------------------------------------------------------------------------------------------------
------------------------------------------------------------------------------------------------------------------------------------------------
------------------------------------------------------------------------------------------------------------------------------------------------
------------------------------------------------------------------------------------------------------------------------------------------------
------------------------------------------------------------------------------------------------------------------------------------------------
------------------------------------------------------------------------------------------------------------------------------------------------
------------------------------------------------------------------------------------------------------------------------------------------------

47
CENTER OF FLOTATION (CF or F)

The center of flotation for a ship is the centroid of the waterplane at that draft, and the point where the ship
heels and trims. The center of flotation is also known as the "tipping center". It is located on the centerline
but may or may not be at the midship point.

Its position alters constantly with changes of draft and trim, which may be forward or abaft of amidships.

Figure 4.3 -The longitudinal position of center of flotation.

Longitudinal Center of Flotation (LCF)

LCF is the distance of the center of floatation from a reference point measured longitudinally. The reference
point can be amidships or aft perpendicular and in some instances the forward perpendicular.

The position of the center of floatation can be found by the principle of moment. On board ships hydrostatic
table provides the corresponding value of the LCF at various drafts.

Moderate weight loaded directly above the center of flotation,


causes the ship to immerse to a new draft but the trim stays the
same. This is called bodily sinkage. The sinkage is calculated by
dividing the weight by TPC.

One of the purposes of the LCF in the trim and stability calculation is to correctly determine the distribution of
trim to the forward and aft drafts.

Since the vessel pivots at the LCF, the distribution of trim forward and aft are not equal but proportionate to
its distance from the perpendiculars.

When the ship changes trim, the greater the distance of LCF from amidships the more pronounced is the
difference in the change of draft forward than in aft draft. This causes the difference In the arithmetic mean
draft and the hydrostatic draft.

NOTE: See also distribution of trim between forward and aft draft at page, and The True Mean Draft
at page.

48
TONS PER CENTIMETER IMMERSION (TPC)

Changes in displacement cause changes in draft. The number of tons, which must be loaded, or discharged
to cause the mean draft of the ship to change 1 cm, is termed the tons per centimeter immersion (TPC).

TPC is applied to the calculation of draft changes due to added or discharged weights.

sinkage or rise = cargo loaded or discharge / TPC

where sinkage or rise is the difference between the initial mean draft and final mean draft expressed in
centimeters.

The Derivation of TPC

The value of TPC is equal to the mass of water displaced whose volume is the area of waterplane (Aw)
times 1 cm.

TPC = Aw x 1.025 / 100

Figure 4.4 -The water plane of a ship

2
Volume of water displaced = Aw [m ] x 1cm or,
Volume of water displaced = Aw/100
TPC = weight of water displaced

Weight = volume x density


Weight of water displaced = (Aw/100) x density
TPC = (Aw/100) x density
If ship is floating in salt water, density = 1.025t/m3
TPC = Aw x 1.025/100

In the hydrostatic table, the value of the TPC is provided for each draft. The tabulated TPC is the value for
the density of seawater at 1.025t/m3. When the ship is floating in water of different density, the TPC value
must be corrected for the density of water.

49
Example 4.1

The areas of waterplane of a vessel at following drafts are as listed below. Find the TPC of the vessel at the
listed drafts.

Waterplane Area TPC


Draft (Aw)
2
4.0 m 5660 m 58.015
2
5.0 m 5733 m 58.763
6.0 m 5795 m2 59.399

TPC = Aw x 1.025 / 100 TPC = Aw x 1.025 / 100 TPC = Aw x 1.025 / 100


= 5660 x 1.025 / 100 = 5733 x 1.025 / 100 = 5795 x 1.025 / 100
= 58.015 MT = 58.763 MT = 59.399 MT

Since the TPC value is dependent on the area of waterplane, then TPC varies with draft for a ship -shaped
vessel. As the ship's draft increases, hence her waterplane, the value of TPC also increases. Usually the
value of TPC used in calculations of cargo loaded or discharged is always approximate to facilitate solution
and for practical purposes.

TPC and Density

The value of TPC normally quoted (from hydrostatic table) is for a ship floating in salt water of density 1.025
3
tons/m . It may be necessary to correct this figure if the ship is floating in water of density other than 1.025
tons/m3.

TPC DW = TPC x dd
1.025
Example 4.2

The vessel MV Nara is to load to a draft of 9.00 m. She presently floats to a draft of 8.65 m at water density
3
of 1.012 t/m . How much more cargo may she load to bring her to her completion draft?

TPC DW = (TPC x dd) / 1.025 Final draft = 9.00 m


Present draft = 8.65 m
TPC 1012 = (34.79 x 1.012) / 1.025 Sinkage = 0.35 m

TPC 1012 = 34.35 cargo to load = sinkage x TPC x 100


= 0.35 x 34.35 x 100
= 1202 MT

Ans. cargo to load = 1202 MT

NOTE: For large changes in draft, the difference between displacements at the respective mean
drafts should be used to determine the amount of cargo to be loaded or discharged.

50
Example 4.3

MV Nara is loaded to her summer mark, and shall sail to discharge port. Bunkers and all other consumption
are calculated to 610 MT. What is the draft when she arrives at discharge port?

Alternative solution using TPC:


Summer displacement = 30884 MT
Consumptions = 610 MT Rise = weight / (TPC x 100)
Displ at disport = 30274 MT
Rise = 610 / (35.46 x 100)
From hydrostatic table
Draft = 9.377 m Rise = 0.172 m

Note: consumption is subtracted from


displacement at departure. Dep. Draft = 9.550 m
Rise = 0.172 m
Arr. Draft = 9.378 m
Example 4.4

A vessel is to load 900 MT of weight. She is presently floating in SW to a draft of 10.52m and TPC is 44.6
MT/cm. Find the draft at completion of loading.

Solution using TPC:

Step 1. Find sinkage Step 2. Find Final draft:

Sinkage = weight Initial draft = 10.52 m


TPC x 100 Sinkage = 0.202 m
Sinkage = 900
44.6 x 100 Ans. Final Draft = 10.722 m

Sinkage = 0.202 m

Example 4.5

A vessel is to complete loading to a mean draft of 9.55 m. She is presently floating to a draft of 9.32m, and
her TPC is 35. 29 MT. Find how much more cargo to load.

Step 1.
Final draft = 9.55 m
-Present draft = 9.32 m
Sinkage = 0.23 m

Step 2.
Cargo to load = sinkage x TPC x 100
= 0.23 x 35.29
= 811.7 MT

Ans. Cargo to load = 811.7 MT

51
Example 4.6

MV Max Panama must arrive at a discharge port with an SW draft of 12.50m. Her consumption during the
voyage from the loading port to discharge port is 467 MT. What should be her SW draft at departure from
loading port?

Arr. Displacement at 12.5 m = 73747 MT


Consumption = 467 MT
Dep. displacement = 74214 MT

Ans. Equivalent draft at SW = 12.573m

Alternative solution using TPC:

Rise = weight Dep. Draft = 12.50 m


TPC x 100 Rise = 0.729 m
= 467
64.0 x 100 Arr. Draft = 12.573 m
Rise = 0.729 m

Example 4.7
3
The vessel MV Max Panama is floating at a draft of 13.25 m in a water density 1.008 t/m . How much more
cargo may she load to bring her to her completion draft of 13.50m?

TPC DW = TPC x dd Final draft = 13.50 m


1.025 -Initial draft = 13.25 m
= 0.25 m
TPC 1012 = 64.4 x 1.008 cargo to load = sinkage x TPC x 100
1.025 = 0.25 x 63.33 x 100
TPC 1012 = 63.33 = 1583.25 MT

Ans. cargo to load = 1583.25 MT

MOMENT TO CHANGE TRIM 1 CM (MTC)

The moment about the center of flotation necessary to change the trim of a ship by 1 centimeter is
commonly called the MTC. MTC is a convenient figure to quote to determine how easy the ship is to trim.
Dividing the trimming moment of the ship by MTC gives us the trim or the change of trim.

The MTC is not a constant figure at all drafts and in all trims. It alters with the change in area of the
waterplane.

The value of MTC may be calculated by the formula:

MTC = Displ x GML


100 x LPP

This formula is not very convenient to use in practice that the value of MTC for each draft is included in the
ship's hydrostatic data.

52
The formula is derived from:

MTC and Density

The tabulated MTC in the hydrostatic table is for a ship floating in seawater. When the density is other than
SW then the MTC value must be corrected proportionate to the density of water.

MTC = Tab. MTC x dd


1.025

NOTE: When using MTC in trim calculations, the value of MTC should always be selected for deeper
draft, that is, the final draft when loading, and the initial draft when unloading.

53
CENTER OF BUOYANCY

The center of buoyancy is the point at which all the vertical upward thrusts generated by the hydrostatic
pressures can be considered to act. It can be said that the ship is lifted at this point. This is also the center of
the volume of the ship under the waterline.

The position of the center of buoyancy changes for every change in draft and trim. To be able to find the
center of buoyancy, it must be calculated for each water plane at fixed intervals. The center of buoyancy at a
certain draft is the resultant center of buoyancy of all water plane areas up to that draft. This average
position of B can be found by Simpson's formula. The calculation of B can be in longitudinal or vertical
direction.

In the hydrostatic tables, the center of buoyancy for upright and even keel is provided in terms of LCB and
KB.

Longitudinal Center Of Buoyancy (LCB)

One way of describing the location of the center of buoyancy is by its longitudinal distance from a reference
point. The reference points can be amidships, the aft or forward perpendicular.

The distance of center of buoyancy measured longitudinally from a reference is called the longitudinal center
of buoyancy (LCB).

Using the draft as argument, the values of LCB for the ship on even keel are found in the hydrostatic table.

Height of Center of Buoyancy (KB)

The keel (K) is the reference point for measuring in vertical direction the distance of B. The distance
between K and B is called the KB.

Figure 4.7

For a box-shaped vessel, the value of KB can be approximated as half the draft. The value of LCB is used in
calculating the trim while the value of KB is used in calculating the transverse stability of ship.

NOTE: Being the center of the submerged portion, the location of center of buoyancy is dependent
on draft. As the draft changes the location of center of buoyancy also changes.

54
METACENTRIC RADIUS (BM)

When the ship is inclined through small angles of heel, the lines of buoyancy force intersect at a point called
the metacenter. For an object whose breadth at the waterline does not change, such as a homogenous log,
the point of intersection is always the same.

Figure 4.8

As the ship is inclined, the center of buoyancy moves in an arc as it continues to seek the geometric center
of the underwater volume of the ship. At the center of this arc is the "metacenter". The distance between the
center of buoyancy and the metacenter is called the metacentric radius.

Figure 4.9

For a ship shape the points of intersection vary after the ship has inclined by a small angle. This is so
because the breadth at the waterline has changed. The successive lines of force of buoyancy therefore
intersect at different locations. Thus the metacenter of a ship is considered to be at the same location only at
small angles of inclination but not when inclined at large angles.

Figure 4.10.

Theoretically, the distance BM at small angles can be calculated by:

For box shaped vessel using metric units:


3 2
BM = Length x Breadth or = Breadth
12 x Length x Breadth x Draft 12 x Draft

55
HEIGHT OF METACENTER (KM)

The location of the metacenter can be described by its distance from the keel. The vertical distance of
metacenter from keel is called height of metacenter or KM. KM is found by:

In theoretical study of box-shape vessel;


KB = the draft
2
BM = B /(12 x draft)
KM = KB + BM

Figure 4.11

The location of M is important when assessing the initial stability of the vessel. The initial stability is
determined by the location of G and that of M.

The hydrostatic table provides for the value of KM.

Longitudinal Height of Metacenter (LKM)

The KM previously described is for the transverse axis of the waterplane. The position of M when considered
through the longitudinal axis of the water plane is called the longitudinal KM or LKM.

LKM is used for finding the value of MTCs as shown in its derivations.

LKM is found by:

LKM = KB + BM L

BML is the BM or metacentric radius when considered through the longitudinal axis. In the basic formula for
BM, the breadth of the ship is considered as length, and the length is considered as the breadth.

Comparing it to the transverse BM, the longitudinal BM for a box shape vessel is:
3
BML = Breadth x Length
12 x Breadth x Length x Draft

Or;
BML = Lenght2
12 x Draft

The BML so found is commensurate to the length of the ship. The longitudinal metacenter being so high up
makes the ship longitudinally stable.

56
Interpolating values from hydrostatic tables

Data on hydrostatic tables are presented in tabular form and are usually necessary to find values which fall
between the values that are listed in the table. The arithmetic process in doing this is called interpolation.

For example, consider a portion of the hydrostatic tables of MV MAX PANAMA:

Draft Displacement TPC


12.68 74899 64.1
12.69 74963 64.1

What is the displacement for a draft of 12.683 m?

To define a formula, the tabular values can be represented as follows:

Draft Displacement TPC


(Draft 1 ) 12.68 74899 (Displ 1+ ) 64.1
(Draft x ) 12.683 (Displx )
(Displ 2 12.69 74963 (Displ 2 ) 64.1

Where Draft x is the reference for draft and Displ x is the interpolated value of displacement, the formula for
interpolation is:

DispI x = (Draft 1 -Draft x ) x ( Displ 1 -Displ 2 ) + Displ 1


(Draft 2 - Draft 1 )

Substituting with numerical values:

DispI x = (12.68 -12.683) x ( 74899 -74963 ) + 74899


(12.69 -12.68)

Displ x = (- 0.003) x ( -64 ) + 74899


(0.0.1)

Displ x = 19.2 + 74899

Displ x = 74918.2 MT

Ans. Displacement at draft 12.683 = 74918.2 MT

As an alternative for finding displacement that fall between tabulated drafts, a practical solution can be used
by modifying the interpolation formula, as:

Displ x = (0.3 x 64.1) + 74899


= 19.23 + 74899
= 74918.23 MT

57
Example 4.8

What is the displacement of MV Max Panama if the hydrostatic draft is 5.406 m?

Tabulated Displ at 5.40 m = 30094 MT TPC = 59.0


Increment = 0.6 x 59.0 = 35.4 MT
Displacement = 30129.4 MT

Example 4.9

From MV MAX PANAMA, what is the hydrostatic draft for displacement of 74932 MT?

Draught Displacement
Draft 1 12.68 74899 DisPl 1
Draftx ? 74932 Displx
Draft 2 12.69 74963 Displ 2

Draft = (Displ 1 - Displx) x ( Draft 1 - Draft 2 ) + Draft 1


(Displ 2 Displ 1 )
Draft x = (74899 -74932) x (12.68 -12.69) + 12.68
(74963 -74899)

Draft x = 12.685 m

Alternatively,
Draft x = (disp. diff) x (0.01) + tab. draft
TPC
= 33 x 0.01 + 12.68
64.1

Draft x = 12.685 m

LONGITUDINAL DISTANCES

Points whose locations are indicated by a distance measured longitudinally from a reference line are:

1. Light ship LCG, hold LCG, or Compartments LCG.

1. LCB -The distance of center of buoyancy


2. LCF -The distance of center of floatation

Distances such as the values of LCB and LCF are measured horizontally from a reference plane, which is
usually amidships. The direction of measurement either forward or aft of the reference plane amidships is
assigned different signs.

Some hydrostatic tables, however, are based on distances from aft perpendicular. Such distances do not
have signs.

For distances measured amidship the sign convention normally is:

1. (-) sign for forward of amidships

2. (+) sign for aft of amidships

58
Note: The sign convention is reversed in some loading manuals, so it is always necessary to be sure
which convention is adopted in the manual.

Figure 4.13 - Normal sign convention for values measured from amidships.

NOTE:
You must be familiar with the different ways of measuring longitudinal distances, as it affects the
formula and methods used in trim calculation. Knowing how the formula was derived helps in
identifying the needed changes in the formula.

Comparing the two systems of measuring longitudinal distances as used in trim calculations.

Figure 4.14 - Distances of centroids measured longitudinally from amidships [x].

HYDROSTATIC TABLE

DRAFT DISPT TPC MTC LCF LCB TKM LKM KB


10.88 45680 46.5 576.2 1.25 -4.21 11.73 233.4 5.65
10.89 45727 46.5 576.4 1.26 -4.20 11.73 233.3 5.65

Figure 4.15 -Distances of centroids measured longitudinally from aft perpendicular [AP]. The ship's LPP is
180 meters.

59
HYDROSTATIC TABLE

DRAFT DISPT TPC MTC LCF LCB TKM LKM KB


10.88 45680 46.5 576.2 88.75 94.20 11.73 233.4 5.65
10.89 45727 46.5 576.4 88.74 94.21 11.73 233.3 5.65

Converting distances measured from amidship to distances from aft perpendicular and vice versa.

Dist from AP = LBP - Dist from amidship

Example 4.10

The ship's LPP is 179.5 m and the LCF is 1.196 m aft of amidship. How far is the CF from aft perpendicular?

LCF [AP] = LBP - LCF [x]


LCF [AP] = (179.5) - (1.196)
LCF [AP] = 88.554 m

Converting distances measured from aft perpendicular to distances measured from amidship.

Dist from amidship = LBP - Dist from AP

Example 4.11

The ship's LBP is 179.5 m and the LCB is 94.943 Find the LCB from amidship.

LCB[x] = (179.5) -94.943 m


LCB[x] = -5.193 m
(LCB is - 5.193 m forward of amidship)

60
The True Mean Draft

The true mean draft, also referred to as the hydrostatic draft, is the draft of the vessel at the center of
flotation.

When the ship is in even keel the forward and aft drafts, the arithmetical mean draft, and the hydrostatic
mean draft, are the same.

Since the vessel trims about the center of flotation, the arithmetical mean draft and the hydrostatic draft may
not be equal.

Arithmetical mean draft or mean draft is the average of forward and aft drafts. Mean draft is obtained by
adding the forward and aft drafts and dividing the sum by two. This method assumes that the draft marks are
equidistant from amidships, that is they are located on the forward and after perpendiculars.

Figure 4.16 -The draft at LCF

In taking particulars from the hydrostatic table, the hydrostatic draft should be used.

The mean draft can be corrected to draft at LCF by the formula;

For LCF measured from amidship;

Hydrostatic draft = Mean draft + LCF [x] x Trim


LPP

Or,
Hydrostatic draft = Aft draft - LPP LCF (x) x Trim
LPP

For LCF measured from aft perpendicular;

Hydorstatic draft = Aft draft - LCF [AP] x Trim


LPP

In general, the farther the CF is from amidship and the greater the trim, the greater the difference of the
mean draft and hydrostatic draft.

61
Example 4.12

MV Max Panama is floating at a draft (at perpendiculars) of 7.89 m fwd and 8.73 m aft.

Calculate her hydrostatic draft and find her displacement (corrected for trim).

Hydorstatic draft = Mean draft + LCF [x] x Trim


LPP

= 8.31 + -6.42 x 0.84


215
= 8.285 m

From hydrostatic table, displacement 8.28 = 47372 MT TPC = 60.9

Increment = 0.5 x 60.9 = 30.45 MT


Displacement = 47402.45 MT

Example 4.13

MV Max Panama is floating at a draft (at perpendiculars) fwd 12.60 m and aft 13.50 m.

Calculate and compare her mean draft and hydrostatic draft. Find her displacement.

Hydrostatic draft = Mean draft + LCF [x] x Trim


LPP
Hydrostatic draft = 13.05 + 0.15 x 0.9
215

= 13.0506 m

From hydrostatic table, displacement at 13.05 = 77273 MT TPC = 64.3

Increment = (0.06 x 64.3) = 3.86 MT


Displacement = 77276.86 MT

Example 4.14

A Mini-Bulker with LPP = 89.95m is floating at draft fwd = 5.41 m and draft aft = 6.16 m. LCF = 1.75 m aft of
amidship. TPC = 14.4 MT. Find the hydrostatic draft and her displacement.

Hydrostatic draft = 5.785 + 1.75 x 0.75


89.95
Hydrostatic draft = 5.80 m

From the hydrostatic table, displacement at 5.80 = 7257 MT

62
CHAPTER 5
QUANTITY OF CARGO
IN THIS CHAPTER

THE AMOUNT OF CARGO THE SHIP CAN CARRY

DEADWEIGHT LIMITED BY DRAFT

DRAFT LIMIT IMPOSED BY THE LOAD LINE RULES

o The Load Lines

o Finding The Amount Of Cargo The Ship Can Load When Limited By Load Line
Regulations

o The Zone Allowance

DRAFT LIMIT

o At Departure Load Port

o At Destination

WHEN AMOUNT OF CARGO IS LIMITED BY VOLUME

Volume Of Cargo Spaces

Stowage Factor (SF)

63
64
THE AMOUNT OF CARGO THE SHIP CAN CARRY

Normally it is on the ship's ballast voyage that the


new transport order is received. There is a
continuous communication between charterer,
shipper and the ship regarding the loading order.

One of the first questions that must be resolved is:


how much cargo can the ship load? It is important to
answer this message as soon as possible, because
there are many companies competing in the market
with available ships. The ship must be able to readily
reply on this message to avoid the cargoes from
slipping away.

There are many details that the ship officer must


look into before he can decide the quantity of cargo
to load. Familiarity with such details is important to
be able to determine the suitable arrangements for
the order and effectively complete the voyage.

To determine the amount of cargo the ship can carry, the different stages of the voyage must be monitored
to determine the limiting point, which may restrict the amount of cargo the ship is able to carry.

The amount of cargo the ship is able to carry may depend on:

1. The deadweight available


Limited by draft:
a. Load line
b. Draft restrictions in ports and canals
i. Draft limit at load port
ii. Draft limit at destination or discharge port

2. The volume of space for the cargo

3. When the tonnage to be carried is stipulated in the charter party

In any case it may be necessary to:

1. Determine the weights already on board at every stage of the voyage.


2. Identify any other limiting point, which will further restrict the amount of cargo which can be loaded.

DEADWEIGHT LIMITED BY DRAFT

Draft limits may restrict the amount of cargo the ship can carry since the draft directly affects the
displacement, hence, the deadweight.

Draft limits may be:

1. Limits imposed by the load line rulesdependent upon the geographical zone and time of the year,
the load line rules impose restrictions on the ship's freeboard and draft amidship but impose no
restrictions upon the ship's trim.

65
2. Limits imposed by the depths and densities of water in ports and canalsships must not exceed
such draft limits to operate safely in these ports and the maximum draft and displacement can be
achieved only if the ship is in even keel

Figure 5.2 -Limits imposed by depths and densities of water in ports and canals
================================================================================

DRAFT LIMITS IMPOSED BY THE LOAD LINE RULES

All ships are subject to the International Load Line Regulations and each ship is capable of carrying a
certain number of tons of deadweight while conforming to these rules. Load line marks on the ship define the
maximum draft to which the ship may float to conform with the minimum required freeboard, in all sea areas
and in rivers and harbors where the density of the water in which the ship floats is not equal to the density of
salt water.

The Load Line

The objective of the load line is to clearly indicate a minimum freeboard to ensure that the ship is seaworthy
when loaded. The minimum freeboard provides the ship with reserve buoyancy, that in the event of/or
following damage (due to flooding), the ship shall have adequate stability and buoyancy to keep the ship
afloat indefinitely, or at least long enough for people to get off.

When a ship is built, appropriate assigning authority assigns a load line to her. This load line is cut into the
ship's shell plating on either side amidships and another mark cut in above the load line on a level with the
uppermost continuous deck.

Draft marks are cut on either bow and on each side of the rudderpost and in the vicinity of the load line. At
the same time the assigning authority authorizes the promulgation of information bearing directly on the
ship's load line and her carrying capacity in certain conditions of loading.

Perhaps the most important information is that which concerns the drafts and freeboards to which the ship
may be loaded to conform to the various marks on her load line. This data is laid out in the Capacity Plan by
the builder's drawing office staff, and further information concerning the ship's stability particulars is
incorporated in the drawings of her Displacement Curves.

66
Figure 5.3 -The dimensions of letters and lines used in the Load line mark.

Indicated by the upper edge of the line which passes


through the center of the ring and also by a line marked
1 Summer Load Line
S. (Maximum draft when floating in SW within the
Summer Zone)
Indicated by the upper edge of a line marked WNA. A line
1/48th of the summer moulded draught below the summer
2 Winter Load Line
load line. (Maximum draft when floating in SW within the
Winter Zone)
Indicated by the upper edge of a line marked WNA. (This
only applies to vessels less than 100 long.) A line 50 mm
3 Winter North Atlantic
below the winter load line. (Maximum draft when floating
in SW within the Winter North Atlantic Zone)
Indicated by the upper edge of a line marked T. A line
1/48th of the summer moulded draft above the summer
4 Tropical Load Line
load line. (Maximum draft when floating in SW within the
Tropical Zone)
5. Fresh Water Indicated by the upper edge of a line
5 Fresh Water Load Line marked F. A line has drawn a distance equal to the fresh
water allowance above the summer load line.
Tropical Fresh indicated by the upper edge of a line
6 Tropical Fresh Water Load Line marked TF. Water Load A line drawn a .distance equal to
the fresh water allowance above the tropical load line.

Account must be taken of the ship's proposed voyage, for the Load Line Regulations stipulate to which
marks she may be loaded in particular areas at specified periods of the year.

A load line chart (copy attached) defines those areas and the periods during which the different load line
marks are operative. This chart must be consulted before making any estimate of the deadweight tonnage
(ex. cargo to load) which the ship may have on board.

67
Figure 5.4 -Freeboard and Load line Mark of "MV NARA".

Figure 5.5 -Table of Load line Particulars for the "MV NARA"

Tropical
Season Summer Fresh Tropical Winter
Fresh
Freeboard M 4.156 3.937 3.957 3.738 4.355
Draft M 9.550 9.769 9.749 9.968 9.351
Displacement T 30884 30886 31588 31576 30183
Lightship T 6772 6772 6772 6772 6772
Deadweight T 24112 24114 24816 24804 23411

Figure 5.6 -Table of Timber Load line Particulars of MV NARA -Complete the table below using the MV
NARA's hydrostatic Table.

Tropical
Season Summer Fresh Tropical Winter L.W.N.A.
Fresh
Freeboard M 3.706 3.478 3.498 3.27 3.984 4.355
Draft M 10.00 10.228 10.208 10.436 9.722 9.351
Displacement T
Lightship T
Deadweight T

68
Sailing Draft when deadweight is limited by the Load Line Regulations.

The following chart serves as a guide to determine the displacement or draft of ship at departure when
governed by load line regulations.

Project voyage Displacement at Departure Sailing Draft at Departure


1. Tropical Zone to Tropical Displacement Tropical load line. If density is not
Tropical Zone SW, apply Cod.
2. Summer Zone to Summer Displacement Summer load line. If density is not
Summer Zone SW, apply Cod.
3. Winter Zone to Winter Displacement Winter load line. If density is not SW,
Winter Zone apply Cod.
4. Winter Zone to Winter Displacement Winter load line. If density is not SW,
Summer Zone apply Cod.
5. Winter Zone to Winter displacement Winter load line. If density is not SW,
Tropical Zone apply Cod.
6. Summer Zone to Summer displacement Summer load line. If density is not
Tropical Zone SW, apply Cod.
7. Tropical Zone to Summer displacement plus zone Summer load line plus Zone
Summer Zone allowance but not greater than Allowance but not greater than
Tropical displacement tropical load line. If density is not
SW, apply Cod.
8. Tropical Zone to Winter displacement plus zone Winter load line plus Zone allowance
Winter Zone allowance but not greater than but not greater than tropical load line.
Tropical displacement If density is not SW, apply Cod.
9. Summer Zone to Winter displacement plus zone Winter load line plus Zone allowance
Winter Zone allowance but not greater than but not greater than tropical load line.
Summer displacement If density is not SW, apply Cod.

Note: Cod = change of draft due to change in water density.

Cod must be applied as appropriate. For sinkage, Cod is plus. For rise Cod is minus.

Change of draft due to density change can be found by:

1. FWA

2. DWA

3. Change of draft formula

The final draft due to density change can also be found by using the hydrostatic table.

69
FINDING THE AMOUNT OF CARGO THE SHIP CAN LOAD WHEN LIMITED BY LOAD LINE
REGULATIONS

The International Convention on Load Line 1966, Article 12 states that;

1. Except as provided in paragraphs 2 and 3 of this article, the appropriate load lines on the
sides of the ship corresponding to the season of the year and the zone or area in which the
ship shall not be submerged at any time when the ship puts to sea, during the voyage or on
arrival.

2. When a ship is in fresh water of unit density the appropriate load line may be submerged by
the amount of the fresh water allowance shown on the International Load Line Certificate
(1966). Where the density is other than unit, an allowance shall be made proportional to the
difference between 1.025 and the actual density.

3. When a ship departs from a port situated on a river or inland waters, deeper loading shall be
permitted corresponding to the weight of fuel and all other materials required for consumption
between the point of departure and the sea.

Example 5.1

A mini-bulker was ordered to load full to her mark at a port located in a summer zone. Her summer
displacement of 7573 MT corresponds to a SW draft of 6.018 m and TPC of 14.57 t/cm. Her light
displacement is 1771 MT. Remaining on board on departure from loading port are as follows:

IFO = 350 MT FW = 152 MT


MOO = 69 MT Ballast = nil
LO = 7 MT Constant = 95 MT

Throughout her voyage her draft is limited only by summer load line.

Find:
1. The quantity of cargo to load.
2. The maximum permissible draft if she is floating in water of density 1.008 t/m3.
3. Her draft if she is floating in sea water (density 1.025 t/m3).
4. If she is about to complete loading and floating at a draft of 5.90 m in water density 1.008
3
t/m . Find quantity of cargo remaining to load.

Solution:
1. Determine the governing zone: summer zone
2. Know the draft and displacement at governing zone. (Since her draft is limited by the zone
at the start of the voyage, then she must be within the limit on departure from loading port)

Dep. Draft SW = 6.018 m


Displacement = 7573 mtons

3. Find the quantity of cargo that can be lifted.

Fuel Oil = 350 Displacement = 7573 mtons


Diesel Oil = 69 Light displacement = 1771
LO = 7
Fresh Water = 152 Deadweight = 5802 mtons
Ballast = 0 Weights on board = 673 mtons
Constant = 95 Max. cargo = 5129 mtons
Weights on board = 673 mtons Ans: Loadable cargo = 5129 mtons

70
4. Find the departure draft DW density

Look up displacement = actual displ x 1.025


dd

Look up displacement = 7573 x 1.025


1.008
Look up displacement = 7701 mtons

Draft at DW 1.008 = 6.106 m

5. Sinkage = Final draft -Present draft TPC DW = TPC x dd


= 6.106-5.90 1.025
= 0.206 m TPC 1008 = 14.57 x 1.008
1.025

rem. cgo to load = sinkage x TPC1008 x 100


= 0.206 x 14.328 x 100 TPC 1008 = 14.328
= 295.2
Ans. rem.cgo to load = 295 mtons

Alternative solution to step 5:

SW Displ at draft 5.90 = 7401 mtons Final Displ = 7573 mtons


N. Dlspl = 7401 x 1.008 Present Displ = 7278 mtons
1.025
Cargo rem. = 295 mtons
N. Displ = 7278 mtons

Example 5.2

3
MV Nara is to load full to her tropical draft. Density of dock water in load port is 1.014 t/m . She has the
following weights remaining onboard on departure from load port:

IFO = 1193 MT FW = 295 M


MOO = 18 MT Ballast = 45 MT
LO = 12 MT Constant = 210 MT

Her draft is limited only by tropical load line throughout her voyage.

Find:
1. The quantity of maximum amount of cargo to load assuming there is sufficient fuel for the voyage.
3
2. The maximum permissible draft if she is floating in water of density 1.014 t/m .
3
3. Her draft if she then proceeds to sea (density 1.025 t/m ).
4. If she is about to complete loading and floating at a draft of 9.63 m in water density 1.014 t/m3, find
quantity of cargo remaining to load.

Solution:

1. Determine the governing zone = Tropical Zone


2. Know the draft and displacement at governing zone. (Since the zone limits her draft at the start of
the voyage, then she must be within the limit on departure from loading port.)

Dep. Draft SW = 9.749 m Dep. Displacement = 31588 mtons

71
3. Find the quantity of cargo that can be lifted.
Fuel Oil = 1193 mtons Dep. Displacement = 31588
Diesel Oil = 18 Light displacement = 6772
La = 12 Deadweight = 24816 .
Fresh Water = 295 Weights on board = 1773
Ballast = 45 Max. cargo = 23043
Constant = 210 Sag Allowance (nil) =
Weights on board = 1773 mtons
Ans: Loadable cargo = 23043 mtons

4. Find the departure draft OW density

Look up displacement = actual displ x 1.025


dd
Look up displacement = 3158 x 1.025
1.014

Look up displacement = 31930.6

5. Take the draft from hydrostatic table using look-up displacement

Dep. Draft DW 1 .O14 = 9.846 m

6. Sinkage = Dep. Draft DW1.O14 -Present draft = 9.846 -9.63 = 0.216 m

7. Rem. Cargo to load = Sinkage x TPC x 100 x dd = 0.216 x 35.52 x 100 x 1.014
1.025 1.025
= 759 mtons
Alternative solution:

SW Displ at draft 9.63 = 31166.5 mtons Final Displ = 31588 mtons

N. Dlspl = 31126.5 x 1.014 Present Displ = 30832 mtons


1.025
N. Displ = 30832 mtons Cargo rem. = 756 mtons

Notice that there is a difference of 3 tons between the two answers. In practice both are considered correct.

Example 5.3 -Consumption Allowance

MV Nara is to load cargo from Montreal to Cartagena during the winter season. The North Atlantic Winter
Seasonal Zone commences on 1st of November. Distance to reach open sea from Montreal is 428 nm. MV
Nara steams at 11 knots.

IFO = 1387 MT FW = 255 MT Consumption:


MOO = 21 MT Ballast = NIL IFO = 25 MT/day
LO = 14 MT Constant = 255 MT MD = 0.5 MT/day
FW = 4 MT/day

Find:

1. The quantity of cargo to load.


3
2. The maximum permissible draft if she is floating in water of density 1.005 t/m .
3
3. Her draft if she then proceeds to sea (density 1.025 t/m ).

72
4. If she is about to complete loading and floating at a draft of 9.36 m in water density 1.005 t/m3. Find
quantity of cargo remaining to load.

Solution:

1. Determine the governing zone = Winter Zone

2. Consumption calculation

IFO consumption/day = 25 mtons


MDO consumption/day = 0.5 mtons
FW consumption/day = 4 mtons
Total cons/day = 29.5 mtons
No. of days = 1.6
consumption allowance = 47.2 mtons

3. Find the quantity of cargo that can be lifted.

Fuel Oil = 1387 mtons


Diesel Oil = 21
LO = 14
Fresh Water = 255
Ballast = 0
Constant = 255
Weights on board = 1932 mtons

Winter Displacement = 30183


Cons. Allowance = 47
True Displ at Dep. = 30230
Light displacement = 6772
Deadweight = 23458
Weights on board = 1932
Max. cargo = 21526

Ans: Loadable cargo = 21479 mtons

4. Know the draft and displacement at limiting zone.

Dep. Draft SW = 9.364 m Dep. Displacement = 30230 mtons

5. Find the departure draft OW density

Look up displacement = actual displ x 1.025


Dd
Look up displacement = 30230 x 1.025
1.005
Look up displacement = 30831.6

6. Take the draft from hydrostatic table using look-up displacement

Dep. Draft DW 1.005 = 9.535 m


7. Sinkage = Dep. Draft DW 1.005 - Present draft = 9.535 - 9.36 = 0.175 m

Rem. Cargo to load = Sinkage x TPC x 100 x dd = - 0.175 x 35.52 x 100 x 1.005
1.025 1.025
= 609 mtons

73
THE ZONE ALLOWANCE

A ship during her projected voyage may enter and leave several zones where different load lines apply. For
instance, a ship may be loading in Tropical Zone but will be at a Summer Zone after several days of
passage. While the ship is permitted to load to Tropical Mark on departure, at the time when she has to pass
the summer zone, she must not be deeper than her authorized summer draft. Unless the weight of bunkers,
water and stores consumed during her passage from tropical to summer zone is at least equal to the
difference between tropical and summer deadweights the ship cannot, therefore, load to tropical marks. In
such a case the deadweight leaving the loading port would be summer deadweight plus the weight of
bunkers, water and stores estimated consumed before entering the limiting zone. This weight estimated to
be consumed is known as the Zone Consumption Allowance.

Figure 5.7

Remember:

Zone Allowance = consumption during the passage from a deeper zone to limiting zone.

74
Example 5.4 -Zone Allowance Calculation

The Master of MV NARA has been instructed to load at Alexandria destined for U.K. She is to leave
Alexandra with 345 MT F.O., 90 MT DO, 65 MT FW, 12 MT L.O., her constant is 185 MT. Leaving
Alexandria on 25th February and steaming 14 knots she will pass Cape Torinana 7 days later consuming 22
tons of FO, 2 tons DO and 6 tons FW per day. Determine her deadweight available for cargo if she is to load
full in Alexandria.

1. Identify the different zones of the voyage. Alexandria -summer zone / UK - winter zone
2. Identify the governing zone -winter zone
3. Calculate the total non-cargo weights to be deducted from the deadweight.
4. Calculate the zone consumption allowance to be added to the limiting zone.

ROB Cons ROB@


Steaming Steaming ROB
Departure Weight per Cons. Cape Cons.
Time Time Arr (UK)
(Alexandria) day Torinana
HFO 345 -22 7 -154.0 191.0
MDO 90 -2 7 -14.0 76.0
LO 12 7 12.0
FW 65 -6 7 -42.0 23.0
BLST 0 7 0.0
CONST 185 7 185.0
TOTAL 697 7 -210.0 487.0

5. Apply the zone consumption allowance in the calculation of the deadweight available for cargo.

Displacement (at governing zone) = 30183 MT


Zone Allowance = 210
True Displacement at Departure = 30393
Light displacement = 6772_________
Deadweight = 23621
Weights on board = 697__________
Maximum cargo = 22924 MT
------------------------------------------------------------------------------------------------------------------------------------------------
------------------------------------------------------------------------------------------------------------------------------------------------
------------------------------------------------------------------------------------------------------------------------------------------------
------------------------------------------------------------------------------------------------------------------------------------------------
------------------------------------------------------------------------------------------------------------------------------------------------
------------------------------------------------------------------------------------------------------------------------------------------------
------------------------------------------------------------------------------------------------------------------------------------------------
------------------------------------------------------------------------------------------------------------------------------------------------
------------------------------------------------------------------------------------------------------------------------------------------------
------------------------------------------------------------------------------------------------------------------------------------------------
------------------------------------------------------------------------------------------------------------------------------------------------
------------------------------------------------------------------------------------------------------------------------------------------------
------------------------------------------------------------------------------------------------------------------------------------------------
------------------------------------------------------------------------------------------------------------------------------------------------
------------------------------------------------------------------------------------------------------------------------------------------------
------------------------------------------------------------------------------------------------------------------------------------------------
------------------------------------------------------------------------------------------------------------------------------------------------
------------------------------------------------------------------------------------------------------------------------------------------------
------------------------------------------------------------------------------------------------------------------------------------------------
------------------------------------------------------------------------------------------------------------------------------------------------

75
Example 5.5 -Zone Allowance Calculation

The vessel M. V. Nara's next voyage is to load full at Coquimbo, for discharge at Longview, Washington.
Basis of loading calculation:

AA) Load Port Coquimbo, Chile DD) FW/BLST/CONSTANT ROB


Load Line Zone Tropical Departure Coquimbo
Max Draft at Berth 10.0 m FW 150 MT
Density 1.025 BLST NIL
CONSTANT 180 MT

BB) Disport Longview Oregon


Load Line Zone Summer EE) Consumption -Laden Voyage
Max Draft at Berth 40 FT IFO = 25 MT/day
Density 1.000 MOO = 1.0 MT/day
FW Production = 6 MT/day
FW Consumption = 12 MT/day

CC) Bunker ROB Departure Coquimbo


IFO 735 MT
DO 35 MT FF) Distance / Speed = 14 knots
LO 15 MT Coquimbo -Longview = 5604 nm.
Coquimbo to 13 N = 3254 nm.

Find:
1. The displacement at departure loading port.
2. The maximum amount of cargo to load.
3. The maximum permissible sailing draft.
3
4. Assuming that the density at load port is 1.009 t/m , find the maximum permissible draft.
5. Assuming that she is about to complete loading and floats at a draft of 9.34 m on dock water density
3
of 1.009 t/m , find how much more cargo she can load.

Solution:

1. Identify the different zones of the voyage

2. Identify the governing zone

3. Calculate the total non-cargo weights to be deducted from the deadweight.

4. Calculate the zone consumption allowance to be added to the limiting zone.

Cons
ROB Steaming Steaming ROB
Weight per Cons. ROB@ Cons.
Departure Time Time Arr
day
HFO
MDO
LO
FW
BLST
CONST
TOTAL

76
EXERCISES 5.1 -Loading involving Load Lines

PROBLEM 5.1.1

Complete the table in page 75, Figure 5.6 -Table of Timber Load line Particulars of MV NARA -using the MV
NARA's hydrostatic Table.

PROBLEM 5.1.2

MV NARA is to load to her summer loadline. Her draft is limited only by the Summer Loadline throughout her
voyage.

1. Find the cargo to load if weights already on board on departure are estimated as follows;

Fuel Oil = 863 MT Displacement =


Diesel Oil = 29 MT Light displacement =
LO = 14 MT
Fresh Water = 310 MT Deadweight =
Ballast = 0 MT Weights on board =
Constant = 220 MT Max. cargo =
Weights on board = 1436 MT Ans: Loadable cargo =
3
2. Find the draft at completion of loading if she floats in density of 1.015 t/m .

3. Find remaining cargo to be loaded if she is now near completion and floating at her summer draft of
9.55 m in dock water density 1.015 t/m3.

Solution:

77
PROBLEM 5.1.3

MV NARA is to load in Tropical Zone and expected to enter Summer Zone eight days after departure from
loading port. Her consumption per day on a laden voyage are; FO= 27 MT, DO = 1.0 MT, FW = 6 MT.

1. Find the cargo to load if weights already on board at departure are estimated as follows;
Fuel Oil = 1263 MT
Diesel Oil = 36 MT
LO = 24 MT
Fresh Water = 259 MT
Ballast = 0 MT
Constant = 180 MT
Weights on board = 1762 MT
3
2. Find the draft at completion of loading if she floats in density of 1.000 t/m .

3. Find remaining cargo to be loaded if she is now near completion and floating at her tropical draft in dock
3
water density 1.000 t/m .

Solution:

1. Identify the limiting zone:

2. Calculate the total non-cargo weights to be deducted from the deadweight.

ROB Departure Cons per


Weight Steaming Time Cons. ROB@
Tropical Zone day
HFO 1263
MDO 36
LO 24
FW 259
BLST 0
CONST 180
TOTAL 1762

3. Apply the zone consumption allowance in the calculation of the deadweight available for cargo.

Displ (governing zone) =


Zone Allowance =
Dep. Displacement =
Light displacement =
Deadweight =
Dep. weights on board =
Max. Cargo =

Ans. Loadable cargo =

NOTE:
______________________________________________________________________________________
______________________________________________________________________________________
________________________________________________________________________________

78
PROBLEM 5.1.4

The vessel MV Max Panama is to load at Winter Zone. She is expected to enter Summer Zone 5 days after
departure from loading port. Her estimated consumption in passage between zones is 205 MT. The non-
cargo weights on board estimated at departure loading port are listed below.

1. Find how much cargo she can load.


Fuel Oil = 1,290 MT
PDiesel Oil = 95 MT
LO = 39 MT
Fresh Water = 268 MT
Ballast = 0 MT
Constant = 230 MT
Weights on board =
3
2. Find the target amidship draft at completion of loading for OW density of 1.010 ton/m
3
3. Assuming she is now floating in her winter draft at OW density 1.010 ton/m , find the quantity of
remaining cargo to load.

NOTE:
____________________________________________________________________________________
____________________________________________________________________________________
____________________________________________________________________________________
____________________________________________________________________________________
____________________________________________________________________________________
____________________________________________________________________________________
____________________________________________________________________________________
____________________________________________________________________________________
____________________________________________________________________________________
____________________________________________________________________________________
____________________________________________________________________________________
____________________________________________________________________________________
____________________________________________________________________________________
____________________________________________________________________________________
____________________________________________________________________________________
____________________________________________________________________________________
____________________________________________________________________________________
____________________________________________________________________________________
____________________________________________________________________________________
____________________________________________________________________________________
____________________________________________________________________________________
____________________________________________________________________________________
____________________________________________________________________________________
____________________________________________________________________________________
____________________________________________________________________________________
____________________________________________________________________________________
____________________________________________________________________________________
____________________________________________________________________________________
____________________________________________________________________________________
____________________________________________________________________________________
____________________________________________________________________________________
____________________________________________________________________________________
____________________________________________________________________________________
____________________________________________________________________________________

79
DRAFT LIMITS IMPOSED BY DEPTHS AND DENSITIES OF WATER IN PORTS AND CANALS

In ports or canals that have draft limit, ships must not exceed such draft limits to operate safely. When the
ship is affected by the draft limit, the quantity of cargo that can be lifted is affected. The maximum draft and
displacement can be achieved if the ship is even keel.

The draft limit can be either;

1. Draft limit imposed in load port or;


2. Draft limit imposed in destination port.

To calculate the amount of cargo, the draft limit is first used to find the displacement of the ship considering
the change of draft due to density. After finding the correct displacement in order to float without exceeding
the draft limit, you can then calculate the quantity of cargo that can be lifted.

DRAFT LIMIT IMPOSED AT LOADING PORT

When draft limit is imposed at the loading port, then the ship must not exceed such draft limit at departure or
until clear of the area where the draft limit is imposed.

Example 5.6

The ship MV Max Panama is scheduled to load full in a port with allowable maximum draft of 11.50 m at
3
density 1.010 t/m . Her estimated ROB's at departure are as follows:

IFO = 986 MT FW = 150 MT


MDO = 67 MT BLST = nil
LO = 35 MT CONST = 285 MT

Find:
1. The ship's displacement to load maximum amount of cargo with out exceeding the draft limit,
2. The amount of cargo that she can lift,
3. The SW departure draft,

Solution:

1. Sum-up the weights already on board at departure.


Fuel Oil =
Diesel Oil =
La =
Fresh Water =
Ballast =
Constant =
Weights on board =

2. From the hydrostatic table take the tabulated


displacement of the limiting draft. (This is the Tabulated Displ = _________________
displacement of the ship if she is to float at that draft in
SW.

80
3. Correct the tabulated displacement Dep. displ = x to
that displacement the ship must assume to float not Tabulated Displ = ________x__________
exceeding the draft limit. 1.025

Dep .displ = Tab. displ x new density


1.025 =

This must be the ship's displacement at


departure in order to float at draft of 11.50 m
in water density 1.010 t/m3.

Dep. Displacement =
Light displacement =
4. Calculate the maximum amount of cargo that Deadweight =
can be Ioaded. Wts. on board @ dep. =
Max. cargo to load =
5. Using the departure displacement as reference,
take the draft from the hydrostatic table. Draft @ SW = ___________

(NOTES)

81
DRAFT LIMIT IMPOSED AT DESTINATION

When your ship is fixed to load with a draft limit at the destination or discharging port, then you must plan to
arrive in that port not exceeding the draft limit and on even keel. To be able to do so, you must take into
account your fuel, diesel, fresh water and other consumption during the voyage. The arrival displacement
can be found based on the draft limit on arrival. Adding the consumption to the arrival displacement, you get
the displacement of the ship on departure from the load port.

How much cargo the ship can load can be calculated using;
1. the displacement at arrival or,
2. if the departure displacement is already knownusing the departure displacement.

Likewise the distribution of cargo can be calculated two ways;


1. based on arrival condition -that is, draft not exceeding the limit and at even keel or,
2. based on departure condition -that is, the required draft and trim on departure to arrive even
keel and not exceed the draft limit on arrival, taking into account the weights consumed and
produced during the voyage.

Example 5.7

MV Max Panama is scheduled to load bauxite in WEIPA, Australia for discharge 1 SB Japan. Maximum
allowable draft at discharge port is 11.00 m at BW density 1012.

Estimated ROB's at departure from WEIPA are as follows:

IFO = 1435 MT FW = 205 MT


MDO = 96 MT BLST = 60 MT
LO = 37 MT CONSTANT = 240 MT

Distance from Weipa to Japan is 3279 nm. Charter speed = 14 knots.


Consumptions per day on laden voyage are as follows:

IFO = 39.0 MT
MDO = 2.0 MT
FW = 6.0 MT

Find;
1. the ships displacement to load maximum amount of cargo without exceeding the draft limit in Japan,
2. the maximum amount of cargo that she can load at WEIPA.
3
3. her departure draft from loading port in SW, and in dock water density 1.010 m /ton.

Solution to no. 1

1. Estimate the voyage consumptions.

ROBs at Departure Steaming ROBs at


Cons. Per day Voyage Cons.
ITEMS Quantity Time Arrival
IFO
MDO
LO
FW
BALLAST
CONSTANT
TOTAL

82
2. From the hydrostatic table take the displacement at the
limiting draft on arrival. (This is the displacement if the
ship is floating at that draft in SW.)

Limiting Draft =
Tab Displ. =

Displ. =

3. Find the displacement that the ship must assume to float


not exceeding the draft limit.

Displacement = Tab. Displ x density


1.025
This must be the ship's displacement when she
arrives in Japan.

4. Load back the consumption and determine Displ Arr. (Disport) =


the displacement at departure Weipa. CaIcuate the + Consumption lowance =
quantity of cargo that can be lifted. Disp Dep (Loadport) =
Light displacement =
Deadweight =
Weights on board =
Max. cargo =

5. Using departure displacement as reference, take the


dep. Draft directly from hydrostatic table; this is the SW
departure draft.

6. From the departure displacement, calculate "Look up" displ = x 1.025


the "look-up" displacement at water density 1.010
1.010 ton/ m3.
Look up displ =

Draft @ BW =
"Look up" displacement = Displ at dep. x 1.025
dd
From hydrostatic tables this corresponds
to the draft _____________________m
and this will be the draft at density
3
_________________ton/m .

83
EXERCISES 5.2 - Draft Limits due to Depths of Water in Ports

Problem 5.2.1
3
M.V. Nara is scheduled to load at Bombay. The port's maximum draft is 9.15 m at density of 1.016 t/m . Find
maximum quantity of cargo to load to that draft if she has estimated weights already on board at departure
as follows.

1. The ship's displacement to load maximum amount of cargo without exceeding the draft limit.
2. Find her SW departure draft.
3. Find the amount of cargo that she can load if her ROB at departure are as follows:

IFO = 1270 MT FW = 275 MT


MOO = 28 MT BLST = 60 MT
LO = 15 MT CONST = 190 MT

Problem 5.2.2

M.V Nara is to load fertilizer in Seattle, Washington for Isabel, Leyte. The maximum SW draft at discharge
port of Isabel is 9.0 m

Estimated ROB's at departure Seattle are as follows:

IFO = 1031 MT Fore FW Tank = 374 MT


MDO = 37 MT FW Tank PIS = 81 MT
LO = 18 MT CONSTANT = 185 MT

Distance from Seattle to Isabel is 5879 nm; Charter speed = 14 knots. Consumptions per day on laden
voyage are as follows:

IFO = 28.0 MT
MOO = 1.0 MT
FW = 6.0 MT

Find:
1. The ship's displacement to load maximum amount of cargo with out exceeding the draft limit at Isabel.
2. Find the maximum amount of cargo that she can load.
3
3. Find her departure draft in SW, in dock water if density is 1.012 m /ton.

Solution:

ROBs at Departure Steaming ROBs at


Cons. Per day Voyage Cons.
ITEMS Quantity Time Arrival
IFO
MDO
LO
FW
BALLAST
CONSTANT
TOTAL

84
Problem 5.2.3

The ship MV Max Panama is fixed to load in a port with a maximum draft of 13.00 m. The dock water density
3
of the loading port is 1.015 t/m ,

Find;

1. What should be the displacement of MV Max Panama to load maximum amount of cargo without
exceeding the draft limit.

2. Find the quanty of cargo to load if the estimated ROB's at departure load port are as follows;

FO = 1136 MT
DO = 65 MT
LO = 40 MT
FW = 250 MT
BALLAST = nil
CONSTANT = 265 MT

Problem 5.2.4

M. V Max Panama is fixing to load in Qinghuandao for Constanta. The maximum draft at discharge port of
3
Constanta is 12.5 m in dockwater density of 1.016 ton/ m

Estimated ROB's at departure are as follows:


IFO = 1351 MT Dri. W Tank = 133 MT
MDO = 79 MT FW Tank PIS = 233 MT
LO = 28 MT CONSTANT = 212 MT

Distance from Qinghuandao to Constanta is 8793 nm. Charter speed = 14 knots. Consumptions per day on
laden voyage are as follows:
IFO = 28.0 MT
MDO = 1.0 MT
FW = 6.0 MT

Find:

1. The ship's displacement and maximum amount of cargo she can load with out exceeding the draft limit
at Constanta.
3
2. Find her departure draft in SW, in dock water if density is 1.020 ton/m .

Solution:

ROBs at Departure Steaming ROBs at


Cons. Per day Voyage Cons.
ITEMS Quantity Time Arrival
IFO
MDO
LO
FW
BALLAST
CONSTANT
TOTAL

85
Problem 5.2.5

MV Nara arrived in port with SW forward draft of 9.07m and aft draft, 9.43m. The maximum allowable draft at
3
berth is 9.00 m. The density of water at berth is 1.011 t/m . She is to lighter at anchorage where the density
is SW.

Find:

1. The quantity of cargo to be discharged.


2. Find the departure draft at anchorage.
--------------------------------------------------------------------------------------------------------------------------------------------
--------------------------------------------------------------------------------------------------------------------------------------------
--------------------------------------------------------------------------------------------------------------------------------------------
--------------------------------------------------------------------------------------------------------------------------------------------
--------------------------------------------------------------------------------------------------------------------------------------------
--------------------------------------------------------------------------------------------------------------------------------------------
--------------------------------------------------------------------------------------------------------------------------------------------
--------------------------------------------------------------------------------------------------------------------------------------------
--------------------------------------------------------------------------------------------------------------------------------------------
--------------------------------------------------------------------------------------------------------------------------------------------
--------------------------------------------------------------------------------------------------------------------------------------------
--------------------------------------------------------------------------------------------------------------------------------------------
--------------------------------------------------------------------------------------------------------------------------------------------
--------------------------------------------------------------------------------------------------------------------------------------------
--------------------------------------------------------------------------------------------------------------------------------------------
--------------------------------------------------------------------------------------------------------------------------------------------
--------------------------------------------------------------------------------------------------------------------------------------------
--------------------------------------------------------------------------------------------------------------------------------------------
--------------------------------------------------------------------------------------------------------------------------------------------
--------------------------------------------------------------------------------------------------------------------------------------------
--------------------------------------------------------------------------------------------------------------------------------------------
--------------------------------------------------------------------------------------------------------------------------------------------
--------------------------------------------------------------------------------------------------------------------------------------------
--------------------------------------------------------------------------------------------------------------------------------------------
--------------------------------------------------------------------------------------------------------------------------------------------
--------------------------------------------------------------------------------------------------------------------------------------------
--------------------------------------------------------------------------------------------------------------------------------------------
--------------------------------------------------------------------------------------------------------------------------------------------
--------------------------------------------------------------------------------------------------------------------------------------------
--------------------------------------------------------------------------------------------------------------------------------------------
--------------------------------------------------------------------------------------------------------------------------------------------
--------------------------------------------------------------------------------------------------------------------------------------------
--------------------------------------------------------------------------------------------------------------------------------------------
--------------------------------------------------------------------------------------------------------------------------------------------
--------------------------------------------------------------------------------------------------------------------------------------------
--------------------------------------------------------------------------------------------------------------------------------------------
--------------------------------------------------------------------------------------------------------------------------------------------
--------------------------------------------------------------------------------------------------------------------------------------------
--------------------------------------------------------------------------------------------------------------------------------------------
--------------------------------------------------------------------------------------------------------------------------------------------
--------------------------------------------------------------------------------------------------------------------------------------------
--------------------------------------------------------------------------------------------------------------------------------------------
--------------------------------------------------------------------------------------------------------------------------------------------
--------------------------------------------------------------------------------------------------------------------------------------------
--------------------------------------------------------------------------------------------------------------------------------------------
--------------------------------------------------------------------------------------------------------------------------------------------

86
WHEN AMOUNT OF CARGO IS LIMITED BY VOLUME

The maximum amount of cargo the ship can carry may also depend on the amount of cargo space and the
stowage factor of the cargo.

A high-density cargo such as ore will bring the ship down to her marks before the cargo spaces are filled. In
this case the maximum cargo will depend upon the maximum acceptable draft and upon the permissible
load for each hold.

With low-density cargo, the cargo spaces will be filled before the ship is down to her marks. Even if there is
more available deadweight, there are no more spaces to fill. The maximum amount of cargo that can be
lifted then depends upon the volume of cargo spaces.

Example 5.8
3
MV Nara is to load full cargo of barley with SF 1.40 m /mton. The weights already on board are distributed
as follows:

FOT 1 (P&S) = 475 mtons DOT (P) = 36 mtons Fore FW Tk = 370 mtons
FOT 2 (P&S) = 512 mtons DOT (S) = 18 mtons Aft FW (P) = 35 mtons
Constant = 280 mtons Aft FW (S) = 45 mtons

Determine how many tons of barley can be loaded if she fills all her holds with ends untrimmed. Find the
draft at completion of loading.

Solution: weight = volume/SF

1. Determine the cargo for each hold:

Hold 1 = capacity = 5503 = 3930mt


SF 1.4
Hold 2 = capacity = 8589 = 6135 mt
SF 1.4
Hold 3 = capacity = 8589 = 6135 mt
SF 1.4
Hold 4 = capacity = 8142 = 5815 mt
SF 1.4
____________________________________________________
Total Cargo = 22015 mt

Cargo = 22015
Weights on board = 1771 After you establish the displacement, it
is then possible to determine the draft
Deadweight = 23786
Light Ship = 6772 of the ship.
Displacement = 30558 mt

87
Volume of Cargo Spaces

The volume of cargo spaces is provided on ships in form of


capacity plans and given in cubic meters or cubic feet.
There are two kinds of figures that express the capacity of
the cargo spaces. These are:

1. Grain Capacity
a. Trimmed ends
b. Untrimmed Ends

2. Bale capacity

1. Grain (loose bulk) Capacity - is the size of the


cargo space or holds that includes spaces between frames,
and used in calculating capacity for loose bulk cargoes
such as grain, coal, ores, etc. In stowing grain, spaces may
remain at the ends of the compartments if the grain is not
trimmed in those places. A capacity figure, less these
spaces, is usually provided in the ship's capacity plans as
"capacity of compartment with ends untrimmed".

2. Bale (packed cargo) Capacity is the space or


volume of the compartment less the spaces in between
frames or spar Figure 5 10 ceiling. The entire available
spaces are not used and there fore the bale capacity is a
smaller volume measurement than grain capacity. The
figure is normally used in calculating box, sack or bale
cargoes etc.

Stowage Factor (SF)

To be able to calculate how many tons can be loaded to a


specific cargo hold, the stowage factor of the cargo should
be known. Stowage Factor is the volume that the cargo will
occupy when stowed, expressed in cubic meters per metric
ton or cubic feet per long ton.

The value of stowage factor should include proper


allowance for broken stowage based on the proper stow of
the cargo. The stowage factor however carefully
determined is always approximate. Since the ratio of
broken stowage varies, influenced by the shape of the
ship's holds, and the condition of cargo Spaces not counted
upon loading as capacity.

The actual size of the cargo is not a stowage factor. Stowage factor is the space occupied when the actual
cargo is stowed. It is the size of the cargo plus spaces that may be lost (broken space) after the cargo is
stowed. Stowage factor figures can be obtained from the shippers via their agents or by the operators, or
obtained from ship's records of previous cargoes carried.

88
The use of Stowage Factor in Cargo Calculations

Stowage factors may be expressed in units of:


3 3
m /mt or in units of - ft /L T

SF = volume / weight or SF = volume / weight


3
volume =m = ft
weight = mt = LT

Using transposition the volume as well as the weight can be found.

Weight = volume / SF or Weight = volume / SF


3 3
volume =m = ft
weight = mt = LT

Example 5.9 -Converting weight to volume using stowage factor

MV shall load 20,000 MT bulk cargo with SF of 1.19 m3/mt. Determine if she can take the cargo using all her
holds.

Volume = weight x SF
= 20000 x 1.19
3
cargo volume = 23800 m

Judgment: Yes
3 3
The total grain capacity of MV NARA is 31958.8 m , while the cargo will occupy only 23,800 m .

Example 5.10 - Converting volume to weight using stowage factor

Weight : volume / SF

How many tons of maize cargo with SF of 1.32 m3/mt would fit into hold #1 if the hold grain capacity is 5712
3
m.

weight = volume / SF
weight = 5712 / 1.32 = 4327 MT
= 4327 MT

Example 5.11
3
MV Nara shall load ore with a SF of 0.35 m /mt in hold #1. How many tons can she take in?

weight = volume
SF
Weight that can be loaded in hold # 1:

weight = 5712 m3 = 16320 mton


0.35 m3/mton
Notice that many more tons of ore than maize will fit into hold #1. Nonetheless, never load a cargo hold full
with ore.

89
Example 5.12
3
A ship has a total grain capacity of 31,958.8 m3 shall load a bulk cargo with SF of 1.219 m /mton. Based on
the cubic capacity, how many tons can the ship take of this cargo?

weight = volume / SF
weight = 31,958.8/ 1.219
weight = 26, 217 mtons Ans. The ship can take 26,217 mtons

Summary -Deciding the amount of cargo the ship can carry

When the ship receives an order to proceed for a certain port and to load a kind of cargo, the steps to be
followed are summarized as follows:

First - Determine the limiting deadweight and draft for the voyage.

Second - Make calculations for non-cargo weights on board: bunkers, diesel, lube oil, water, ballast,
constant, and others.

Third - Make cargo calculations.

Fourth - Make the cargo loading plan so that cargo is well distributed with minimal stress condition,
but optimum sailing trim and acceptable stability characteristics.

90
DISTRIBUTION OF CARGO BETWEEN HOLDS

After calculating the total quantity of cargo to be loaded, the next appropriate step is to decide the quantity of
cargo that must be loaded to each hold. Several factors must be considered, depending on the
circumstances, in making the decision. It must be ensured at all times that the stowage is safe and does not
endanger the ship.

To start working on the sharing of cargo, a tentative loading arrangement may be tried to check that the
requirements of the voyage are met. Such tentative plan may then be revised if any unacceptable results
arise, or modified to best suit the requirements of the voyage.

The tentative sharing of cargo can be based on:


1. standard conditions from ship's stability and loading manuals
2. previous voyages
3. sharing by proportion

Stowage should aim at distributing the cargo for any particular port equally or as nearly equal, as
circumstances permit, in every hold. All the cargo handling equipments are thus fully employed throughout
the time of loading and discharging the ship.

Sharing by proportion

Also referred to as the percentage method of distributing cargo is dividing the cargo according to the
proportion of the hold against the total hold volume to be used. A hold, which is 25% of the total hold
volume, can be allocated 25% of the total cargo. This way the holds are equally filled. (The quantities of
cargo for each hold are not the same except on holds that are of the same size.)

Share of Hold -is the quantity of cargo the hold will get from the total cargo load. The amount is based on
the capacity of the hold compared to the total capacity of the ship.

The share of hold is expressed in term of percentage and calculated as:

Share of Hold (%) = (Hold capacity / Total hold capacity) x 100

The share of the hold in terms of weight can then be calculated as:

Share of Hold (MT) = Total cargo x Share of hold (%)

The above formula can be combined to give:

Share of Hold = total cargo x (Hold capacity/Total hold capacity)

Example 5.13
3
MV Max Panama is to load 66588 MT of potassium sulphate whose SF is 0.9 m /MT. Determine how much
cargo goes into each hold if the cargo is divided proportionally.

Share of hold % = (hold capacity/ total hold capacity)


Share % of Hold 1 = (9081.11/ 82025.08) x 100 = 11.07 %
Share % of Hold 2 = (12891.56/82025.08) x 100 = 15.72 %
Share % of Hold 3 = (12831.48/82025.08) x 100 = 15.64 % Set aside for future use.
Share % of Hold 4 = (11671.47/82025.08) x 100 = 14.23 %
Share % of Hold 5 = (12927.24/82025.08) x 100 = 15.76 %
Share % of Hold 6 = (12881.13/82025.08) x 100 = 15.70 %
Share % of Hold 7 = (9741.08/ 82025.08 ) x 100 = 11.88 %

91
Cargo for hold 1 = 66588 x 0.1107 = 7370 mt
Cargo for hold 2 = 66588 x 0.1572 = 10460 mt
Cargo for hold 3 = 66588 x 0.1564 = 10420 mt
Cargo for hold 4 = 66588 x 0.1423 = 9470 mt
Cargo for hold 5 = 66588 x 0.1576 = 10500 mt
Cargo for hold 6 = 66588 x 0.1570 = 10450 mt
Cargo for hold 7 = 66588 x 0.1188 = 7918 mt

Answer:
* CARGO HOLD (GRAIN)

NAME CAP A C I T Y Share Cargo Rounded


3
M Percentage
No.1 CARGO HOLD 9081.11 11.07 7370 7380
No.2 CARGO HOLD 12891.56 15.72 10460 10450
No.3 CARGO HOLD 12831.48 15.64 10420 10450
No.4 CARGO HOLD 11671.47 14.23 9470 9500
No.5 CARGO HOLD 12927.24 15.76 10500 10458
No.6 CARGO HOLD 12881.13 15.70 10450 10450
No.7 CARGO HOLD 9741.08 11.88 7915 7900
TOTAL 82025.08 100.00 66588 66588

In practice, distribution (as shown in the final column) is usually rounded off to simplify calculations which
must follow.

The resulting distribution should not be readily treated as final. This only serves as a convenient starting
point in your calculation. It must be tested that the requirements of the voyage are met. If any unacceptable
result arises then it must be modified to suit well the requirements of the voyage.

Example 5.14
3
MV Nara is to load 22,300 MT of gypsum stone whose SF is 1.06 m /MT. Determine how much cargo goes
into each hold if the cargo is divided proportionally.

Answer:
* CARGO HOLD (GRAIN)

NAME Total Share of hold Cargo Rounded


Cargo
No.1 CARGO HOLD = 22300 x (5712.1 / 31958.8) = 3,992 3,990
No.2 CARGO HOLD = 22300 x (8874.6/ 31958.8) = 6,199 6,200
No.3 CARGO HOLD = 22300 x (8874.7/ 31958.8) = 6,199 6,200
No.4 CARGO HOLD = 22300 x (8497.4 / 31958.8) = 5,932 5,910.
TOTAL 22,300 22,300

Fill Percentage - indicates how much space is occupied by cargo in a hold. It is the ratio between the volume
occupied by the cargo and the hold capacity.

Fill% = (vol. of cargo / hold cap) x 100

Fill% = (weight x SF) / hold cap) x 100

Fill% = (weight/cargo density/hold cap) x 100

92
Example 5.15
3
Calculate the fill percentage of the cargo hold filed with 7380 MT cargo with SF 0.9 m /MT

Sol.

Fill% = 7380 x 0.9 / 9081 x 100 = 73 %

Example 5.16

Calculate the fill percentage of the cargo hold loaded as follows.

ITEM Capacity Weight SF Volume Fill%


Hold 1 9081.11
Hold 2 12891.56
Hold 3 12831.48
Hold 4 11671.47
Hold 5 12927.24
Hold 6 12881.13
Hold 7 9741.08
TOTALCARGO

Example 5.17
3 3
How much cargo goes into the hold with capacity 9081 m if filled up to 90 %? Cargo SF is 1.13 m /mt.

Solution:

Cargo weight = 9081 x .9/ 1.13 = 7232 MT


____________________________________________________________________________________
______________________________________________________________________________________
______________________________________________________________________________________
______________________________________________________________________________________
______________________________________________________________________________________
______________________________________________________________________________________
______________________________________________________________________________________
______________________________________________________________________________________
______________________________________________________________________________________
______________________________________________________________________________________
______________________________________________________________________________________
______________________________________________________________________________________
______________________________________________________________________________________
______________________________________________________________________________________
______________________________________________________________________________________
______________________________________________________________________________________
______________________________________________________________________________________
______________________________________________________________________________________
______________________________________________________________________________________
______________________________________________________________________________________
______________________________________________________________________________________
______________________________________________________________________________________
______________________________________________________________________________________
______________________________________________________________________________________

93
94
PART II

TRIM CALCULATIONS

CHAPTER
6 Center of Gravity

7 Longitudinal Stability and Trim

95
CHAPTER 6

IN THIS CHAPTER

Center of Gravity (G)

Longitudinal Center of Gravity (LCG)

- The Use of Moment in Finding the Location of G

- Movement of Center of Gravity

- Finding the LCG from TRIM

Vertical Center of Gravity

- Light Ship KG

- Final KG

- The Vertical Movement of G

96
CENTER OF GRAVITY (G)

Figure 6.1 -The downward force of the weight of the ship and the center of gravity. The center of gravity of a
body is defined as the point at which all the mass of the body may be assumed to be concentrated. It is the
point through which the force of gravity is considered to act vertically downwards, with a force equal to the
weight of the body.

Figure 6.2 -Center gravity of the ship

The position of center of gravity is three-dimensional. Its location can be described by three coordinates,
measured longitudinally, vertically and transversely from three reference lines.

1. Its height above the keel (KG).


2. Its longitudinal distance from amidships, or aft and fwd perpendicular (LCG).
3. Its transverse distance from centerline.

NOTE: The location of center of gravity depends upon the distribution of weight within the ship and
location of center of gravity changes every time masses within the ship are moved, added or
discharged.

Longitudinal Center Of Gravity (LCG)

The distance of center of gravity of the ship or the center of gravity of a mass on the ship measured
longitudinally from amidships, aft perpendicular or sometimes fwd perpendicular is called the Longitudinal
Center of Gravity.

97
THE USE OF MOMENTS IN FINDING THE LOCATION OF G

Moment -The tendency of force to produce a rotation or turning effect" about a pivot point.

1. This turning effect is called the moment of force (often abbreviated to "moment"), and the distance is
called the moment arm (or lever arm) of the force.

2. The turning effect of the force is dependant on the distance of the force from the pivot.

3. Units of moments are Newton-meter, ton-meter, foot-ton, etc.

4. Moment is calculated by;

Moment = weight x distance

Moments can be used to find:

1. Shift of center of gravity (GG 1 )


2. Longitudinal center of gravity(Final LCG)
3. Vertical center of gravity (Final KG)

Figure 6.4 -Consider a homogenous iron bar. Its center of gravity will be exactly at the middle.

If we add a piece of Iron bar at the right side of the bar, its center of gravity will be at new mid-length.

G1

The center of gravity has moved from G to G 1 . This distance is called GG 1 , The GG 1 can be found if the
weight and length of the added piece is known. The principle of moment can then be used to determine the
new center of gravity of the bar.

98
Figure 6.5 -Choosing the mid-length [x] as a
reference point and assigning values as follows:

Length of iron bar = 3.2 m


Weight of the bar = 15 kg
Length of added bar = 0.8
Weight of added bar = 3.75 kg

Moment = weight of added bar x distance

The same moment must be equal to;

Moment = Total weight x GG 1

Total weight x GG 1 = weight of added bar x distance

GG 1 = Weight of added bar x distance


Total weight

GG 1 = 3.75 kg x 2.0 m
18.75 kg

GG 1 = 0.4 m

Figure 6.6 -Using the same example as above,


but choosing a different reference point, point
[A] (see figure). GG 1 can also be calculated as
follows:

Moment of bar around point [A] = 15.0 kg x 1.6 m = 24.0 kg. m


Moment for added piece around [A] = 3.75 kg x 3.6 m = 13.5 kg .m
Total weight and moment = 18.75 g = 37.5 kg. m

Distance from [A] of new center of gravity = Total moment / Total weight

Distance from [A] of new center of gravity = 37.5 kg.m / 18.75 kg

Distance from [A] of new center of gravity = 2.0 m

This means that the center of gravity has move from G, which was 1.6m from A to G 1 , 2.0m from [A].

To find the distance between GG 1 ; GG 1 = 1.6 m -2.0 m Ans. GG 1 = 0.4 m

99
Figure 6.7 - We will now consider an example with several weights on one bar.

Length of iron bar = 4.6 m


Weight of the bar = 15 kg
Center of gravity of bar = at mid-length [x]

W1 =2 kg Dist from [x] = 1.6 m


W2 =5 kg Dist from [x] = 0.8 m
W3 =9 kg Dist from [x] = 1.0 m
W4 =3 kg Dist from [x] = 1.9 m

Weight Dist Counter Clockwise


(kg) from [x] clockwise moment
(m) moment (kg m)
(kg m)
Bar 15 x 0.0 = 0.0 0.0
W1 2 x 1.6 = 3.2 -
W2 5 x 0.8 = 4.0 -
W3 9 x 1.0 = - 9.0
W4 3 x 1.9 = - 5.7
Total 34 7.2 14.7

Sum of weights = 34 kg Sum moments = 7.5 kg m clockwise

GG 1 = 7.5 kg m
34 kg
Ans.
GG 1 = 0.22 m

G moved from [x] by 0.22 m to the right

Figure 6.8 -We will now consider the same


example but using point [A] as reference point.

Length of iron bar = 4.6 m


Length of the bar = 15 kg
Center of gravity of bar = 2.3 m from [A]

W1 =2 kg Dist from [A] = 0.7 m


W2 =5 kg Dist from [A] = 1.5 m
W3 =9 kg Dist from [A] = 3.3 m
W4 =3 kg Dist from [A] = 4.2 m

100
Weight Dist Moment around
(kg) from [A] [A] (kg m)
(m)
Bar 15 X 2.3 = 34.5
W1 2 0.7 = 1.4
W2 5 1.5 = 7.5
W3 9 3.3 = 29.7
W4 3 4.2 = 12.6
Total 34 85.7

Location of G 1 from [A] = 85.7 kg. m


34 kg
Location of G 1 from [A] = 2.52 m

Location of G 1 from [A] = Total moment


Total weight

The shift of G (GG 1 ) = 2.52 m -2.3 m

GG 1 = 0.22 m

101
FINDING THE CENTER OF GRAVITY (G) OF THE SHIP

Movement of center gravity

The location of center of gravity depends upon the distribution of weight within the ship. It will remain at its
position provided that no weights within the ship are moved, added or removed.

Every time a weight is moved, added or removed -

1. The G moves directly towards the center of gravity of the added masses.
2. The G moves directly away from the center of gravity of removed masses.

3. The G moves parallel to the path of movement of the masses already onboard.

The shift of the G (GG 1 ) is found by the formula:

GG 1 = weight x distance
Final displacement

To find the final location of center of gravity, we use the principle of moments.

Final LCG = Total longitudinal moment / displacement

Final KG = Total vertical moment / displacement

It must be noted, however, that the initial location of CG must be known before the new or final position of
center of gravity can be calculated.

To calculate the longitudinal moments, the distances (from a common reference) of the weights shifted,
loaded or discharged must be known.

NOTE: The LCG of the ship is necessary for the calculation of the ship's trim. The trim of the vessel
is almost wholly dependent on the position of center of gravity and center of buoyancy.

102
Example 6.1

A ship with a present displacement of 16, 725 tons has center of gravity 2.227m forward of amidship. Her
LBP is 151m. You plan to load the ship as follows:

Cargo Hold 1 = 700 t 49.19 m forward of amidship


Cargo Hold 2 = 800 t 24.30 m forward of amidship
Cargo Hold 3 = 500 t 5.30 m aft of midship
Cargo Hold 4 = 550 t 34.53 m aft of midship

Find the new displacement and the final LCG from amidship.

Hold No. Weight LCG [x] LCG Moment

I. Displ 16,725 t -2.227 m -37,247 t-meters


Hold 1 700 t -49.19 m -34,433 t-meter
Hold 2 800 t -24.30 m -19,440 t-meter
Hold 3 500 t 5.30 m 2,650 t-meter
Hold 4 550 t 34.53 m 18,992 t-meter
New Displ 19,275 t -69;478 t-meter

F. LCG = Total moment


New displ
F. LCG = -69, 478 t-meter
19,275 t

F. LCG = -3.605 m

The center of gravity of the ship is calculated as 3.605 m forward of amidshlp.

The LCG[x] has moved a little forward or GG1 = 1.378 m forward.

Calculating the new LCG when unloading is done the same way. The difference is that the weight is
subtracted from the present displacement and moment.

NOTE: You can find the Longitudinal Center of Gravity (LCG) of all cargo holds, tanks or
compartments in the ship's Capacity Plan. The listed values of LCG is the distance from amidship
(or aft perpendicular) to the geometric center of the holds or compartments.

103
Example 6.2

The ship in Example 6.1 is going to discharge ballast as follows;

No.1 TST P & S 424 t 48.522 m forward of amidship


No.2 TST P & S 696 t 24.300 m forward of amidshlp
No.3 TST P & S 696 t 5.300 m aft of amidship
No.4 TST P & S 659 t 34.610 m aft of amidship

Find new displacement and LCG.

Solution

Compartment Weight LCG [x] LCG Moment


I. Displ. 19,275 t -3.605 m -69,486 t-meters
No.1 TST P & S -424 t -48.522 m 20,573 t-meter
No.2 TST P & S -696 t -24.300 m 16,913 t-meter
No.3 TST P & S -696 t 5.300 m -3,689 t-meter
No.4 TST P & S -659 t 34.610 m -22,808 t-meter
New Displ 16,800 t Total moment -58,497 t-meters

F. LCG = -58,497 t-meter


16,800 t

F. LCG = -3.482 m

When loading and discharging simultaneously, the new LCG can be calculated in the same manner as
above, but weights loaded are treated as positive value and weights discharged are treated as negative
value. Rules of signed numbers must be followed when taking the product and the sum of moments.

Example 6.3

A ship with a present displacement of 29,650 mtons has her LCG at 2.60 m forward of amidship, is to load
bunker and diesel oil as follows;

No.1 FO Tank (P & S) 520 t 24.30 m fwd of amidship


No.2 FO Tank (P & S) 400 t 5.30 m aft of amidshlp
DO Tank (P) 44 t 52.495 m aft of amidship

At the same time she is to discharge her ballast as follows:

Fore Peak Tank 250 t 70.99 m fwd of amidship


Aft Peak Tank 232 t 2.23 m aft of amidship

She is to load fresh water:

Aft FW Tank 36 t 75.56 m aft of amidship


Potable Water Tank 30 t 75.06 m aft of amidship

104
Find the new displacement and LCG.
Solution :
Compartment Weight LCG [x] LCG Moment
I. Displ 29,650 t -2.60 m -77,090 t-meter
Fore Peak Tank -250 t -70.99 m 17,748
No.1 FO Tank (P & S) 520 t -24.30 m -12,636
No.2 FO Tank (P & S) 400 t 5.30 m 2,120
DO Tank (P) 44 t 52.5 m 2,310
Aft Peak Tank -232 t 72.23 m -16,757
Aft FW Tank 36 t 75.56 m 2,720
Potable Water Tank 30 t 75.06 m 2,252
New Displ 30,198 t total moment -79,333 t-meter

F. LCG = -79,333 t-meter


30,198 t

F. LCG = -2.627 m
Ans. New displacement = 30198 t and F. LCG = 2.627 m forward of amidship

Example 6.4

A ship has a displacement of 15800 mt. Some containers with a total weight of 60mtons are moved from aft
deck to the fore deck of the ship -a distance of 83 m. By how much will the ship's LCG move?

Solution
GG 1 = weight x distance
Total weight
GG 1 = 60 mt x 83 m
15800 mt

GG 1 = 0.315 meters going forward

NOTE: G will always moves in the same direction as the shifted weight.

Example 6.5

The ship's displacement is 30884 mt with her LCG calculated at 3.09 m forward of amidship. You wish to
have an LCG 3.277 m forward of amidship. By how much will you move the weight of 285 mt?

Solution:

GG1 = Final LCG -Initial LCG


Final LCG = -3.277 m
Initial LCG = -3.09 m
GG1 = 0.187 m (the G will move forward)
Distance = GG 1 x displ
weight
= 0.187 m x 30884 mt
285 mt
Distance = 20.26 m

Ans: The weight must move 20.26 m forward.

105
EXERCISE 6 - CENTER OF GRAVITY

Problem 6.1

Using the example 7.1 calculate the new LCG using moment about the aft perpendicular.
Solution:

Hold No. Weight LCG [AP] LCG Moment [AP]


Hold 1 700 t 124.69 m 87,283 t-meter
Hold 2 800 t 99.80 m 79,840 t-meter
Hold 3 500 t 70.20 m 35,100 t-meter
Hold 4 550 t 40.97 m 22,534 t-meter
I. Displ 16,725 t 77.727 m 1,299,984 t-meter
New Displ 19,275 t 1, 524,741 t-meter

New LCG = Total moment


The new location of center of gravity for
New displ the loading condition will be 79.105 m
New LCG = 1,524,741 t-meter from aft perpendicular.
19,275 t
New LCG = 79.105 m

Problem 6.2

Using the example 7.2 and 7.3, calculate the new LCG using distances from aft perpendicular and moments
about aft perpendicular.

(Remarks: The example 7.2 with the LCG from AP)

Compartment Weight LCG [AP] LCG Moment [AP]


I. Displ 19, 2 75 t ________________ _____________________
No.1 TST P & S -424 t ________________ _____________________
No.2 TST P & S -696 t ________________ _____________________
No.3 TST P & S -696 t ________________ _____________________
No.4 TST P & S -659 t ________________ _____________________
New Displ 16,800 t Total moment = _____________________

F. LCG = _________________
F. LCG = _________________

Problem 6.3
(Remarks: The example 7.3 with the LCG from AP)

Compartment Weight LCG [AP] LCG Moment [AP]


I. Displ 29,650 t _________________ _____________________
Fore Peak Tank -250 t _________________ _____________________
No.1 F 0 Tan k (P & S) 520 t _________________ _____________________
No.2 FO Tank (P & ) 400 t _________________ _____________________
DO Tank (P) 44 t _________________ _____________________
Aft Peak Tank -232 t _________________ _____________________
Aft FW Tank 36 t _________________ _____________________
Potable Water Tank 30 t
New Displ. 30,198 t Total moment = _____________________

F. LCG =
F. LCG =

106
NOTE: In practice, the lightship condition data of the ship is supplied in the loading manual. The
lightship weight and LCG serves as the initial condition from which the Final LCG for any loading
condition is calculated provided that the dispositions of all the weights already onboard, to be
loaded or discharged, are known.

Finding the LCG from TRIM

In calculating the trim when handling weights, it is essential to have an initial LCG. There are instances
wherein the initial LCG is not readily known. If the positions of weights onboard can no longer be
ascertained with accuracy, the longitudinal moments from which to derive the LCG cannot be calculated.

There is another method to calculate the LCG by deriving it from the ships present trim.

LCG = Trim x MTC x 100 + LCB


Displacement

When the ship's trim is known, the trimming moment can be calculated by;

Trimming moment = Trim x MTC x 100 (i.e. trim is in meters)

or,

Trimming moment = change of trim x MTC x 100(I.e. change of trim in meters).

But, trimming moment is also:

Trimming moment = (LCG -LCB) x displacement

Then,

(LCG - LCB) x displacement = Trim x MTC x 100

LCG - LCB = Trim x MTC x 100


Displacement

Trim x MTC x 100


LCG = displacement + LCB

The LCG as calculated serves as the initial location of G and can be used for any subsequent calculation of
trim. This method must be used when handling weights causing large changes in mean draft.

107
The Vertical Center of Gravity

Finding final vertical center of gravity or VCG the same


principle of moment is employed. But, instead of
calculating moment about [x] or [AP], it is done about the
keel [K]. That is using vertical distances from keel. The
sum G of all vertical moments when divided by the
displacement gives the vertical location of center of
gravity. It describes the vertical distance from keel, and
called KG.

Lightship KG

The mass of the ship when floating with no cargo, fuel,


stores or any other weights not forming part of the hull or
machinery or fixed equipment of the ship is referred to as
the lightships. The lightship serves as an initial condition
from which the displacement and Final KG for any
condition of loading may be calculated.

Inclining experiment is performed to determine the KG of


a ship in light condition. The experiment is carried out by
builders when the ship is as near to completion as
possible.

The KG for lightship is determined by shipbuilders and provided in the ship's stability manual.

Final KG

The final KG is found by using the principle of moments. To start with, the moment about the keel at initial
condition is required. Adding the moments about the keel of all weights loaded and subtracting the moments
about the keel of all weights discharged to the moment at initial condition gives the final moment. This final
moment must be equal to the final displacement multiplied by KG. So, dividing the final moment by
displacement gives the final KG.

Final KG = Total vertical moment / Displacement

To obtain moments about the keel the distances of the weights' center of gravity from the keel must be
known. The values for the center of gravity of all compartments and tanks are found on the ship's capacity
plan. These values are commonly labeled "KG" or "VCG" (vertical center of gravity). The values are the
geometric center of the compartments when filled with homogenous cargo (cargo that has the same
density).

When these compartments are partially filled, it might be necessary to use corrected values of VCG.

108
The vertical movement of G

Figure 6.11 -The G moves directly towards the center of gravity of the added masses.

Figure 6.12 -The G moves directly away from the center of gravity of the removed masses.

Figure 6.13 -The G moves parallel to the path of movement of the masses already onboard.

109
110
CHAPTER 7
IN THIS CHAPTER

Longitudinal Stability

Trim
- Trimming Moment

- Calculating Trim

- When Involving Large Changes in Mean Draft

- Using LCG Method

- Trim and Stability Calculation Sheet

- The Initial Draft as Initial Condition

- When Handling Single / Several Moderate Weights

- Shifting Weights

- Loading or discharging weights

- The Use of Trimming Table

- Finding Required Trim to Arrive Even Keel

- Trimming Pours

- Trim Considerations

- Distribution of Trim between forward and aft drafts

- Change of Trim Due To Change in Density

111
112
LONGITUDINAL STABILITY and TRIM
LONGITUDINAL STABILITY

The ability of the ship to regain its original position when pitching is a measure of her longitudinal stability.
The concepts involved are the same as for transverse stability.

But for longitudinal inclination, the stability depends upon the distance between the center of gravity and the
longitudinal metacenter (or GM L ).

GM L = KM L - KG

In longitudinal stability for normal ship forms, the value of KM L is many times the value of transverse KM,
this is because KM L is equal to KB plus BM L and BM L is found by dividing the second moment of the area of
waterplane by the volume of displacement. Unlike transverse stability, the BM L depends upon the draft and
length, instead of draft and breadth. The BM L so found is commensurate with the length of the ship.

KM L = KB + BM L

Thus, ships are longitudinally stable compared to their transverse stability, that it is virtually impossible for an
undamaged ship to be longitudinally unstable.

In longitudinal stability, the major consideration focuses on trim.

113
TRIM
When the forward and aft drafts are the same, the ship is said to be in even keel. Moving loads already on
board forward or aft or by adding or removing loads at a position forward of or abaft the center of flotation
causes the forward draft and aft draft of the ship to change. The amount of change depends on the weight of
the load and its distance from the center of flotation. The difference between the forward and aft draft of a
ship is called trim.

A small weight moved at a greater distance has the same effect as a large weight moved at a lesser
distance from the center of flotation.

Figure 7.2

Trim = Aft draft -Fwd Draft

The trim of the ship is described as:

Trim by the stern - if the aft draft is greater than the forward draft.
Trim by the head - if the forward draft is greater than the aft draft.

By convention, trim by astern is designated as positive, and trim by the head as negative.

NOTE:

WHAT CAUSES THE TRIM OF THE SHIP TO CHANGE?

1. Moving loads fore and aft from one point to another changes the trim of the ship, but not the
mean draft.

2. Adding or removing loads to and from the ship will involve a decrease or increase of the mean
draft as well as a change in trim.

3. The change in water density where the ship is floating changes the mean draft as well as the trim
of the ship.

To understand how the ship changes trim, consider a ship floating even keel and a weight move
from forward to aft.

114
Ships floating in even keel have her center of gravity and center of buoyancy at the same distance from
amidship and lie in the same vertical line. The location of center of buoyancy depends upon the underwater
volume of the ship, while the center of gravity depends upon the weight distribution.

Depending on the size of the weight and its distance from center of flotation, moving the weight from forward
to aft, changes the position of center of gravity. When the position of the center of gravity and buoyancy
move out of the same vertical line, a moment is created and the ship will trim until the center of gravity and
center of buoyancy are again in the same vertical line.

If the aft draft is greater than the forward draft the trim is by the stern". If the forward draft is greater than the
aft draft the trim is by the head".

Trimming moment
The moment that was created when the center of gravity (G) and center of buoyancy (B) of the ship moved
out of the same vertical line is called the trimming moment. The moment causes the ship to trim about its
center of flotation (known as tipping center). The magnitude of the trimming moment is equal to the
displacement times the horizontal distance between the G at even keel and the final position of G after the
handling of weights. This horizontal distance is called the trimming arm or trimming lever.

115
Figure 7.6 -The final position of G and the position of tabulated B. (The ship is drawn constrained to even
keel.)

Trimming moment = displacement x trimming arm

Trimming moment = displacement x the horizontal distance GG 1 .

At even keel the LCG and LCB are equal, hence the LCG at even keel can be substituted by the value of
LCB. Thus, trimming arm equals;

Trimming arm (GB) = LCG -LCB


To get the trim of the ship, the trimming moment is divided by the ship's MTC at that draft.

Trim [in meters] = Displ x (Final LCG -LCB)


MTC x 100
As stated earlier, when the ship is even keel the distances LCG and LCB are equal. Thus, the value of LCG
when the ship is even keel can be substituted by the value of LCB. The distance between the tabulated LCB
and the final LCG can be considered as the shift of G (GG1). This distance is called trimming arm. The
trimming arm when multiplied by displacement gives the moment that causes the ship to trim.

Trim = Trimming moment


MTC x 100

Trim = Displacement x trimming arm


MTC x 100 The unit of trim in the formula
Trim = Displacement x (LCG LCB) is meter.
MTC x 100

Trim is by the stern if the final LCG is abaft the tabulated position of LCB

116
NOTE: Multiplying the trimming arm with displacement gives the entire trimming moment of the ship,
thus when divided by WTC, the change of trim so calculated is the trim of the ship.

The distance LCG is essential in the calculation of trim. It is best that you are familiar with the methods for
locating the ship's center of gravity before going into the calculation of trim.

The value of LCB at even keel is provided in the hydrostatic tables.

Ship Squat

A ship proceeding through water produces differences in pressure at the ship at side and ship's bottom
causing the ship to drop vertically in the water. The ship will also generally trim forward or aft depending of
the ship's block coefficient.

The overall decrease of underkeel clearance forward or aft, compared to under keel clearance of the ship
when at rest, is called squat.

Squat is more pronounced in shallow water than in deep water. To understand squat merit a separate
discussion.

CALCULATING TRIM

Methods in calculating the trim can be categorized as;

LCG Method -the ship's final LCG and the ship's hydrostatic data is used to calculate the ships trim. The
method uses the principles of moment to find the ship's final LCG.

This method is used when handling several large weights or involving large changes in mean draft.

Change of Trim Method (CoT Method) -the amount the trim will change due to the effects of adding,
removing or shifting longitudinally of weights is calculated and applied to the initial trim to find the ship's final
trim. The method also uses the principles of moment to find the weight's trimming effect.

For every change of draft, the TPC, MCT and LCF changes and that may introduce appreciable error in
calculating draft and trim. Employing a method that may remove this error is a complex calculation, such as
when calculating trim involving large changes in mean draft (LCG Method).

In small changes of mean draft the errors brought about by the TPC, MTC and LCF, are no longer significant
and a much simpler calculation may be employed. Such as in change of trim (CoT) calculation.

TRIM CALCULATION WHEN HANDLING WEIGHTS INVOLVING LARGE CHANGES IN MEAN DRAFT

Using the LCG Method

Trim calculation involving large changes in mean draft, the calculation using LCG method can be divided
into two instances depending on the initial condition.

1. Trim calculation with the lightship as initial condition (I.e. using stability calculation sheet).

This method of calculation Is not dependent on the initial draft. This calculation requires that
the correct location and correct quantity of each weights presently on board is known. The
trim is calculated by;

Trim (in meters) = Displ x (Final LCG LCB)


MTC x 100

117
2. Trim calculation with the present drafts as initial condition.

There are times that the locations and quantity of each weight presently already on board
can not be ascertained, that the final LCG of the ship can not be calculated using moments.
To calculate the trim, the initial condition of the vessel is based on its present draft. The
ship's initial LCG is found by;

LCG = Trim x MTC x 100 + LCB


Tab. Displacement

TRIM AND STABILITY CALCULATION SHEET

Deadweight = Sum of CARGO, FO, DO, LO, BALLAST, FW, CONSTANT


Displacement = Deadweight + Light Ship
Longitudinal Moment = weight x its LCG
Final LCG = Total Longitudinal Moment/Displacement
Trim = Displacement x (Final LCG -LCB) / ( MTC x 100 )
Draft Fwd = Hydrostatic Draft -(LPP/2 + LCF) / LPP) x TRIM)
Draft Aft = Draft Fwd + Trim

The trim and stability calculation sheet is used to conveniently calculate at once the effects of several
weights to the ship's draft and trim. That is the initial condition is represented by the lightship (remember the
homogenous bar). The final distribution of the weights on board, cargo and non-cargo is considered as
added weight on the lightship.

The work is always done from the light condition using the lightship displacement, LCG and KG, which
serves as the initial condition. Then, the weight in each compartment is listed. The total of all these weights
including the lightship must equal the ship's displacement.

All the longitudinal moments are then taken to determine the ships final LCG. Then the trim is calculated by
taking the trimming arm and multiplying it with the displacement and divided by the MTC.

By using the stability calculation sheet, the ship's draft and trim can be calculated at any condition of the ship
provided that all the present weights onboard and its location on the ship are known. The same form is also
used to calculate the ship's fluid GM (GM corrected for Free Surface Effect).
Although the calculation sheet may vary from ship to ship, they contain the same basic Information as listed

1. the column containing the name of holds/compartments, lightship, constant


2. the column for weight
3. the column for LCG
4. the column for longitudinal moment
S. the column for VCG
6. the column for vertical moments
7. the column for free-surface Inertia

NOTE: The first objective in the calculation is to find the displacement and the Final LCG of the ship.
The longitudinal moments are calculated from the reference point amidship. Once the displacement
and final LCG are known the draft and trim can be calculated.

118
Example 7.1

You plan to load the MV Max Panama as shown in the table below.

Compartment / Item Weight Compartment / Item Weight


NO.1 F.O.T. (C) NO.1 CARGO HOLD 7474
NO.2 F.O. T. (P) NO.2 CARGO HOLD 10400
NO.2 F.O.T. (S) NO.3 CARGO HOLD 10400
NO.3 F.O. T. (P) 225 NO.4 CARGO HOLD 9500
NO.3 F.O. T. (S) 230 NO.5 CARGO HOLD 10400
NO.4 F.O. T. (C) 402 NO.6 CARGO HOLD 10400
NO.5 F.O. T. (C) 175 NO.7 CARGO HOLD 8000
FUEL OIL TOTAL 1032 CARGO TOTAL 66574
NO.1 D.O. T (P) 76 CONSTANT 280
NO.1 D.O. T (S) 30
DIESEL OIL TOTAL 106
LUBE OIL STORAGE TK 15
AUX. LUBE OIL 12
LUBE OIL TOTAL 27
D.W.T. (S) 130
F.W.T. (P) 80
F.W.T. (S) 80
FRESH WATER TOTAL 290

1. Use the trim Calculation Sheet and Hydrostatic tables provided to calculate the draft and trim.

MY MAX PANAMA Trim and Stability Calculation Sheet

ITEM WEIGHT M.G. MT KG MT I.SG


% MID. G KG (M)
(T) (T- M) (T-M) (T- M)
NO.1 CARGO HOLD 7474
NO.2 CARGO HOLD 10400
NO.3 CARGO HOLD 10400
NO.4 CARGO HOLD 9500
NO.5 CARGO HOLD 10400
NO.6 CARGO HOLD 10400
NO.7 CARGO HOLD 8000
CARGO TOTAL 66574
NO.1 F.O.T. (C)
NO.2 F.O.T. (P)
NO.2 F.O.T. (S)
NO.3 F.O. T. (P) 225
NO.3 F.O. T. (S) 230
NO.4 F.O.T. (C) 402
NO.5 F.O.T. (C) 175
FUEL OIL TOTAL 1032
NO.1 D.O.T (P) 76
NO.1 D.O. T (S) 30
DIESEL OIL TOTAL 106
M.E. LUBE OIL 15
AUX. LUBE OIL 12

119
LUBE OIL TOTAL 27
D.W.T. (S) 130
F.W.T. (P) 80
F.W.T. (S) 80
FRESH WATER TOTAL 290
F.P.T. P
NO.1 WBT/TST (P&S)
NO.2 WBT/TST (P&S)
NO.3 WBT TST (P&S)
NO.4 WBT/TST (P&S)
NO.5 TST (P&S)
NO.1 WBT (P&S)
NO.2 WBT (P&S)
NO.3 WBT (P&S)
NO.4 WBT (P&S)
NO.5 WBT (P&S)
A.P.T.
NO.4 CARGO HOLD
WATER BALLAST TOTAL
CONSTANT 280
DEADWEIGHT
LIGHT SHIP 9614
DISPLACEMENT

1. Put all the weight of cargo, ballast, IFO, 0, lubes, h water, constant, into the weight column in their
appropriate rows.

2. Put the weight's longitudinal center of gravity (LCG).

3. Sum up all the weights on board; cargo and non cargo to obtain DEADWEIGHT

DEADWEIGHT = SUM (FO, DO, LO, BALLAST, FW / CARGO, CONSTANT)

4. Sum up the DEADWEIGHT and LIGHTSHIP to obtain DISPLACEMENT. This will be the actual
displacement.

DISPLACEMENT = DEADWEIGHT + LIGHTSHIP

5. Multiply the weight by its LCG and put it In the longitudinal moment column.

LONGITUDINAL MOMENT = WEIGHT x LCG


6. Sum up the longitudinal moment to obtain total longitudinal moment.

TOTAL LONGITUDINAL MOMENT = SUM (FO LCG_MOMENT, DO LCG_MOMENT,


LO LCG_MOMENT, BALLAST LCG_MOMENT, FW LCG_MOMENT, CARGO
LCG_MOMENT, CONSANT LCG_MOMENT)

7. Calculate the Final LCG by dividing the total longitudinal moment with the displacement.

FINAL LCG = TOTAL LONGITUDINAL MOMENT / DISPLACEMENT

8. Using the displacement, take the Hydrostatic Draft, LCB, LCF and MTC from the Hydrostatic table.

120
NOTE: The sign convention is reversed in some loading manuals and these
affect the method of calculation described above. So it is necessary to be
absolutely sure which sign convention is adopted in the manual.

Hydrostatic Draft =

LCB =
LCF =
MTC =

9. Solve the draft and trim using the following formula. This would be the draft and trim at SW.

TRIM = DISPLACEMENT * (F. LCG - LCB) / (MTC x 100)

DRAFT FWD = HYD. DRAFT - (((LPP/2 + LCF) / LPP) X TRIM)

DRAFT AFT = DRAFT FWD + TRIM

MEAN DRAFT = (DRAFT FWD + DRAFT AFT)/2 trimming moment

9.1 F.LCG = trimming arm


-LCB =
Trimming Arm =
x Displacement = Trim = Displ x (F.LCG LCB)
Trimming moment = MTC x 100
/ (MTC x 100) =
Trim =

Trim is by the stern if the final LCG is abaft the tabulated position of LCB.

9.2 To obtain the final draft, calculate the change of draft forward and aft and apply to the hydrostatic draft.

Change of draft(cd) fwd = '/2 LPP + ( LCF ) x trim


LPP
Change of draft(cd) fwd =

Change of draft (cd) aft = trim - cd fwd

9.4 Fwd Aft Alternatively: Alternatively:


Hyd. Draft =
Hyd. Draft = - Cd Fwd =
Cd Fwd and Aft = Fwd draft =
Draft SW = + Trim =
Aft draft =

9.5 M. Draft = (Fwd Draft + Aft Draft)/2


M. Draft =

121
Using the same problem as in Example 7.1, find the departure draft when the ship floats in dock water of
3
1.009 ton/m .

1. Use the "'look-up displacement". Divide the actual displacement by dock water density to get the volume
of displacement. Convert the volume of displacement to the "look-up displacement" by multiply it with
density of SW.

Look-up Dlspl = Actual Displ x 1.025


dock water density
Look-up Displ =

Look-up Displ =

The "look up" displacement is for locating the draft (at density other than 1.025) from the hydrostatic
table.

Note: It should be clearly understood that the actual displacement of the ship remains as the total of
weights (including lightship). Her "look up" displacement was found only to facilitate obtaining the
data from the hydrostatic table which tabulates draft and displacement for the ship in SW.

2. Take the corresponding draft, LCB, LCF and MTC from the hydrostatic table using the look- up
displacement.

Corr. Draft =
LCB [x] =
LCF [x] =
MTC =
3. Use the value look-up displacement, and the corresponding draft, LCB, LCF and MTC, to calculate the
draft and trim. This would be the draft and trim in dock water density.

Trim BW = Look-up Displ x (F.LCG -LCB)


MTC x 100
Trlm BW =

4 Cd Fwd =1/2 LPP + LCF x Trim


LPP
Cd Fwd =

5. Cd Aft = Trim -Cd Fwd

Cd Aft =

6. Fwd Aft
D. Draft BW =
Cd Fws & Aft =
Draft BW =
M. Draft = (Fwd Draft + Aft Draft) / 2
M. Draft =

NOTE: Each "Trim and Stability Calculation Sheet" is only good for one condition. Calculating a new
condition must be done on another sheet.

122
Example 7.2 - MV Max Panama displaces 44756 MT and final LCG 6.52 forward of amidship. Using data
from the ship's hydrostatic table. Calculate the ship's trim, final fwd and aft drafts and the mean draft.

Hydrostatic draft = 7.85 m


LCB = -9.45
LCF = -6.98
MTC = 820.4
TPC = 60.6

Trim [in meters] = Displ x (Final LCG -LCB)


MTC x 100
= 44756 x ( -6.52 -( -9.45))
820.4 x 100

Trim [in meters] = 1.598 m or 1.60 m

Trim is by the stern if the final LCG is abaft the tabulated position of LCB.

Change of draft fwd = LPP + (LCF) x trim


LPP

Cd fwd = 107.5 + (-6.98) x 1.60


215

The trimming moment so calculated is the trimming moment of the whole ship. Thus the resulting trim is for
the entire ship. That the change of draft due to trim is applied to the hydrostatic draft (or mean draft) and not
to the initial forward and aft (as in this case there is none)

Change of draft aft = trim - cd fwd

Cd fwd = 1.60 - 0.748

Cd aft = 0.852

Fwd Aft:
Hydrostatic draft: 7.85 m 7.85 m
Cd Fwd and Aft 0.748 0.852
Draft 7.102 m 8,702 m

Mean Draft = 7.90 m

Note that the hydrostatic draft and the mean draft are not equal. The hydrostatic draft is the draft of the ship
at the center of flotation, while the mean draft is average of the forward and aft drafts.

Example 7.3

The displacement of MV Nara is 30558 mt and her Final LCG Is 2.094m forward of amidships. Using data
from the ship's hydrostatic table. Calculate the ship's trim, final fwd and aft drafts and the mean draft.

Solution;

Hydrostatic draft = 9.46 m


LCB = -3.320 m
LCF = 0.436 m

123
MTC = 363.34 m-tons
TPC = 35.40 mtons

Trim [in meters] = Displ x (Final LCG -LCB)


MTC x 100

Trim [in meters] = 30558 x ( -2.094 -(- 3.32))


363.34 x 100

= 1.03 m

Change of draft fwd = LPP + ( LCF ) x trim


LPP

Cd fwd = 75.5 + (0.436 ) x 1.03


151

= 0.518 m

Change of draft aft = trim -cd fwd


Cd aft = 1.03 -0.518
Cd aft = 0.512 m

Fwd Aft
Hydrostatic draft 9.460 m 9.460 m
Cd Fwd and Aft 0.518 0.512
Final Draft 8.942 m 9.972 m

Mean Draft = 7.457 m

NOTE: From the above examples, we were able to calculate the trim because the LCG was already
given. In actual cases the center of gravity of the ship must be first determined prior to the
calculation of trim. The ship's LCG is found by using the principles of moment.

To facilitate the calculation of ship's LCG, trim and initial stability, the "Trim and Stability Calculation
Sheet" is used.

Example 7.4

MV Nara's light displacement is 6772 mtons and LCG 38.63 m aft of amidships. She is to load full cargo of
barley with SF 1.40 m3jmton. The weights already on board are distributed as follows;

FOT 1 (P&S) = 47S mtons DOT (P) = 36 mtons Fore FW Tk = 370 mtons
FOT 2 (P&S) = 512 mtons DOT (S) = 18 mtons Aft FW (P) = 3S mtons
Constant = 280 mtons (LCG= 38.63 m aft [x]) Aft FW (5) = 45 mtons

124
Calculate the draft and trim considering all ballast tanks are empty and all holds are loaded as follows:

MID.G M.G MT KGMT I.SG


ITEM % WEIGHT KG (M)
(T-M) (T-M) (T-M)
NO.1 CARGO HOLD 3930
NO.2 CARGO HOLD 6135
NO.3 CARGO HOLD 6135
NO.4 CARGO HOLD 5815
CARGO TOTAL 22015
F.P.T
Fore WBT
NO.1W.B.T. F (P&S)
NO.1W.B.T.A (P&S)
NO. 2W.B.T. F (P&S)
NO. 3W.B.T. F (P&S)
NO.4W.B.T.P S (P&S)
NO.4 W.B.T. C (P&S)
NO.1 T.S.T. (P&S)
NO.2 T.S.T. (P&S)
NO.3 T.S.T. (P&S)
NO.4T.S.T. (P&S)
A.P. T.
WATER BALLAST TOTAL
Fore FW Tank 370
F.W.T. (P) 35
F.W.T. (S) 45
FRESH WATER TOTAL 450
NO.1 F.O.T. (P & S) 475
NO.2 F.O.T. (P & S) 512
F.O. Sett. T.
F.O. Daily T.
FUEL OIL TOTAL 987
NO. D.O.T (P) 36
NO. D.O.T (S) 18
D.O. Sett. Tank
D.O. Daily Tank
DIESEL OIL TOTAL 54
M.E. LUBE OIL
AUX. LUBE OIL
LUBE OIL TOTAL
CONSTANT 280
DEADWEIGHT
LIGHT WEIGHT 6772
DISPLACEMENT

1. Calculate the Final LCG by dividing the total longitudinal moment with the displacement.

F. LCG = Tot. Long. Mmt / Displ.


F. LCG =

2. Using the displacement, take the Hydrostatic Draft, LCB, LCF and MTC from the hydrostatic table.

Hydrostatic Draft =
LCB =

125
LCF =
MTC =

3. Solve the draft and trim. Trim = Displ x (F.LCG -LCB)


MTC x 100
F. CG =
- LCB = By Inspection, trim is "by the stern" if the final LCG is
Trimming Arm = abaft the tabulated position of LCB.
x Displacement = By calculating using signed numbers, positive trim
Trimming moment = means "trim by the stern", and negative trim means
/ (MTC x 100) = "trim by the head".
Trim =

Cd Fwd = LPP + LCF x Trim


LPP
Cd Fwd =
Cd Fwd =

Cd Aft = Trim -Cd Fwd


Trim =
-Cd Fwd =
Cd Aft =
Alternatively:
Fwd Aft Alternatively:
Hyd. Draft =
Hyd. Draft = - Cd Fwd =
Cd Fwd and Aft = Fwd Draft =
Draft SW = + Trim =
Aft Draft =
M, Draft = (Fwd Draft + Aft Draft)/2
M. Draft =
From the same Example 7.4, calculate the draft: and trim using LCG's from aft perpendicular and the
excerpt of the ship's hydrostatic table.

MV NARA HYDROSTATIC TABLE

DRAFT. DISPL LCF LCB KB TKM MTC LKM TPC

9.45 30531.73 75.073 78.824 4.896 10.623 363.13 184.49 35.39


9.46 30566.95 75.064 78.820 4.901 10.624 363.34 184.39 35.40
9.47 30602.18 75.054 78.815 4.906 10.624 363.56 184.30 35.40

MID.G M.G MT KGMT I.SG


ITEM % WEIGHT KG (M)
(T-M) (T-M) (T-M)
NO.1 CARGO HOLD 3930
NO.2 CARGO HOLD 6135
NO.3 CARGO HOLD 6135
NO.4 CARGO HOLD 5815
CARGO TOTAL
F.P.T
Fore WBT
NO.1W.B.T. F (P&S)
NO.1W.B.T.A (P&S)

126
NO. 2W.B.T. F (P&S)
NO. 3W.B.T. F (P&S)
NO.4W.B.T.P S (P&S)
NO.4 W.B.T. C (P&S)
NO.1 T.S.T. (P&S)
NO.2 T.S.T. (P&S)
NO.3 T.S.T. (P&S)
NO.4T.S.T. (P&S)
A.P. T.
WATER BALLAST TOTAL
Fore FW Tank 370
F.W.T. (P) 35
F.W.T. (S) 45
FRESH WATER TOTAL
NO.1 F.O.T. (P & S) 475
NO.2 F.O.T. (P & S) 512
F.O. Sett. T.
F.O. Daily T.
FUEL OIL TOTAL
NO. D.O.T (P) 36
NO. D.O.T (S) 18
D.O. Sett. Tank
D.O. Daily Tank
DIESEL OIL TOTAL
M.E. LUBE OIL
AUX. LUBE OIL
LUBE OIL TOTAL
CONSTANT 280
DEADWEIGHT
LIGHT WEIGHT 6772
DISPLACEMENT

Finding draft and trim when handling weights Involving large changes in mean draft by way of the
Initial draft.

In calculating the trim when handling weights, it is essential to have an initial LCG. In practice, ships are
supplied with Information regarding the ship's lightweight displacement. This information includes the LCG
(and KG) at lightweight condition. The lightweight displacement, LCG (and KG) serves as the initial condition
from which the final LCG for any loading condition is calculated (provided that the disposition of all the
weights already onboard, to be loaded or discharged are known).

There are instances wherein the calculation of draft and trim do not start from the lightship condition. That Is,
lightweight displacement and LCG is not used in calculation as the Initial condition. These happens when
the LCG of the weights already onboard cannot be ascertained, thus longitudinal moments from which to
derive the final LCG, cannot be calculated. Instead of calculating from lightweight displacement and Its LCG,
calculation is started from the present condition of the ship. Wherein the initial LCG is derived from the
present trim.

LCG = Trim x MTC x 100 + LCB


Tab. Displacement

This formula can be used when handling weights causing large changes in mean draft. The LCG as
calculated serves as the initial location of G and can be used for any subsequent calculation of trim.

127
Use MAST Form no.7 In your calculation.

To complete the form:

1. Take the mean draft and using the LCF, correct the draft to draft at CF (true mean draft).

2. Using true mean draft as argument, enter the hydrostatic table and take the tabulated displacement,
LCB, and MTC.

3. Using the ship's initial trim, calculate the ship's initial LCG.

4. Correct the tabulated displacement to initial displacement of the ship.

5. Determine the ship's final displacement and LCG after loading or discharging.

6. Convert the final displacement to look-up displacement.

7. Using look-up displacement as argument, extract the data from hydrostatic table the value of draft, LCB,
LCF, and MTC.

8. Find the trim using the LCG and LCB and displacement at final condition.

Example 8.10
3
M.V. Nara's present draft forward is 7.60 m and aft is 8.95 m. Dock water density is 1.017 t/m .

She is to load: She is to discharge:


Hold 1 = 3000 MT No.1 WBT (F) = 420 MT
Hold 3 = 3500 MT No.1 WBT (A) = 400 MT
Hold 4 = 2000 MT No.3 WBT = 450 MT
No.1 TST = 400 MT

What is the draft and trim after completion?

128
FINDING THE DRAFT AND TRIM WHEN HANDLING WIEGHTS INVOLVING LARGE CHANGES IN
MEAN DRAFT
1. Find present mean draft and trim:
Mean draft = (Draft fwd + Draft aft)/2
Trim = Draft aft -Draft fwd

2. Take from hydrostatic table corresponding LCF.


3. Correct the mean draft to mean draft at center of Initial =
floatation: displ =
Md CF = (Trim x (LCF) / LPP) + Mean draft Tab.Displ
4. Using Md CF , take from hydrostatic table the x dd
Displacement (Displ), LCB and MTC. /1.025
5. Find ship's LCG (Initial LCG) based on the trim and Density
displacement: Draft fwd =
Draft aft =
Initial LCG = (Trim x MTC x 100/ Tab.Displ ) + LCB 1. Mean draft =

6. Correct the Displ to true displacement (Initial displ) by 2. LCF = LPP =


applying the effect of density: 3. Md CF =
(dd is dock water density)
Tab. Displ =
7. Find the ship's final displ and total LCG moment after LCB = MTC =
loading or discharging;
Initial LCG =
Final Displ = Initial displ + weights loaded - weights
discharged Initial displ =
Total LCG moment = ship moment + moment of
weights loaded -moment of weights discharge.
Weights LCG LCG Moment
8. Find the ship's LCG (Final LCG) after loading or Initial displ
discharging:

Final LCG = Total LCG moment / Final Displ

9. Correct the Final Displ to look-up dlspl due to the


effect of density to find the mean draft;

Look-up Displ = Final Displ x 1.025 / dd

10. Using the look-up displ, take from the hydrostatic


tables the draft, LCB, MTC, LCF, TPC. Final LCG =

11. Find the final trim after loading or discharging: Look-up Disp =
Corresp. Draft =
Final Trim = (LCG -LCB) x Look-up Displ /( MTC x LCF = LCB =
100) TPC = MTC =
12. Find change of draft forward (Cd fwd) and change of Final Trim =
draft aft (Cd aft).
Fwd Aft
Cd fwd = ((LPP/2 + LCF) / LPP) x Final Trim
Draft
Cd Fwd & Aft
Cd Aft = Final Trim -Cd fwd
Final drafts
13. Apply the Cd fwd and Cd aft. to draft to get final drafts
fwd and aft after loading or discharging.

129
TRIM CALCULATION WHEN HANDLING SINGLE/SEVERAL MODERATE WEIGHT

Change of Trim (CoT) Method

Adding weight to, or removing it from, a particular position, or by shifting a weight already on the ship also
produces trimming moment. The magnitude of the moment can be found by:

For weights loaded or discharge,

Trimming moment = weight x its distance forward or aft of center of flotation

and,

For shifted weight already on board,

Trimming moment = weight x the distance move forward or aft

The trimming moment so produced when divided by the MTC will indicate the amount by which the trim will
change.

Change of trim (CoT) = Trimming moment = weight x distance


MTC x 100 MTC x 100

Change of trim is the amount by which the trim changes when loading or discharging or shifting weights
already on board. It is the difference between the trim before handling weights and the trim after handling
weights.

CoT = Final Trim -Initial Trim

If the sign Ct is positive the trim changes by the stern.


If the sign Ct is negative the trim changes by the head.

The following examples illustrate the change of trim.

Before After Chance of Trim Sign


Fwd Draft 9.00 9.20
Aft Draft 10.00 9.80
Trim 1.00 by the stern 0.60 by the stern 0.40 going fwd Negative
Fwd Draft 12.00 12.10
Aft Draft 12.50 13.20
Trim 0.50 by the stern 1.10 by the stern 0.60 going aft Positive
Fwd Draft 9.30 8.90
Aft Draft 9.00 9.40
Trim 0.30 by the head 0.50 by the stern 0.80 going aft Positive

NOTE: The change of trim as calculated deals only with the trimming moment of weights added,
removed or shifted and not of the entire vessel. The change of trim then is not the total trim of the
vessel.

130
Finding draft and trim after shifting single weight

In shifting of weights, the displacement of the ship does not change since no weights are added or
discharged. Only the location of the weights has changed. The mean draft stays the same, but the forward
and aft draft changes.

When the weights are moved longitudinally, the center of gravity of the ship also moves parallel to the path
of the movement of the shifted weight. The center of gravity and buoyancy separates and creates a trimming
moment.

The distance the ship's G traveled is labeled GG1. This time the initial position of G is not equal to the
position of B as in the calculation of ship's trim. But the trimming moment is equal to the GG1 times the
displacement, which is also equal to the weight times the distance it moved. This moment is responsible for
the change of the fwd and aft draft from the initial forward and aft draft and do not present the trimming
moment for the entire ship.

This time, dividing the trimming moment by the MTC will give the amount by which the trim will change (CoT)
and not the whole trim of the ship.

Figure 7.7

Trimming moment = weight x distance it moved.

Change of trim (CoT) = Trimming moment


MTC x 100

Change of trim (CoT) = weight x distance it moved


MTC x 100

The final trim after handling the weight is calculated by applying the change of draft forward and aft to the
initial forward and aft drafts, By how much the forward and aft draft will change is taken from the Cot, taking
into account the location of LCF.

Cd Fwd = LPP + LCF x Cot


LPP

Example 7.7

The ship MV Max Panama is floating at drafts F 11.13 m and A 11.35 m. Find the drafts fore and aft if 370
MT of FO is transferred from No.1 Fuel Oil Tank (C) to No.4 Fuel Oil Tank (C).

131
Solution:

1. Find the mean draft; Mean draft = (fwd draft + aft draft) / 2

Mean draft = (11.13 + 11.35) /2 = 11.24 m I. Trim = 0.22 m

2. Take from the hydrostatic tables, the MTC, LCF[x];

M TC = 929.8 t-m LCF[x] = -1.65 m

3. Find the distance travelled by the shifted weight.

LCG[x] of No.1 FaT C =-35.29 m


LCG[x] of No.4 FaT C = 42.95 m
Distance = 78.24 m

4. Find Change of Trim, Cot = weight x distance it moved


MTC x 100

Change of trim (Cot) = 370 x 78.24


929.8 x 100

Change of trim (Cot) = 0.311 m

5. Find Change of draft fwd, Cd Fwd = LPP + LCF x Ct


LPP

Cd Fwd = 107.5 + (-1.65) x 0.311


215

Cd Fwd = 0.153 m

6. Find Change of draft aft (Cd Aft) = Ct - Cd Fwd

Ct = 0.311 m
Cd Fwd = 0153 m
Cd Aft = 0.158m

7. Apply Cd Fwd and Aft to initial Draft Fwd and Aft.

Fwd Aft
Initial Draft = 11.130 11.350
Cd Fwd and Aft = 0.153 0.158
Final Draft = 10.977 m 11.508 m
F. Trim = 0.531 m

132
M. Draft = Fwd Draft + Aft Draft

M. Draft = 11.242m

Finding amount of weight to shift to obtain desired draft from given initial draft

Weight to shift = CoT x MTC x 100 / distance it moved

Figure 7.9

Example 8.7

While making a loading plan for MV Max Panama, you obtained a draft forward 12.93 m and aft 13.29 m. To
make your plan even keel how much cargo are you going to shift from Hold 7 to Hold 1.

1. Find the mean draft; Mean draft = (fwd draft + aft draft) / 2

Mean draft = (12.93 + 13.29) / 2 = 13.11 m I. Trim = 0.36 m

2. Take from the hydrostatic tables, take the MTC, LCF;

MTC = 980.0 t-m LCF = 0.20 m

3. Find the distance traveled by the shifted weight (this is the distance of G between two hold);

(x)LCG of No.7 HOLD = 66.32 m


(x)LCG of No.1 HOLD = -86.98 m
Distance = 153.3 m
4. Find Change of Trim (Ct);

Final Trim = 0.00


Initial Trim = 0.36
Cot = - 0.36 m

5. Find the weight to shift;

Wt = (Ct x MTC x 100) / distance


Wt = (0.36 x 980.0 x 100) /153.3
Wt = 230.1 mtons

Ans. 230.1 mtons of cargo is shifted from hold 7 to hold 1.

Finding the distance of shifting a weight to obtain desired draft.

133
Example 7.8

MV Nara is floating at drafts fwd =9.47 m and aft = 9.63 m. The weight 220MT is initially 38.63 m aft of
amidships. How far should it be shifted to make the ship even keel.

1. Find the mean draft; Mean draft = (fwd draft + aft draft) / 2

Mean draft = (9.47 + 9.63) / 2 = 9.55 m I. Trim = 0.16 m

2. Take from the hydrostatic tables; take the MTC, LCF;

MTC = 365.28 t-m LCF = 0.522 m

3. Find Change of Trim (Ct);

Final Trim = 0.00 m


Initial Trim = 0.16 m
---------------------------
CoT = -0.16 m

4. Find the distance for shifting; Dist = ( Ct x MTC x 100 ) / Wt

Dist = 0.16 x 365.28 x 100/220


Dist = -26.57 m
Ans. The weight must move 26.57 m forward.

5. The weights new LCG from amidships.

Weight's initial LCG = 38.63 m


Distance = -26.57m
----------------------------
Weight's new LCG = 12.06 m aft of amidships

EXERCISES 7.1 -SHIFTING OF WIEGHTS

7.1.1 MV Max Panama floats at draft forward, 12.18 m and aft, 12.75 m. She discharged her ballast 200 MT
from APT and pump-In ballast also 200 MT to her FPT. Find final drafts after the operation.

7.1.2 MV Max Panama is trim by the head with a forward draft of 13.37 m and aft, 13.22 m. To bring the ship
to even keel, you decided to transfer fuel from No.1 FOT to No.5 FOT. Find how much FO must be
transferred.

7.1.3 In MV Max Panama while making your initial cargo distribution, you obtained forward draft 13.07 m
and aft 12.83 m. To make the ship's trim to 0.50 m., you decided to shift cargo from hold 2 to 7. Determine
the quantity of cargo that must be shifted.

7.1.4 While making initial cargo distribution for MV Nara, you obtained forward draft of 9.35 m and aft 9.75
m. How much cargo must be shifted from hold 4 to hold 1 to make your plan even keel.

7.1.5 While making initial cargo distribution for MV Nara, you obtained forward draft of 8.92m and aft draft
10.38 m. Your intention is to have a trim of O.sOm at departure so you decided to shift cargo from hold 4 to
hold 1. Find the quantity of cargo to be shifted to make your plan 0.50 m trim by the stern.
7.1.6 MV Max Panama is floating at forward draft 11.65 m and aft draft 12.15 m. If 300 MT of FO is shifted
from No.5 FOT to No.1 FOT, find the final draft after the operation.

134
7.1.7 A weight of 180 MT onboard MV Nara is to be shifted. The weight's initial LCG from amidship is 24 m
aft of midship. MV Nara's present draft forward is 9.23m and aft is 9.37 m. Find how far should the weight be
move forward to make the ship even keel. What Is weight's new LCG from amidship.

Loading or Discharging weights

When handling single or several moderate weights, involving SMALL change in mean draft, it is assumed
that the weight is added or removed at the tipping center and to calculate the inkage using the TPC.

The change of trim is then calculated by taking the moment from the LCF.

In these cases, the TPC, MTC, and LCF can be taken from the hydrostatic table for either initial or final draft
condition without appreciable error.

This method is used when loading and/or discharging weights involving small change in mean draft. As in
small changes of mean draft, the values of TPC, MTC and LCF may not change considerably.

Example 7.9

M. V. Nara's present draft forward is 4.40 m and aft,S. 06 m. Find the draft after discharging 446 MT ballast
water from No.1 WBT F (P&S).

Solution:

1. Find the mean draft; Mean draft = ( fwd draft + aft draft) / 2
= 4.73 m

2. Take from the hydrostatic tables the values of MTC, LCF[x];


TPC = 32. 87 t MTC = 294.60 t-m LCF[x] = -4.519 m

3. Correct the LCG of No.1 WBT F (P&S) to distance from CF.

LCG of No.1 WBT F (P&S) = -54.173 m


LCF = -4.519 m
------------------------------------------------------------
Distance from CF = 49.654 m

weight x distance from LCF


4. Find the change of trim; Change of Trim = ------------------------------------
MTC x 100

135
446 x 49.654
Change of Trim (CoT) = ------------------------------------
294.6 x 100
Change of trim (CoT) = 0.752m

Find change of draft forward; 1/2 LPP + LCF


Cd Fwd = ------------------- x Ct
LPP
75.5 + (-4.519)
Cd Fwd = ------------------- x 0.752 m
151
Cd Fwd = 0.353 m

5. Find Change of draft aft (Cd Aft) = Ct - Cd Fwd

Ct = 0.752 m
Cd Fwd = 0.353 m
---------------------------
Cd Aft = 0.399 m

6. Apply bodily sinkage or rise and the Cd Fwd and Aft to Initial Draft Fwd and Aft.

Fwd Aft
Bodily rise or sinkage:
Initial Draft = 4.40 m 5.06 m
Bodily Rise = 0.136 m 0.136 m
----------------------------------------------------------------- Rise = weight / (TPC x 100)
= 4.264 m 4.924 m
Cd Fwd and Aft = 0.353 m 0.399 m
Final Draft = 3.911 m 5.323 m

Example 7.10

MV Rubin Panama has draft forward of 13.00 m, and aft is 13.52 m.


1. How much ballast must be pump into FPT to bring her to even keel.
2. What is the final draft after ballasting.

Solution:

1. Find the mean draft; Mean draft = (13.0 + 13.52) / 2


= 13.26 m

2. Take from the hydrostatic tables the values of MTC, LCF[x];


TPC = 64.4 t MTC = -983.3 t-m LCF[x] = 0.29 m

3. Correct the LCG of ___________________ to distance from CF.

LCG of FPT = -102.99 m


LCF = 0.29 m
----------------------------------------------
Distance from CF = 103.28 m
4. Find the change of trim; Change trim = Final Trim -Initial Trim

Final Trim = 0.00 m

136
Initial Trim = 0.52 m
-------------------------------------------------
Ct = -0.52 m

5. Find Change of draft fwd;


107.5 + (0.29)
Cd Fwd = ------------------- x 0.52 m
215
Cd Fwd = 0.26 m

6. Find Change of draft aft (Cd Aft) = Ct - Cd Fwd


Ct = 0.52 m
Cd Fwd = 0.26 m
-------------------------------------------------
Cd Aft = 0.26 m

7. Find the weight;


Wt = (Ct x MTC x 100) / distance
Wt = (O.52 x 983.3 x 100) /103.28
Wt = 495.1 MT

1. Find sinkage = weight / (TPC x 100)


Sinkage = 495.1/ (64.4 x 100)
Sinkage = 0.077 m

10. Apply Sinkage and the Cd Fwd and Aft to Initial Draft Fwd and Aft.

Fwd Aft
Initial Draft = 13.00 m 13.52 m
Sinkage = 0.077 m 0.077 m
-------------------------------------------------------------------------------------------------
= 13.077 m 13.597 m
Cd Fwd and Aft = 0.26 m 0.26 m
-----------------------------------------------------------------------------------------
Final Draft = 13.337 m 13.337 m

Simultaneously loading or discharging weights

When handling several weights involving small change of mean draft, similar method as when handling
single weight can be employed. Only this time, new LCG representing all the weights is first calculated. This
is then corrected for the distance from CF. Then on, calculation is done as previously described.

137
Example 7.11

M.V. Max Panama had an initial draft of forward 3.75m and aft 7.09m. 5he loaded IFO, MOO, FW to the
following tanks/compartment:

IFO No.1 FOT (C) = 350 MT MOO No.1 DOT (5) = 60 MT


No.2 FOT (P) = 150 MT FW OR.WT. (P) = 50 MT
No.2 FOT (5) = 200 MT FWT (P) = 80 MT
No.4 FOT (C) = 250 MT FWT (5) = 80 MT

She also discharged ballast water from:


APT = 589 MT

Find the drafts after completion of operations.

How to Find Trim When Handling Several/Moderate Weights

1. Find present mean draft: Draft: Fwd =

Present Mean Draft = (Draft Fwd + Draft Aft) / 2


Draft: Aft = /2
=
2. Take from hydrostatic tables corresponding LCF, Present Mean =
MTC and TPC. Draft

LCF =
MTC =
3. Find the total weight and weight moment: TPC =
LCG moment = weight x LCG Weight
Compartment LCG LCG Moment
Loaded/Disch
Consider all signs when calculating the weight
LCG moment.
a. Loaded weights treat as positive
b. Discharge weights treat as negative
c. LCG forward of midship treat as negative
d. LCG aft of midship treat as positive

4. Find the total weight new LCG:

New LCG = Total moment/total weight

5. Correct the total weight New LCG to distance


from CF. Total = (w)
Note: same sign subtract; different sign add. New LCG =
Corrected weight distance fwd of LCF assign LCF =
negative, and aft of LCF assign positive. Corrected distance from CF (d) =

6. Find the change of trim (Ct): Cot = w x d / (MTC X 100) =

Cot = w x d / (MTC x 100): (Cot in meters)

7. And the change of draft fwd and aft (in meters): Cd Fwd = ((LPP/2) + LCF) / LPP x Cot =

Cd Fwd = ((LPP/2) + LCF) / LPP x Cot Cd Aft =


Cd Aft = Ct - Cd Fwd

138
8. Find the sinkage or rise due to loaded or Draft Fwd Aft
discharged weight. Present draft =
Sinkage or Rise =
Sinkage or Rise = Total weight /(TPC x 100) New Draft due
sinkage / rise =
Cd Fwd and Cd Aft =
9. Find new drafts fwd and aft: Final Draft =
Apply the sinkage or rise to present draft and
apply Cd Fwd and Cd Aft to get Final Draft Mean Draft =

Note: Above solution is based on longitudinal distances measured from amidships.

The Use of Trimming Tables

Ship's are provided with trimming tables in its loading manuals. This may be presented In different forms but
their basic elements are the same.
Trimming tables are used to quickly calculate the resulting draft after handling weights, may it be single or
several moderate weights.
Trimming tables are arranged to show the correction of fore and aft draft in units of cm (or mm, depending
on the table) due to loading 100 tons at any position.
When the actual weight handled is not 100 tons, the corrections must be multiplied by the weight ratio.
Corrections to draft carry signs which must be reverse when unloading.

TRIMMING TABLES SHOULD NOT BE USED IN CALCULATING CHANGE OF TRIM WHERE THE
CHANGES OF THE MEAN DRAFT IS LARGE.

Example 7.11

Find the final draft of MV Max Panama after loading 250 MT ballast water to FPT. Present draft forward is
11.87 m and aft 12.64 m.

Solution:

1. Get the mean draft; mean draft = (11.87 + 12.64) / 2


= 12.255 m

2. Enter the trim table using the mean draft as argument.


3. To facilitate the solution, tabulate as shown below.

Forward Aft
Compartment Weight Ratio
Corr Product Corr Product
FPT 2.5 6.9 17.25 -3.8 -9.5 cm
4. Apply the correction to initial draft. (When loading follow the sign)

FWD AFT
Initial draft = 11.870 m 12.640 m
Total corr = 0.173 m -0.095 m
--------------------------------------------------------------------------------------------------------------
Final draft = 12.043 m 12.545 m

139
MV MAX PANAMA TRIMMING TABLE

This table shows the correction of fore and aft draft in cm due to loading 100 tons at any position.

Unit: cm

140
EXERCISE 7.3

7.3.1

Forward Aft
Compartment Weight Weight Ratio
Corn Product Corn Product

SUM

Fwd Aft
Initial Draft =
Corn =
----------------------------------------------------
Final Draft =

How to Find Required Trim at Departure to Arrive Even Keel

Calculation to determine the maximum mean draft and correct trim at completion of loading at a loading port
in order that the vessel will arrive to the destination port not exceeding the maximum permissible draft and
on even keel.

Vessel ___________________Loading Port: ___________________Date:_______________

1. Maximum permissible draft at destination:


2. Water density at destination:
3. Displacement from Hydrostatic Table for Draft at (1):
4. L C B from Hydrostatic Table for Draft at (1):

ITEM Weight L C G (x) Longitudinal Moment


5. S. W. Displacement at Arrival
Display (3) x Density (2) / 1.025 =
6. Consumption (+) and Production (-)
-----------------------------------------------
-----------------------------------------------
-----------------------------------------------
-----------------------------------------------
-----------------------------------------------
7. S.W. Displacement at Departure
8. Look-up displacement adjusted for water density of -at loading berth.
Displ (7) x 1.025 / Density =
9. Take the draft corresponding to look up displacement at (8):
Note:
a. This is the maximum mean draft at the loading berth which will result in draft at (1) at
destination.
b. If the vessel is sagged, the mean midship draft must not exceed this value.
c. If the vessel is hogged, the mean fwd and aft draft must not exceed this value.
d. Any safety margin should be deducted from this value.

141
10. L C G at loading berth (same as (7) :
11. L C B at Hydrostatic draft at (9) :
12. M T C at Hydrostatic Data from Draft at (9) :
13. LCF at Hydrostatic draft at (9) :
14. Trimming Arm (LCG- LCB) :
15. Trimming moment (Trimming arm x displ at (8)):
16. Required Trim on completion of loading (Trimming moment /( MTC xl00)):
17. Change of draft fwd and aft:
18. Find Fwd and Aft draft on completion of loading

F A
Corresponding Draft :
Cd Fwd and Aft :
-------------------------------------------------------------------------------------------------------
Fwd and Aft Draft on completion of loading :

EXERCISES 7.4
Problem 7.4.1
MV Max Panama is scheduled to load bauxite in Australia for discharge 1 SB Japan. Maximum allowable
3
draft at discharge port is 11.00 m at BW density 1012. Water density at load port is 1.010 t/m .
During the voyage, estimated fuel and fresh water consumptions are as follows;

FO FOT No.5 (c) = 261.6 MT FW FW T (P & S) = 58.8- MT


FOT No.4 (c) = 120.6 MT
DO DOT No.1 (s) = 19.6 MT
1. Find the trim required at departure from loading port to arrive even keel in Japan.
2. Find the draft at completion of loading.
Solution:
1. Maximum permissible draft at destination:
2. Water density at destination:
3. Displacement from Hydrostatic Table for Draft at (1):
4. L C B from Hydrostatic Table for Draft at (1):
ITEM Weight L C G (x) Longitudinal Moment
5. S.W. Displacement at Arrival
Displ (3) x Density (2) / 1.025 =
6. Consumption (+) and Production (-)
------------------------------------------------
------------------------------------------------
------------------------------------------------
------------------------------------------------
7. S. W. Displacement at Departure

----------------------------------------------------------------------------------------------------------------------------
----------------------------------------------------------------------------------------------------------------------------
----------------------------------------------------------------------------------------------------------------------------
----------------------------------------------------------------------------------------------------------------------------
----------------------------------------------------------------------------------------------------------------------------
----------------------------------------------------------------------------------------------------------------------------
----------------------------------------------------------------------------------------------------------------------------
----------------------------------------------------------------------------------------------------------------------------
----------------------------------------------------------------------------------------------------------------------------

142
Problem 7.4.2
MV Max Panama is fix for a voyage from New Orleans to China via Panama Canal. Maximum allowable
3
draft at the canal is 12.04m, at present water density of 0.996 t/m . At loading berth in New Orleans the
3
density is 1.009 t/m . The estimated consumption from New Orleans to Panama;

FO FOT No.5 C = 163.4 MT DO DOT l(S) = 10.5 MT


1. Find the trim required at departure from loading port to arrive even keel in Panama Canal.
2. Find the draft at completion of loading.
Solution:

1. Maximum permissible draft at destination:


2. Water density at destination:
3. Displacement from Hydrostatic Table for Draft at (1):
4. L C B from Hydrostatic Table for Draft at (1):
ITEM Weight L C G (x) Longitudinal Moment
5. S. W. Displacement at Arrival
Display (3) x Density (2) /1.025 =
6. Consumption (+) and Production (-)
7. S. W. Displacement at Departure

------------------------------------------------------------------------------------------------------------------------------------------------
------------------------------------------------------------------------------------------------------------------------------------------------
------------------------------------------------------------------------------------------------------------------------------------------------
------------------------------------------------------------------------------------------------------------------------------------------------
------------------------------------------------------------------------------------------------------------------------------------------------
------------------------------------------------------------------------------------------------------------------------------------------------
------------------------------------------------------------------------------------------------------------------------------------------------
------------------------------------------------------------------------------------------------------------------------------------------------
------------------------------------------------------------------------------------------------------------------------------------------------
------------------------------------------------------------------------------------------------------------------------------------------------
------------------------------------------------------------------------------------------------------------------------------------------------
------------------------------------------------------------------------------------------------------------------------------------------------
------------------------------------------------------------------------------------------------------------------------------------------------
------------------------------------------------------------------------------------------------------------------------------------------------
------------------------------------------------------------------------------------------------------------------------------------------------
------------------------------------------------------------------------------------------------------------------------------------------------
------------------------------------------------------------------------------------------------------------------------------------------------
------------------------------------------------------------------------------------------------------------------------------------------------
------------------------------------------------------------------------------------------------------------------------------------------------
------------------------------------------------------------------------------------------------------------------------------------------------
------------------------------------------------------------------------------------------------------------------------------------------------
------------------------------------------------------------------------------------------------------------------------------------------------
------------------------------------------------------------------------------------------------------------------------------------------------
------------------------------------------------------------------------------------------------------------------------------------------------
------------------------------------------------------------------------------------------------------------------------------------------------
------------------------------------------------------------------------------------------------------------------------------------------------
------------------------------------------------------------------------------------------------------------------------------------------------
------------------------------------------------------------------------------------------------------------------------------------------------
------------------------------------------------------------------------------------------------------------------------------------------------
------------------------------------------------------------------------------------------------------------------------------------------------
------------------------------------------------------------------------------------------------------------------------------------------------
------------------------------------------------------------------------------------------------------------------------------------------------
------------------------------------------------------------------------------------------------------------------------------------------------

143
TRIMMING POURS

The final stage of loading a bulk cargo either solid or liquid is normally done with trimming pours. Trimming
pours is the final quantity of cargo held in reserve to be loaded partly into a forward and partly into an after
position as necessary to bring the ship's mean draft and trim to the desired values.

Trimming pours are necessary because the ship's draft and trim in the later stages of loading are seldom
exactly in accordance with the calculated values.
Such errors maybe caused by:
1. Inaccurate quantities loaded.
2. Failure to load the cargo in the exactly intended position.
3. Inaccuracies in the loading calculations.
4. Inaccuracies in the assumption of the weights LCG's.
5. Error in the assumed position of the constant.

When the stage of loading trimming pours is reach the loading is halted and a draft survey is performed to
determine precisely the quantity of cargo remaining to be loaded.

Quantity of cargo set aside for trimming pours.

Type of ship BWt Trimming


(mt) (mt)
Cape-sized 170,000 5,000
Panamax 85,000 2,500
Handy-sized 27,000 1,000
Mini-bulker 3,000 300

To distribute the remaining cargo between forward and aft holds to obtain the required trim.
1. Find the present mean draft.

2. From the hydrostatic table find the TPC, MTC, and LCF at present mean draft.

3. Correct the LCG of aft hold to distance from LCF.

4. Find the distance between forward and aft holds.

5. Find the change in trim.

6. Use the formula to determine the weight to be loaded forward.

The formula used to find the weights for forward and aft compartments:

[(Dist aft hold from CF x remaining cargo to load) - (Cot x MTC x100)]
Wt Fwd = --------------------------------------------------------------------------------------------
Distance between hold
Wt Aft = remaining cargo - Wt Fwd
Change of Trim = required trim - Present trim
Note: Sign of Cot
Change of trim Sign of Change of
Trim (Ct)
Present trim by the stern is increased (Cot is going aft) Positive
Present trim b the stern is reduced (Cot is going fwd) Negative
Present trim b the head is removed (Cot is going aft) Positive
Present trim by the head is change to by stern (Cot is going aft) Positive

144
EXERCISES 7.5

Problem 7.5.1

MV Max Panama is about to complete loading. The remaining cargo is 1965 MT. Her present draft forward is
11.07m and aft 11.35m. Distribute the remaining cargo between holds 2 and 6 to get a required trim of even
keel.

------------------------------------------------------------------------------------------------------------------------------------------------
------------------------------------------------------------------------------------------------------------------------------------------------
------------------------------------------------------------------------------------------------------------------------------------------------
------------------------------------------------------------------------------------------------------------------------------------------------
------------------------------------------------------------------------------------------------------------------------------------------------
------------------------------------------------------------------------------------------------------------------------------------------------
------------------------------------------------------------------------------------------------------------------------------------------------
------------------------------------------------------------------------------------------------------------------------------------------------
------------------------------------------------------------------------------------------------------------------------------------------------
------------------------------------------------------------------------------------------------------------------------------------------------
------------------------------------------------------------------------------------------------------------------------------------------------
------------------------------------------------------------------------------------------------------------------------------------------------
------------------------------------------------------------------------------------------------------------------------------------------------
------------------------------------------------------------------------------------------------------------------------------------------------
------------------------------------------------------------------------------------------------------------------------------------------------
------------------------------------------------------------------------------------------------------------------------------------------------
------------------------------------------------------------------------------------------------------------------------------------------------
------------------------------------------------------------------------------------------------------------------------------------------------
------------------------------------------------------------------------------------------------------------------------------------------------
------------------------------------------------------------------------------------------------------------------------------------------------
------------------------------------------------------------------------------------------------------------------------------------------------
------------------------------------------------------------------------------------------------------------------------------------------------
------------------------------------------------------------------------------------------------------------------------------------------------
------------------------------------------------------------------------------------------------------------------------------------------------
------------------------------------------------------------------------------------------------------------------------------------------------
------------------------------------------------------------------------------------------------------------------------------------------------
------------------------------------------------------------------------------------------------------------------------------------------------
------------------------------------------------------------------------------------------------------------------------------------------------
------------------------------------------------------------------------------------------------------------------------------------------------
------------------------------------------------------------------------------------------------------------------------------------------------
------------------------------------------------------------------------------------------------------------------------------------------------
------------------------------------------------------------------------------------------------------------------------------------------------
------------------------------------------------------------------------------------------------------------------------------------------------
------------------------------------------------------------------------------------------------------------------------------------------------
------------------------------------------------------------------------------------------------------------------------------------------------
------------------------------------------------------------------------------------------------------------------------------------------------
------------------------------------------------------------------------------------------------------------------------------------------------
------------------------------------------------------------------------------------------------------------------------------------------------
------------------------------------------------------------------------------------------------------------------------------------------------
------------------------------------------------------------------------------------------------------------------------------------------------
------------------------------------------------------------------------------------------------------------------------------------------------
------------------------------------------------------------------------------------------------------------------------------------------------
------------------------------------------------------------------------------------------------------------------------------------------------
------------------------------------------------------------------------------------------------------------------------------------------------
------------------------------------------------------------------------------------------------------------------------------------------------
------------------------------------------------------------------------------------------------------------------------------------------------
------------------------------------------------------------------------------------------------------------------------------------------------
------------------------------------------------------------------------------------------------------------------------------------------------

145
Problem 7.5.2

MV Max Panama is about to complete loading. Her present draft forward is 11.45m and aft is 11.27m.
Remaining cargo is 2040 MT. Distribute the remaining cargo between hold 1 and 7 to obtain a trim of 0.40m
by the stern.

------------------------------------------------------------------------------------------------------------------------------------------------
------------------------------------------------------------------------------------------------------------------------------------------------
------------------------------------------------------------------------------------------------------------------------------------------------
------------------------------------------------------------------------------------------------------------------------------------------------
------------------------------------------------------------------------------------------------------------------------------------------------
------------------------------------------------------------------------------------------------------------------------------------------------
------------------------------------------------------------------------------------------------------------------------------------------------
------------------------------------------------------------------------------------------------------------------------------------------------
------------------------------------------------------------------------------------------------------------------------------------------------
------------------------------------------------------------------------------------------------------------------------------------------------
------------------------------------------------------------------------------------------------------------------------------------------------
------------------------------------------------------------------------------------------------------------------------------------------------
------------------------------------------------------------------------------------------------------------------------------------------------
------------------------------------------------------------------------------------------------------------------------------------------------
------------------------------------------------------------------------------------------------------------------------------------------------
------------------------------------------------------------------------------------------------------------------------------------------------
------------------------------------------------------------------------------------------------------------------------------------------------
------------------------------------------------------------------------------------------------------------------------------------------------
------------------------------------------------------------------------------------------------------------------------------------------------
------------------------------------------------------------------------------------------------------------------------------------------------
------------------------------------------------------------------------------------------------------------------------------------------------
------------------------------------------------------------------------------------------------------------------------------------------------
------------------------------------------------------------------------------------------------------------------------------------------------
------------------------------------------------------------------------------------------------------------------------------------------------
------------------------------------------------------------------------------------------------------------------------------------------------
------------------------------------------------------------------------------------------------------------------------------------------------
------------------------------------------------------------------------------------------------------------------------------------------------
------------------------------------------------------------------------------------------------------------------------------------------------
------------------------------------------------------------------------------------------------------------------------------------------------
------------------------------------------------------------------------------------------------------------------------------------------------
------------------------------------------------------------------------------------------------------------------------------------------------
------------------------------------------------------------------------------------------------------------------------------------------------
------------------------------------------------------------------------------------------------------------------------------------------------
------------------------------------------------------------------------------------------------------------------------------------------------
------------------------------------------------------------------------------------------------------------------------------------------------
------------------------------------------------------------------------------------------------------------------------------------------------
------------------------------------------------------------------------------------------------------------------------------------------------
------------------------------------------------------------------------------------------------------------------------------------------------
------------------------------------------------------------------------------------------------------------------------------------------------
------------------------------------------------------------------------------------------------------------------------------------------------
------------------------------------------------------------------------------------------------------------------------------------------------
------------------------------------------------------------------------------------------------------------------------------------------------
------------------------------------------------------------------------------------------------------------------------------------------------
------------------------------------------------------------------------------------------------------------------------------------------------
------------------------------------------------------------------------------------------------------------------------------------------------
------------------------------------------------------------------------------------------------------------------------------------------------
------------------------------------------------------------------------------------------------------------------------------------------------
------------------------------------------------------------------------------------------------------------------------------------------------
------------------------------------------------------------------------------------------------------------------------------------------------
------------------------------------------------------------------------------------------------------------------------------------------------

146
TRIM CONSIDERATIONS

In calculating the ships draft and trim there are several items that need to be considered, such as, how to
distribute the trim change between forward and aft drafts, and the effect of change in density to the trim.

DISTRIBUTION OF TRIM BETWEEN FORWARD AND AFT DRAFT

Change of draft forward and aft will obviously occur when the ship changes trim. If the ship pivots exactly at
amidships then the change of draft aft will be equal to the change of draft fwd and these will be equal to
change of trim divided by two.

Ship pivot on the center of flotation and this point is not always at the amidships. The distribution of change
of draft is therefore not equal forward and aft but proportionate to the distance of LCF from the
perpendiculars.

Figure 7.12 -The distribution of trim as the ship pivots on the LCF and distances measured from amidships.

11. Consider a ship on even keel and the weight at the fore part is transferred to aft.

trim

4 Tan 0 = ------------
LPP
Change of draft fwd Change of draft fwd trim
Tan e = --------------------------- -------------------------- = ------------
1/2 LPP + LCF LPP + LCF LPP
(1/2 LPP + LCF) X trim
Change 0 = ------------------------------
LPP

147
Figure 7.12 - Calculation of change of draft when distances are measured from aft perpendicular.

AP FP

Waterline

Initial draft aft Initial draft

Trim
Tan e = ---------
LPP
Change of draft aft
Tan e = -------------------------
LCF [AP]

Change of draft aft trim


------------------------------- = ---------
LCF [AP] LPP

LCF (AP) x trim


Change of draft aft = ---------------------
LPP

148
CHANGE OF TRIM DUE TO CHANGE IN DENSITY

When a ship passes from water of one density to water of another density her mean draft is changed. As the
draft changes, the longitudinal position of the center of buoyancy will change causing the trim to also
change.

The formula for change in trim due to change in density;

Displ x (LCB1-LCB2)
Change of Trim (in meters) = --------------------------
MTC X 100

Where:
Displacement = displacement in MT at SW draft
MTC = MTC at SW draft
LCB 1 = is the LCB at SW draft
LCB 2 = is the LCB at BW draft

When moving from SW to BW I change of trim is by the head If the position of CB moves aft.

Example 7.12

MV Rubin Panama displaces 69658 MT and floats at drafts: forward 11.48 m and aft 12.24 m in density
1.025 t / m^3. She is to enter dock water density 1.005 t/m^3. Find her drafts fore and aft in dock water,
taking into account the change of trim due to change of density.

Example 7.13
3
MV Rubin Panama is to dock even keel at draft of 12.50 meters in dock water density of 1.009 t/m .
Calculate the end drafts in SW before she enters port, taking into account the change of trim due to change
of density.

149
150
CHAPTER 8

PART III: STABILITY

CHAPTER 8 -Transverse Stability

151
CHAPTER 8

IN THIS CHAPTER
Transverse Stability

Behavior of Ships When Influenced by Internal Force


Behavior of Ships When Influenced by External Force
Transverse Ship of B
The Righting Lever (GZ)
Equilibrium of a Floating Ship

The Initial Stability

The Metacenter
The Metacentric Height
Light Ship KG
Final KG
Free Surface Correction

Assessing The Initial Stability

Stability at Large Angles of Inclinations

The Statical Stability Curve


The Assumed Righting Lever
Dynamic Stability

Calculating the Initial Stability


Large Angle Stability Calculation
Stability Considerations
Grain Loading Calculations

152
Transverse Stability
STATICAL STABILITY

In the absence of external forces, the ship floating freely in water remains at her original position, in a state
of equilibrium. Equilibrium or balance of forces exists when the force of buoyancy is equal to the weight of
the ship and if the center of gravity and center of buoyancy are vertically in line.

The ship, however, is always exposed to forces which disturbs the state of equilibrium. These forces are

1. Internal forces -forces generated within the ship


2. External forces -forces due to environment such as wind and waves

How well the ship will respond to these forces is described by its stability.

Stability of a ship is defined as the tendency of a ship to return to her original position, after being inclined by
external forces such as wind, waves, etc. This tendency is dependent upon the interaction of forces within
the ship

1. the force of weight of the ship and,


2. force of buoyancy acting on the ship. Figure 8.1

The weight force of the ship is considered to act at a


point called center of gravity. The location of the
center of gravity is described by the longitudinal,
transverse or vertical distance from a reference
point. Vertically, center of gravity is measured from
the keel and referred to as KG.

Up thrust force of buoyancy is considered to act at


the geometric center of the submerged portion of the
ship. This point is called center of buoyancy. The
location of the center of buoyancy is also described
by longitudinal and vertical distances from a
reference point. The distance or height of center of
buoyancy from the keel is called KB.

The location of center of gravity is


dependent on the distribution of
weight on the ship, while the center
of buoyancy is dependent on the
draft of the ship.

The forces of gravity and buoyancy constantly


interact with each other and their locations influence
the ship's stability.

BEHAVIOR OF SHIP WHEN INFLUENCED BY INTERNAL FORCES

A ship in upright position has her center of gravity at


the centerline of the ship. The line of force of gravity
Is also vertically in line with the center of buoyancy.
The force of buoyancy supports the weight of the
ship.

153
When weights on the ship are unevenly distributed Figure 8.2
that the weight on one side is greater than on the
other side of the ship, the center of gravity of the
ship is off the centerline. As the G moves away from
the centerline so does the B until both are again in
the same vertical line. By then, the ship's has
assumed a listed position. The list will be at the side
of G and will stay on that side. If the ship is forcibly
disturbed by external forces, the ship returns to the
same listed position.

The list is removed by adding weight to the opposite


side of the list. (This is so if you are sure that the list
is due to the G off the center line.)

The unbalanced weight causes the ship to list.

The transverse moment is equal to weight times its


distance from center line.

Vertical moment is equal to weight times its distance


from keel.

The transverse shift of G is equal to transverse


moment / displacement.

The vertical shift of G is equal to vertical moment/


displacement.

NOTE:
______________________________________________________________________________________
______________________________________________________________________________________
______________________________________________________________________________________
______________________________________________________________________________________
______________________________________________________________________________________
______________________________________________________________________________________
______________________________________________________________________________________
______________________________________________________________________________________
______________________________________________________________________________________
____________________________________________________________________________________

154
BEHAVIOR OF SHIPS WHEN INFLUENCED BY EXTERNAL FORCES

Transverse shift of B

When external forces (such as wind and wave


action) incline a ship, the inclination does not affect
the position of G. The G remains on the same
position, provided no weights within the ship are free
to move.

Due to the inclination, the shape of the immersed


portion of ship changes. In effect, B moves out to the
low side, seeking the new center of buoyancy of the
under-water volume. The movement of B is parallel
to the shift of the center of gravity of the immersed
and emerged righting wedges.

As B separates from the vertical line over G, an


internal moment is created to counteract the external
force. The internal moment that counteracts the
external force is referred to as the righting moment.

The righting moment will tend to bring the ship back


its upright position. The ship in this case is said to be
stable.

It is the movement of B which results in a tendency


to restore the ship to its original position. The
magnitude of this tendency is a measure of the
ship's stability.

When the center of gravity is to the outboard of the


center of buoyancy, the moment that is created
tends to capsize the ship and the ship is said to be
unstable.

NOTE The Righting Moment - the turning effect that tends to bring the ship back to
upright.
Upsetting Moment - the turning effect that tends to capsize the ship.

155
The Righting Lever (GZ) and Moment of Statical Stability

As the ship inclines, the weight acting downwards and the buoyancy force of equal magnitude acting
upwards no longer lie on the same line but is parallel to each other. Two equal forces acting on a body in
opposite direction and along parallel lines, such as the two lines of force of gravity and buoyancy, form a
couple.

Moment = weight x distance

All couples are expressed as force times distance, or moment. The force being the weight of the ship and
the distance, the horizontal distance between the line of force of G and line of orce of B. The distance
between these lines are constant, -wherever measured. For practical reasons, the distance is measured
from point G. This distance is labeled GZ and also known as the righting lever or righting arm.

The magnitude of the couple about G as expressed in moment is then equal to displacement times GZ.

Moment of statical stability = displacement x GZ

The moment referred to is the moment of statical stability. It can be a righting moment as in a stable
equilibrium or an upsetting moment as in unstable equilibrium.

Since displacement is constant at all angles of heel, the moment of statical stability varies only with the
length of GZ.

The length of the couple depends upon the position of G and B. When the angle of heel is small, B moves
just little to the side, and consequently the distance GZ Is a small. When the angle of heel is increasing, B
will move further out to the side, and GZ becomes bigger. The value of GZ continues to increase until either
the deck edge immersed or the bilge emerges. Hereafter the GZ decreases again.

NOTE A short GZ gives a small stability moment and a long GZ gives a large stability moment.
This makes it important to know the length of GZ at different angles of inclination.

156
EQUILIBRIUM OF A FLOATING SHIP

A body or a ship is in equilibrium when the resultant


of the forces acting on the body is equal and
opposite to the force of gravity acting on the body,
and the forces are in the same vertical line.

The ship that is floating freely is in a state of


equilibrium. The force acting downwards in G, and
the force acting upwards in B, has same value, in
opposite direction and vertically in line.

There can be three possible conditions of equilibrium


for a ship, depending on how she will respond to
external forces such as wind and waves.

Stable Equilibrium

A ship is in stable equilibrium if, when disturbed from an equilibrium position, such as from upright, and
inclined by an external force, she tends to return to her initial condition. A ship of stable equilibrium
possesses positive stability. Figure 2.4 shows how this occurs.

Figure 8.5

1. The ship is disturbed from equilibrium or


inclined by an external force such as wind
and waves.

2. The position of G is not affected by the


heel and remains in its position. The force of
gravity is considered to act vertically
downward through this point.

3. The heel changes the shape of the


immersed portion and the center of
buoyancy B moves to the new position of
the center of under water volumetowards
the low side.

4. The upward force acting on the new


center of buoyancy and the downward force
acting on the center of gravity separate and
create a couple to counteract the external
force and return the ship to upright. The
couple is referred to as righting moment.

NOTES________________________________________________________________________________
______________________________________________________________________________________
______________________________________________________________________________________
______________________________________________________________________________________
______________________________________________________________________________________

157
Unstable Equilibrium

A ship is in unstable equilibrium if, when disturbed from an equilibrium position, such as from upright, and
inclined by an external force, she tends to incline further rather than return to her initial condition. A ship of
unstable equilibrium possesses negative stability.

Figure 8.6

1. The ship is inclined by an external


force.

2. G remains on the same position on


the ship.

3. While B moves to new center of


underwater volume of the ship. But B
lies nearer to the center line than G.

4. The line of force of buoyancy and the


line of force of gravity create an
upsetting moment which inclines the
ship still further from the vertical and
may heel the ship to a considerable
angle or even capsize.

Neutral Equilibrium

A ship is in neutral equilibrium if, when the ship is disturbed from an equilibrium position, such as from
upright and inclined by an external force, she has no tendency to return to her initial condition or a tendency
to incline even further. A ship of neutral equilibrium possesses neutral stability.

For a ship-shape, neutral equilibrium can only be maintained for a very small angle of heel. After which a
small righting moment would develop tending to right the ship, or the ship would become unstable and
capsize.

Figure 8.7

1. An upright ship in neutral equilibrium is


disturbed by an external force such as
wind and waves.

2. G is not affected by the heel and


remains in its position on the ship.

3. B moves to the new center of buoyancy


but the position remains directly under
G.

4. No couple is created so there is no


righting moment or heeling moment.

5. When the external force is removed, the


ship remains in the new inclined
position until another external force is
applied.

158
THE INITIAL STABILITY

One of the objectives of masters and officers of a


ship is to ensure that the ship remains stable at all
times and cannot capsize. The immediate concern is
to know how the ship will behave when inclined to a
small angle under the action of wind and waves. It is
to the best interest also to know the ship's stability at
large angles of inclination - meaning during heavy
weather, heavy lift operations, or to correct
conditions of the ship when damaged due to
flooding, cargo shifting, or grounding.

Stable, unstable and neutral equilibrium refers only


to initial stability, that is, the tendency of the ship at
small angles of inclination. It is important to know
how the ship will behave when inclined to a small
angle under the action of the wind and the waves
since; being inclined to a small angle is what the
ship is almost always subjected to.

What makes the ship stable or not is determined


almost wholly by the transverse location of two
points;
1. the center of gravity (G) and
2. the center of buoyancy (B).

But to be able to calculate the ships initial stability


we need another point called the metacenter.

The Metacenter

Note that the vertical line of force acting through B1 Intersects the original line of force of buoyancy. This
point of intersection is called metacenter. For small angles of inclinations, the metacenter can be taken as
fixed point on the center line and referred to as initial metacenter (M). Its position can be describe .by its
height above the keel, called the height of metacenter (KM).

159
The Metacentrlc Height

The ship, when inclined to small angles, the


metacenter coincides with the intersection of
the centerline and line of buoyancy force.
Making it easy to calculate the length of GZ.
As the buoyancy line of force, the center line
and GZ forms a small triangle.

The triangle is bounded by the points M, G


and Z. The length of the ship's righting arm,
GZ, is directly proportional to the length of
GM. GM is called the metacentric height.

To find the length of GZ

GZ= GM x sin (angle of heel)

The moment of statical stability at small angles of heel is;

Moment of statical stability = displacement x GM x Sin e

The above consideration applies to the initial stability of the


ship. That is the stability of the ship at about 10-15 degrees
of inclination. At these small angles, the metacenter's
movement is negligible and considered to be a fixed point on
the centerline. At this initial inclination, the GM can be used
to calculate the GZ.

At these "initial" stages of inclination, the GM can be used as


a measure of the "initial stability" of a ship since, GM is a
function of GZ and GZ is a function of righting moment, thus,
GM must be a function of the righting moment. As the GM
increases, so will the GZ and the righting moment. When GM
decreases, the GZ also decreases.

NOTE
As a general rule if initial stability
of a ship is improved, then
stability at all angles of
inclination will also improve.

At greater angles of inclination, M will move and the


metacenter cannot be reckoned as a fixed point anymore.
The GZ is no longer equal to GM x sin 9. Calculating GZ
must be resorted to by using other methods.

160
Lightship KG

The mass of the ship when floating with no cargo, fuel, stores or any other weights not forming part of the
hull or machinery or fixed equipment of the ship is referred to as the lightship. The lightship serves as an
initial condition from which the displacement and Final KG for any condition of loading may be calculated.

The KG for lightship is determined by shipbuilders and provided in the ship's stability manual.

Inclining experiment is performed to determine the KG of a ship in a light condition.


The experiment is carried out by builders when the ship is as near to completion as
possible.

Compartments VCG or KG

The geometric centers of the compartments are usually found in the ship's capacity plan, normally labeled
"KG" or "VCG". This geometric center coincides with the weight's center of gravity when the compartment is
filled with homogenous cargo. When compartments are partially filled, it might be necessary to use the
corrected values of VCG.

Multiplying the weight by its VCG gives the moment of that weight about the keel. This moment is commonly
called the vertical moment. Taking the sum of all vertical moments and dividing by displacement gives the
ship's Final KG.

Vertical moment = weight x its VCG


Vertical moment is abbreviated as VCG_MMT or KG_MMT

Final KG

The final KG is found by using the principle of


moments. To start with, the moment about the keel
at initial condition is required. Adding the moments
about the keel of all weights loaded and
subtracting the moments about the keel of all
weights discharged to the moment at initial
condition gives the final moment. This final
moment must be equal to the final displacement
multiplied by KG. So, dividing the final moment by
displacement gives the final KG.

Final KG = Total vertical moment / Displacement

To obtain moments about the keel, the weight is multiplied by the distance of its center of gravity from the
keel (VCG, KG). The values for the center of gravity of all compartments and tanks are found on the ship's
capacity plan.

161
FREE SURFACE EFFECT CORRECTION

Another factor that affects the location of G is liquids freely moving in ship's tanks. Liquids freely moving in
ship's tanks produce the so-called free surface effect and primarily cause the virtual rise of G.

Values for calculated free surface effects are provided in loading manuals. The effect of free surface
maybe presented in graphical or tabulated form.

In tabulated form, the free surface effects are normally incorporated in tank calibration tables and
shown for each sounding. The free surface effects maybe in terms of free surface Inertia (M4) or free
surface moment (t-m).

To get free surface correction, the free surface Inertia (M4) must be multiplied first with the density
of liquid in the tank and then divided by the displacement.

If the supplied data for free surface effects are already in terms of "moment- (t- m), these are
already multiplied by the assumed density of liquid in tanks. To get the free surface correction they
are to be divided only by displacement.

Examples of the free surface effect values in graphical and tabulated form.

Free Surface Effects Consideration

Free surface effect depends upon the dimension of the liquid surface in the tank than to its quantity. The
effect of the tanks breadth is greater than the effect of its length.

162
ASSESSING THE INITIAL STABIUTY

Once the final KG is determined, the GM can be found by subtracting KG from KM. The GM is further
corrected for the effects of free surface (GGo), to obtain GoM. The GoM is equal to GM minus the GGo.

The relationship between the points may be summarized as,

KM KG KM
- KG + GGo - KGo
------------------------ ------------------------- -------------------------
GM KGo GoM
- GGo
------------------------
GoM

Assessment of Initial Stability


When the ship is in stable equilibrium, M is above G, giving a positive stability, and GM is regarded
as positive. If in unstable equilibrium, M is below G, giving a negative stability and both GM and GZ
are considered negative. When in neutral equilibrium, no moment is acting on the ship, giving
neutral stability. Both GM and GZ are zero. In summary:

M above G GM and GZ is positive initially stable


M at G GM and GZ is zero initially neutral
M below G GM and GZ is negative initially unstable

When G moves farther from M, the GZ also increases. When GM decreases, the GZ also decreases.

163
STABILITY AT LARGE ANGLES OF INCLINATION

It is wrong to assume that the vessel with positive stability can not capsize. Initial stability pertains only to the
initial" response of the vessel to forces causing small angles of Inclination and not for the complete range of
inclination. As mentioned earlier, the GZ lengthens as the ship inclines developing the moment to counter
the heeling forces. But at certain angle of inclination the GZ decreases again. The overall stability of the ship
is best described by constructing the statical stability curve.

THE STATICAL STABILITY CURVE

To determine the overall stability of the ship at any particular loading condition, the value of GZ for each
angle of inclination must be calculated. The calculation is based on the location of G for that particular
loading condition.

With the final GZ known for a series of angles of inclination, these can be plotted as the statical stability
curve of the ship for that particular displacement and KG. From this graph we can obtain accurate
conclusions as to the stability of the ship.

1. the range of stability


2. the angle of vanishing stability
3. the maximum GZ
4. the angle at which the maximum GZ occur
5. the dynamical stability

Statical stability curve is the graphical representation of the length of the GZ at all angles of heel. The curve
is obtained by plotting the GZ against the angle of heel as shown in figure 8.16.

It must be emphasized that only the early part of the curve up to say 400 heel can be regarded as giving a
reasonable representation .of the actual GZ value, as in practice at very large angles of heel beyond
approximately 40 degrees, it is probable that,

1. cargo will have shifted


2. equipment will have broken loose
3. water will have entered the ship

Thus making invalid the assumption that G does not shift.

NOTE The values given by the curves are all based on the underwater form of the ship. The
ship designer assumes the ship to be in even keel with a straight waterplane. Hence the
term static; that is, the values cannot be computed for an infinite number of wave
profiles and trimmed waterplanes such as a ship under dynamic sea conditions.

164
Range of Stability -This is the range over which the ship has positive GZ. The range is from the point where
the curve crosses the baseline and Increases in positive value to the point where the GZ returns to zero. It
must be emphasized that this range of stability is only theoretical one. It is based on the assumption that G
has not moved in all angles of inclination. In practice however, at angle say greater than 40, cargoes may
have already shifted, equipment broken loose or water entered the ship causing the G to shift. If so, the
calculated GZ no long represents real GZ.

Vanishing point -or angle of vanishing stability, is that angle of heel at which the GZ returns to zero. This is
the end point for the range of stability.

Maximum GZ -is indicated by the highest point in the curve. This is the longest GZ or the largest separation
between the forces of buoyancy and gravity. The angle at which the maximum GZ occurs is important. At
this angle, the ship exerts the most energy to right itself. The maximum stability is associated with the angle
of deck edge Immersion as it is near this angle that the maximum GZ occurs.

Initial metacentric height -indicates the initial slope of the GZ curve at the origin. If a perpendicular line is
drawn through 57.3 to a height equal to the value of GM and a line drawn to the origin, then the slope so
created indicates the initial slope of the GZ curve.

Dynamical Stability -is the work expended in heeling a ship from upright to a specified angle of inclination.
When the ship is heeling the resistance is also increasing until the GZ or righting arm reaches maximum.
The energy building up in the ship is the dynamical stability, Indicated by the area under the GZ curve up to
the angle of heel. The general value of dynamical stability to any angle of heel can be found if the curve of
statical stability has been constructed.

Dynamical Stability = displacement x area under the stability curve up to angle of inclination

165
Figure 8.17 -- OVERALL STABILITY CHARACTERISTIC

1. Initial Stability (GM) Indicated by the slope of the curve at the origin

2. Maximum righting arm Maximum vertical distance from the baseline to the
original curve

3. Maximum righting moment Multiply displacement by maximum GZ

4. Angle of maximum stability Corresponds approximately to the angle of deck


edge immersion

5. Angle of maximum list Corresponds approximately to the angle of deck


edge immersion (unless range is greater than 90)

6. Angle of dangerous roll Deck edge Immersion

7. Angle of dangerous list Corresponds approximately to one-half the angle of


deck (off-center weight only) edge immersion

8. Angle of dangerous list (due Any angle is dangerous when due to negative GM
to negative GM)

9. Angle of list (due to off- Indicated by intersection of (a) cosine curve, or (b)
center weight) corrected stability curve

10. Angle of fist (due to negative Indicated by intersection of (a) sine curve with original
GM) curve or (b) corrected stability curve with baseline

11. Angle of maximum roll (end Vanishing point is located at intersection of stability
of range of stability) curve with base line

12. Dynamic stability Area under stability curve and above base line.

166
THE ASSUMED RIGHTING LEVER

The use of metacentric height (GM) alone as an indication of stability is valid only for initial stability, that is,
the tendency of the ship at small angles of inclination. The GZ at small angles of inclination can be
represented by GM x sin 9. As the ship is inclined to a greater angle the metacenter moves away from its
initial position. Hence, the righting arm (GZ) can no longer be equal to GM x sin 9.

To know the GZ of the ship at large angles of inclination, the following methods can be used.

1. The use of Cross Curves of Stability (graphical or tabulated form from stability manual).

2. The use of KN (graphical or tabulated form from stability manual).

3. Wall-sided formula (only up to the angle of deck edge immersion).

The Use of Cross Curves of Stability

As a basis for calculating the "real" GZ, ship designers provide the lengths of GZ's base on assumed
position of G. These may be in form of curves, known as cross curves of stability. The length of the GZ is
read at the vertical scale and the displacement at the horizontal scale.

Figure 8.18 CROSS CURVES OF STABILITY

167
To use the cross curves of stability (follow the numbers in figure 8.18),

1. Locate the displacement of the ship on the horizontal scale and, through this point draw a
perpendicular line to cut all the "angle of inclination" curves.
2. The length of the GZ is then the intersection of the perpendicular line and the curve. Read the GZ
for each "angle of inclination" at the vertical scale of the graph.
3. The GZ obtained from this graph is for the assumed KG, and this must be corrected to get
the "real" GZ .based on the true KG of the ship for a particular loading condition.

There are two variants of presenting the length of GZ base on the assumed KG. Ship designers may provide
the length of GZ base on assumed height of G from keel (assumed KG not zero) and G at the keel
(assumed KG equal zero).

The Use of KN Table

To aid the ship officer determine the stability of his ship at a large angles of inclinations, ship builders
provide the length of GZ for a series of angles of inclination at any particular displacement about an
assumed position of center of gravity of the ship.

Ship designers provide the horizontal distance (assumed GZ) from K to a vertical line through B at a series
of inclinations. This horizontal distance is known as KN. The KN must be corrected to obtain the true GZ.

Figure 10.19

where KGo = the height of center of gravity corrected for free surface effect.
KGo = Final KG + GGo

The final or true GZ can be found:


Final GZ = KN -( Final KG x sin e) or

GoZ = KN -(KGo x sin e)

168
The Wall-Sided Formula

Up to an angle at which the deck edge immersed, GZ can be approximated by the wall- sided formula.

2
GZ = (GM + BM tan e) sin e

The wall-sided formula is based on the assumption


that the sides of the ship within the immersed wedge
and emerged wedge are parallel (modern merchant
ship's have parallel mid body) and will give accurate
results so long as successive waterplanes intersect
at the centerline.

Figure 8.20 -The ship sides at the immersed and


emerged wedges are parallel.

The wall sided formula can be used to obtain GZ at


any angle up until the assumptions becomes invalid.
One condition that invalidates the assumption is
when the deck edge immerses or when the bilge
emerges.

Figure 8.21 -Ship's side is no longer parallel at the


immersed and emerged wedges.

Dynamic Stability

Dynamic stability is the work done in inclining the ship to that angle. It is the energy expended to resist
external heeling forces or the energy the ship possesses to right itself. When the ship is exposed to heeling
moments such as beam wind the ship inclines. If the heeling energy imparted instantaneously, the ship must
be able to overcome that energy or it will capsize.

The righting moment (GZ x displacement) at each angle heel when plotted produces the righting moment
curve. Since the displacement remains constant through all angles of heel, the righting moment curve
retains the same shape as the righting arm curve. The vertical scale for the righting arm can be change to
righting moment by a factor of displacement.

Dynamical stability is equal to the area under the curve to the angle concerned and multiplied by the ship's
displacement.

Dynamic stability = displacement x area under the GZ curve to the angle of heel.

169
To calculate the area under the curve up to say 400 angle of heel is by means of Simpson's Formula.

Figure 8.22

ANGLE OF INCUNATION
common interval

Using Simpson's formula, the curve is divided into parts of equal intervals. The dividing line is called ordinate
and the distance between ordinates is called common interval.

In the above figure, the length of the curve is up to 40 degrees. Dividing this into four parts, results into a
common interval of 10 degrees. In the figure we have five ordinates (the ordinates can be numbered for
identification). The length of ordinate is equal to GZ.

To calculate the area, we use the Simpson's First Rule. The data is organized into a table.

Angel of
GoZ SM Product
Heel The column SM is the Heel Simpson's
0 0.0 1 0 multiplier or factor. The column product is
10 0.408 4 1.632 length of ordinate (i.e. GoZ) multiplied by the
20 0.882 2 1.764 factor. At the bottom of product column is the
30 1.209 4 4.836 total of the products.
40 1.270 1 1.27
Sum
Products 9.502

To get the area, the sum of product is multiplied by length of common interval and then divided by the
constant 3. The unit of the area will be consequently meter- degrees. To convert to meter-radians, use the
factor, 1 radian is equal to 57.3 degrees.

170
Sum of products x interval
Areao-40 = ------------------------------------ / 57.3
3
9.502 x 10
Areao-40 = ------------------------------------ / 57.3
3
AreaO-40 = 0.552 m -radians

The Simpson's Second Rule can be also used to calculate the area under the curve up to for example 300
angles of heel.

Figure 8.23

ANGLE OF INCLINATION
common interval

Angel of
GoZ SM Product
Heel The column SM is the Simpson's multiplier or
0 0.0 1 0 factor. The column product is length of
10 0.408 3 1.224 ordinate (i.e. GoZ) multiplied by the factor. At
20 0.882 3 2.646 the bottom of product column is the total of the
30 1.209 1 1.209 products.
Sum
Products 5.079

[Sum of products x interval x 3


AreaO-30 = ---------------------------------------- / 57.3
8
5.079 X l0 3l
Areao-30 = ---------------------------------------- / 57.3 = 0.332 m-radlans
8

171
SUMMARY
Stable, unstable, and neutral equilibrium refer to the initial stability of the ship, that is
from 0 to about 10-15 degrees angle of heel. The ship's initial stability is indicated by
its GM.

The overall stability of the ship at all angles of inclination from 0 - 90 degrees is
represented by the statical stability curve.

The area under the righting arm curve is the energy required to heel the ship to a
particular angle. This is called dynamical stability.

The ship stability calculation is divided into several segments,

1. The Initial Stability -calculation of GM


2. Stability at All Angles -calculation of GZ or righting arm and statical stability curve
3. Dynamical Stability -calculation of area under the curve.

IMO RECOMMENDED CRITERIA ON INTACT STABILITY (Resolution A. 167)

GZ shall be at least 0.20


m at an angle equal or
greater than 30 Max. GZ shall occur at more
than 30 angle of heel but not
less than 25 degrees

172
1. The area under the righting lever curve (GZ curve):
- from 0 to 30 degrees angle of heel should not be less than 0.055 meter- radians, and
- from 0 to 40 degrees angle of heel or angle of flooding (if this is less than 40) should not be less
than 0.090 meter-radians

2. The area under the righting lever curve (GZ curve):


- between the angles of heel of 30 to 40 degrees or between 30 and angle of flooding (if this is less
than 40) should not be less than 0.03 meter-radians

3. The righting lever GZ should be at least 0.20 m at an angle of heel equal to or greater than 30
degrees.

4. The maximum righting arm should occur at an angle of heel preferably exceeding 30 but not less
than 25 degrees

5. The initial metacentric height GM, should not be less than 0.15 meter.

Note: Angle of flooding Is an angle of heel at which openings in the hull, superstructures or
deckhouses which cannot be closed weather tight, immerse.

CALCULATING THE INITIAL STABILITY

Final KG = Total vertical moment / Displacement

GM = KM -KG (Solid GM)

GGo = Total Free Surface Moment / Displacement

GoM = GM -Ggo (Liquid GM; GM corrected for the effects of free surface)

KGo = KG + GGo

GoZ = GoM x sin 8

G = Center of Gravity
B = Center of Buoyancy (upright)
B1 = Center of Buoyancy (inclined)
M = Metacenter
GM = Metacentric Height
BM = Metacentric Radius
GZ = Righting arm
KB = Height of Center of Buoyancy
KG = Height of Center of Gravity

As a first requirement in the practical application of stability principles by ship's officer, full information
concerning his vessel should be at hand. That is the ship's stability and loading manual.

173
For the ship's officer, stability is mainly a problem of finding the position of the vessel's vertical center of
gravity, KG, and obtaining the height of meta center, KM. Subtracting KG from KM produces meta centric
height (GM). KM is readily available to him from the vessel's hydrostatic data, while the KG must be
calculated for that particular loading condition.

Figure 8.25 -Representation of a compartment's VCG.8

Vertical moment = weight x its VCG

Final KG = total vertical moment/ displacement

Multiplying the weight by its VCG gives the moment of the weight about the keel. This moment is commonly
called the vertical moment. Taking the sum of vertical moments and dividing by displacement gives the final
KG (representing the vertical distance of the center of gravity of the ship from kee).
Once the final KG is determined, the GM can be found by subtracting KG from KM. The GM so found can be
then corrected for the effect of free surface -GGo, to obtain GoM. The GoM is equal to GM minus the GGo.

The relationships can be summarized in two ways,

KM KG KM
- KG + GGo - KGo
----------------------- ----------------------- ------------------------
GM KGo GoM
- GGo
-----------------------
GoM

THE STABILITY and TRIM CALCULATION SHEET

The effects of handling several weights can be calculated at once using the stability calculation sheet. The
form is essentially used to conveniently calculate the ship's final KG after several weight movements. The
form organizes the weights and VCG's in rows and columns to facilitate its multiplication and the adding of
moments obtained. It can also serve as a record of stability calculations taken.

Example 8.1 - MV Mini-Bulker Stability Calculation Sheet

You plan to load the MV Mini-Bulker as shown.


1. Calculate the ship's final KG.
2. Calculate the ship's GM corrected for free surface.
3. What kind of stability the ship possess?

ITEM WEIGHT KG KG M'T I.SG


% MID. G M.G M'T (T-M)
(T) (M) (T-M) (T-M)
NO. 1 HOLD 1643 4.67 7673 0
NO. 2 HOLD 3465 4.48 15523 0
CARGP TOTAL 5108
F.P.T.
NO. 1 W.B.T. (P)
NO. 1 W.B.T. (S)

174
NO. 2 W.B.T. (P)
NO. 2 W.B.T. (S)
NO. 3 W.B.T. (P)
NO. 3 W.B.T. (S)
WATER BALLAST TOTAL
F.W.T. (P) 18 7.92 143 8
F.W.T. (S) 18 7.92 143 8
A.P.T 116 7.08 821 803
FRESS WATER TOTAL
F.O.T. (P) 207 3.75 776 243
F.O.T. (S) 205 3.75 769 234
FUEL OIL TOTAL
NO. 1 D.O.T. (C) 63 3.75 236 22
NO. 2 D.O.T. (C) 19 0.61 12 83
DIESEL OIL TOTAL
CONSTANT 48 6.13 294 0
DEADWEIGHT 5802
LIGHT WEIGHT 1771 7.5 13282 0
DISPLACEMENT 7573 39672 1401

3 4 7 6
Solution:

1. F. KG = Total vertical moment/ 4. Find GoM corrected for free surface


Displacement effect.
F.KG = GM =
GGo =
2. From hydrostatic table, take the value of KM, ---------------------
and subtract from it the F.KG GoM =
KM =
Final KG =
------------------ 5. Assessment of initial stability
GM =

3. Calculate the loss of GM due to the effect of free


surface:

GGo = Total free surface moment / displacement


GGo =

------------------------------------------------------------------------------------------------------------------------------------------------
------------------------------------------------------------------------------------------------------------------------------------------------
------------------------------------------------------------------------------------------------------------------------------------------------
------------------------------------------------------------------------------------------------------------------------------------------------
------------------------------------------------------------------------------------------------------------------------------------------------
------------------------------------------------------------------------------------------------------------------------------------------------
------------------------------------------------------------------------------------------------------------------------------------------------
------------------------------------------------------------------------------------------------------------------------------------------------
------------------------------------------------------------------------------------------------------------------------------------------------
------------------------------------------------------------------------------------------------------------------------------------------------
------------------------------------------------------------------------------------------------------------------------------------------------

175
Example 8.2

The M. V. Mini-Bulker's present displacement is 7573 mtons and total vertical moment 39672 tons-m and
free surface effect correction is 0.185 m.
Excerpt from the MINI-BULKER's RIGHTING LEVER TABLE
Righting Arm Table Assumed KG = 0.0 m

Disp. 10 12 15 20 25 30 35 40 45 50 60 75 90
7500 1.385 1.665 2.088 2.804 3.469 4.033 4.504 4.897 5.207 5.435 5.685 5.631 5.149
7550 1.385 1.664 2.087 2.802 3.461 4.020 4.490 4.884 5.194 5.423 5.674 5.625 5.148
7600 1.384 1.663 2.086 2.800 3.453 4.008 4.475 4.870 5.180 5.410 5.663 5.619 5.148
7650 1.383 1.663 2.085 2.797 3.445 3.995 4.461 4.856 5.167 5.397 5.653 5.613 5.147
7700 1.383 1.662 2.084 2.795 3.437 3.983 4.447 4.843 5.153 5.384 5.642 5.607 5.147

1. Calculate the ship's stability for large angles of inclination.


Solution:

1. Calculate GoM and KGo


KM = F.KG =
F.KG = GGo =
------------------------ --------------------------
GM = KGo =
GGo =
------------------------
GoM =

2. Read from the Righting Lever Table the value of KN for each angle of inclination.
3. Calculate the Final GZ for each angle of inclination
GoZ = KN - (KGo x sin 8)

Angle of KN - KGo x Sin 8 = Goz


Inclination (Assumed GZ) - (GZ correction) = KN (KGo x sin8)
10 1.384 - 5.424 x sin 10 =
15 2.086 - 5.424 x sin 15 =
20 2.801 - 5.424 x sin 20 =
25 3.457 - 5.424 x sin 25 =
30 4.014 - 5.424 x sin 30 =
35 4.483 - 5.424 x sin 35 =
40 4.877 - 5.424 x sin 40 =
45 5.188 - 5.424 x sin 45 =
50 5.417 - 5.424 x sin 50 =
60 5.669 - 5.424 x sin 60 =
75 5.622 - 5.424 x sin 75 =
90 5.148 - 5.424 x sin 90 =

4. Draw the statical stability curve.


1. Draw on the vertical axis the scale of GZ.
2. Draw on the horizontal axis the scale for angle of heel.
3. Plot the GoZ against the inclination and connect these points by a fair curve.
4. Locate 57.30 and draw a vertical line. The height of which equals GoM.
5. Connect the height to the origin.

176
STATICAL STABILITY CURVE

0o

0o

5. From the above Statical Stability Curve, determine,

1. The range of stabilitythe angle of inclinations with positive righting


arm
2. The angle of vanishing stability -the angle of inclination at which the
righting arm returns to zero
3. The maximum GZ; can be obtained by drawing a horizontal line
tangent to the highest point and reading off the value from the GZ
scale
4. The angle at which the maximum GZ occurto determine at which
angle this occurs, draw a perpendicular from the point of tangency
to the angle of heel scale and read off the value

6. Calculating the area under the GoZ curve.

177
Example 8.3

MV MINI BULKER has a displacement of 7573 MT and calculated values of KG = 6.32 m, Ggo = 0.18m,
KGo= 6.50 m, KM = 7.91.

Righting Arm Table Assumed KG = 0.0 m

Disp
10 12 15 20 25 30 35 40 45 50 60 75 90
.
750 1.38 1.66 2.08 2.80 3.46 4.03 4.50 4.89 5.20 4.43 5.68 5.63 5.14
0 5 5 8 4 9 3 4 7 7 5 5 1 9
755 1.38 1.66 2.08 2.80 3.46 4.02 4.49 4.88 5.19 5.42 5.67 5.62 5.14
0 5 4 7 2 1 0 0 4 4 3 4 5 8
760 1.38 1.66 2.08 2.80 3.45 4.00 4.47 4.87 5.18 5.41 5.66 5.61 5.14
0 4 3 6 0 3 8 5 0 0 0 3 9 8
765 1.38 1.66 2.08 2.79 3.44 3.99 4.46 4.85 5.16 5.39 5.65 5.61 5.14
0 3 3 5 7 5 5 1 6 7 7 3 3 7
770 1.38 1.66 2.08 2.79 3.43 3.98 4.44 4.84 5.15 5.38 5.64 5.60 5.14
0 3 2 4 5 7 3 7 3 3 4 2 7 7

Calculate and draw the GZ curve.

Solution:

1. Calculate GoM and KGo

KM = F.KG =
F.KG GGo =
--------------------- ---------------------
GM = KGo =
GGo =
---------------------
GoM =

2. Read from the Righting Lever Table the value of KN for each angle of inclination and correct to true
KG.

Gol = KN - (KGo x sin 9)

Angle of (Assumed GZ) - (GZ correction) = Goz


Inclination KN - KGo x Sin 8 = KN (KGo x sin8)
10 1.384 - x sin 10 =
15 2.086 - x sin 10 =
20 2.801 - x sin 10 =
25 3.457 - x sin 10 =
30 4.014 - x sin 10 =
35 4.483 - x sin 10 =
40 4.877 - x sin 10 =
45 5.188 - x sin 10 =
50 5.417 - x sin 10 =
60 5.669 - x sin 10 =
75 5.622 - x sin 10 =
90 5.148 - x sin 10 =

178
3. Plot the Gl in the same diagram used in Example 10.2.
4. From the above Statical Stability Curve, determine:

1. The range of stability -the angle of inclinations with positive


righting arm
2. The angle of vanishing stability -the angle of inclination at
which the righting arm returns to zero
3. The maximum GZ; can be obtained by drawing a horizontal
line tangent to the highest point and reading off the value
from the GZ scale
4. The angle at which the maximum GZ occur to determine at
which angle this occurs, draw a perpendicular from the point
of tangency to the angle of heel scale and read off the value
5. Calculate the area under the GoZ curve.

EXERCISES 8.1

Problem 8.1.1

M. V Nara has cargo at the full length of her deck and loaded up to her Timber summer Draft. The calculated
KG is 10.736 m and FSE corr'n GGo = 0.130 m.

KN Table assumed KG = 0
Heel
10 20 30 40 50 60
Displacement
32000 1.864 3.739 5.505 7.311 8.735 9.507
32479 1.864 3.731 5.488 7.293 8.71 9.484

Calculate the ship's stability at all angles

1. Draw the Stability Curve


2. Find the angle of loll
3. Find the ship's range of stability ,

Solution 8.1.1

Displacement KM =
KG = GM =
GGo = GGo =
KGo = GoM =

179
KN
Angle of Goz
(Assumed KGo x SinO
Heel (KN-KGo x sin O)
GZ)
0 0 0.0000 0.0000 1 0.00 1 0.00

Total Total

CRITERIA ACTUAL
Area (0 to 30) 0.055 m-radian m-radian
Area (0 to 40) 0.09 m-radian m-radian
Area (30 to 40) 0.03 m-radian m-radian
GoZ at angle equal to or greater than 30 0.20 m m
GoZ Max M m
The angle of heel at which the max GoZ occurs >30 degrees degrees

When Assumed KG is not Equal to Zero

Remember that the GZ obtained from cross curves of stability is for an assumed KG equals to zero (KG =
0). To obtain the "real" GZ based on the "true" KG (referred to as KGo) of the ship, correction must be
applied.

For an assumed KG not equal to zero, the correction is found by:

GZ correction = ( KGo -KGa ) x sin 8

Then the final or true GZ (GoZ) is found by:

True GZ (GoZ) = Assumed GZ -GZ correction,

or- True GZ (GoZ) = GZa -[(KGo -KGa) x sin 8]

Note:

In the calculation of GoZ, the rules of signed numbers must be observed.

180
Example 8.4 -True KG is greater than Assumed KG (KGo > KGa)

The ship MV CERLEON has the following condition:

Displacement = 22,737 Hydrostatic mean draft = 6.53 m


Final LCG = -3.48
Final KG = 9.67
GGo = 0.14 m

The following data obtained from its loading manual;

Hydrostatic Table
Draft (M) Displ<A> Mid B Mid F (M) TKM (M) LKM (M) MTC (T- TPC (T) KB (M)
(MT) (M) M)
6.52 22700 -5.17 -3.89 12.22 266.89 367.35 37.33 3.36
6.53 22737 -5.16 -3.88 12.21 266.62 367.56 37.33 3.36
6.54 22775 -5.16 -3.86 12.20 266.34 367.78 37.34 3.37

Righting Lever Table Assumed KG = 9.0 m


Angle Assumed GZ
of o o o o o o o o o o o 0
5 10 12 15 20 30 40 45 50 60 75 90
Heel
Displ
22700 0.285 0.587 0.716 0.924 1.306 2.142 2.392 2.308 2.116 1.498 0.243 -1.155
22737 0.285 0.586 0.715 0.922 1.304 2.139 2.389 2.305 2.113 1.496 0.242 -1.155
22775 0.284 0.585 0.713 0.920 1.301 2.136 2.386 2.302 2.110 1.493 0.240 -1.550

181
Find:
1. GoM 3. Draw the Statical Stability curve
2. KGo

Solution:

1. Calculate GoM and KGo

KGo = KG + GGo where KGo = the height of center of gravity corrected


for free surface effect.

KM = KG =
KG = + GGo =
GM = KGo =
GGo = KGa =
GoM = GoGa =

2. Read from the Righting Lever Table the value of Assumed GZ for each angle of inclination.

3. Calculate the Final GZ for each angle on inclination

GoZ = Assumed GZ ((KGo- KGa) x sin e)

Angle of Assumed GZ - ( KGo - KGa x sin e ) = Actual GZ


Heel
5o - ( x sin 5o) =
10o - ( x sin 10o) =
12o - ( x sin 12o) =
15o - ( x sin 15o) =
20o - ( x sin 20o) =
30o - ( x sin 30o) =
40o - ( x sin 40o) =
45o - ( x sin 45o) =
50o - ( x sin 50o) =
60o - ( x sin 60o) =
75o - ( x sin 75o) =
90o - ( x sin 90o) =

182
4. Draw the statical stability curve

1. Draw on the vertical axil the scale for GZ.


2. Draw on the horizontal axiz the scale for angle of heel.
3. Locate the 57.30 and draw a vertical line the height of which equals GoM.
4. Connect the height of GoM at 57.30 to the origin.
5. Plot the GoZ against the inclination, and connect these points by fair curve.

0o 10o 20o 30o 40o 50o 60o 70o 80o 90o


Angle of Inclination
5. Calculate the area the GoZ curve.

183
Example 8.5 -Actual KG is less than Assumed KG (KGo < KGa)

The ship MV CERLEON has the following condition:

Displacement = 34,541 M Hydrostatic mean draft = 9.58


Final LCG = -3.48
Final KG = 8.17
GGo = 0.11

The following data obtained from its loading manual;

Hydrostatic Table
Draft (M) Displ<A> Mid B Mid F (M) TKM (M) LKM (M) MTC (T- TPC (T) KB (M)
(MT) (M) M)
9.57 34501 -3.81 1.38 11.32 215.16 446.13 40.00 4.97
9.58 34541 -3.80 1.39 11.32 215.03 446.37 40.00 4.97
9.59 34581 -3.80 1.40 11.32 214.91 446.62 40.01 4.98

Righting Lever Table Assumed KG = 9.0 m


Angle Assumed GZ
of Heel 5o 10o 12o 15o 20o 30o 40o 45o 50o 60o 75o 900
Displ
- -
34501 0.203 0.418 0.508 0.653 0.906 1.228 1.314 1.228 1.072 0.615
0.263 1.196
- -
34541 0.203 0.418 0.508 0.653 0.906 1.225 1.310 1.224 1.068 0.611
0.265 1.196
- -
34581 0.203 0.418 0.508 0.653 0.906 1.222 1.306 1.220 1.064 0.608
0.267 1.196

184
0o 10o 20o 30o 40o 50o 60o 70o 80o 90o
Angle of Inclination

6. Calculate the area under the GoZ curve.

185
EXERCISES 8.2

Problem 8.2.1

The ship MV CERLEON has the following condition:

Displacement = 32,230 M
Final LCG = -4.15
Final KG = 10.05

The following data obtained from its hydrostatic table,


Hydrostatic Draft = 9.00 M
KM = 11.36 (x)G =-4.15 (x)F = 0.61 MTC =431.56
GGo =0.11 (x)B =-4.15 TPC = 39.51

Find: Find
1. GoM 3. Draw statical stability curve
2. Kgo 4. Use the statical stability calculation form

Righting Lever Table


Assumed KG = 9.0 m
Angle of
5o 10o 12o 15o 20o 30o 40o 45o 50o 60o 75o 900
Heel
Displ
- -
34501 0.208 0.427 0.520 0.670 0.955 1.390 1.547 1.460 1.293 0.795
0.166 1.193
- -
34541 0.208 0.426 0.519 0.670 0.954 1.388 1.543 1.456 1.289 0.792
0.167 1.193
- -
34581 0.208 0.426 0.519 0.669 0.954 1.385 1.539 1.452 1.285 0.789
0.169 1.193

Solution

186
MV CERLEON : STATICAL STABILITY CURVE CALCULATION

Displacement KM =
KG = GM = KGo (Actual
GGo = GGo = KGa (Assumed)
KGo = GoM = KGo Kga =

Angle of (KGo Kga) Actual GZ


Assume GZ SM Product SM Product
Heel x Sin O GoZ
0 0 0.0000 0.0000 1 0.00 1 0.00
o
5
o
10
12o
o
15
o
20
30o
o
40
45o
50o
60o
75o
90o Total Total
CRITERIA ACTUAL
Area (0 to 30) m - radian m - radian
Area (0 to 40) m - radian m - radian
Area (30 to 40) m - radian m - radian
GoZ at angle equal to or greater than 30 m m
GoZ Max m m
The angle of heel at which the max GoZ occurs degrees degrees

Problem 8.2.2
The ship MV CERLEON has the following condition:
Displacement = 34,541 M Hydrostatic mean draft = 9.58
Final LCG = -3.48
Final KG = 8.17
The following data obtained from its hydrostatic table,
KM = 11.32 (x)G =-3.48 MTC =446.37
GGo = 0.12 (x)B = -3.80 TPC = 40.00
(x)F = 1.39
Righting Arm Table: Assumed KG = 9.0 m
Angle of o o o o o o o o o o o 0
5 10 12 15 20 30 40 45 50 60 75 90
Heel
Displ
0.203 0.418 0.508 0.653 - -1.196
34501 0.906 1.228 1.314 1.228 1.072 0.615
0.263
0.203 0.418 0.508 0.653 - -1.196
34541 0.905 1.225 1.310 1.224 1.068 0.611
0.265
0.203 0.418 0.508 0.653 - -1.196
34581 0.904 1.222 1.306 1.220 1.064 0.608
0.267
-
34621 0.203 0.418 0.508 0.653 0.903 1.219 1.302 1.216 1.060 0.605 -1.196
0.268

187
Find:

1. GoM 3. Draw statical Stability curve


2. KGo 4. Calculate the area under the curve 30, 40 degrees
and between 30 and 40 degrees angle of inclination.

Solution
MV CERLEON : STATICAL STABILITY CURVE CALCULATION

Displacement KM =
KG = GM = KGo (Actual
GGo = GGo = KGa (Assumed)
KGo = GoM = KGo Kga =
(KGo
Angle of Actual GZ
Assume GZ Kga) x Sin SM Product SM Product
Heel GoZ
O
0 0 0.0000 0.0000 1 0.00 1 0.00
5o
10o
12o
15o
20o
30o
40o
45o
50o
60o
75o
90o Total Total
CRITERIA ACTUAL
Area (0 to 30) m - radian m - radian
Area (0 to 40) m - radian m - radian
Area (30 to 40) m - radian m - radian
GoZ at angle equal to or greater than 30 m m
GoZ Max m m
The angle of heel at which the max GoZ occurs degrees degrees

STABILITY CONSIDERATIONS

Deck Edge Immersion Angle

Information about the angle of deck edge immersion is usually supplied in stability manuals in form of table
or graph. Figure 9.18 shows the deck edge immersion angle for the ship in study -MV Mini-Bulker. The
vertical axis of the graph is the angle of deck edge immersion and the horizontal axis is the draft. To find the
deck edge immersion, locate the draft at the horizontal scale. Then draw a perpendicular line through the
draft to cut the curve on the graph. The angle of deck edge immersion is the intersection of the line and the
curve. Read off the value from the scale on the left vertical axis.

188
Figure 10.3 -Angle of deck edge immersion -M. V. Mini-Bulker

The angle of deck edge immersion can also be found by the formula:
Tan 8 = freeboard / half breadth

Stiff and Tender Ships

When the GM of a ship is comparatively large, the righting moment at small angles of inclination will also be
comparatively large. When inclined she will restore her -initial position more quickly. The ship's rolling period
is comparatively short. The rolling period could be as low as 8 seconds. A ship behaving this way is said to
stiff".

In contrast, when the GM of a ship is comparatively small, the righting moment at small angles of inclination
will also be small. When inclined, she will not tend to return to her initial position as quickly. The ship's rolling
period will be comparatively long, possibly 30 to 35 seconds. A ship in this condition is said to be tender".

189
Both conditions are not desirable and steps should be taken to obtain an easy rolling period, neither too fast,
nor too slow. Halfway between these two conditions or a rolling period of 20 to 25 seconds is generally
acceptable. A vessel cannot always be loaded ideally, but a ship officer loading a ship should always aim for
stability consistent with safety.

Correcting unstable and neutral equilibrium

To correct a vessel when she is in unstable or neutral equilibrium, one or more of the following methods may
be done.

1. remove all free surfaces within the ship


2. lower the weights already in the ship
3. load weights below the center of gravity of the ship
4. discharge weights from a position above the center of gravity of the ship

Angle of Loll
The vessel that is unstable may assume an angle of inclination that is called the angle of loll. A vessel in
unstable equilibrium when inclined to a small angle; creates an upsetting moment or capsizing moment and
the vessel will incline more and more. As the angle of heel increases, the center of buoyancy moves out still
further to the low side.

It may occur that at certain angle of heel, the center of buoyancy moves out to a position vertically under G,
and the capsizing moment would then be reduced to zero. The angle of heel at which this occurs is called
angle of loll.

The vessel will roll about the angle of loll instead of the upright. When the ship is inclined less than the angle
of loll, a capsizing moment will again form to return her to the angle of loll.

The angle of loll could be at either side of the vessel. A vessel may lie to an angle of loll to port and or to
starboard and then back to port again, depending on the external forces such as wind and waves.

190
When the ship is inclined beyond the angle of loll, the center of buoyancy will move out still further to the low
side. A righting moment will be created that would tend to return her to the angle of loll. If the center of
buoyancy does not move out far enough to get vertically under G, the ship will capsize.

Rules when filling double bottom tanks, in case a ship is found to be at an angle of loll as a step taken
to improve her stability.

Pairs of small DB tanks should be filled first, before pairs of large DB tanks.
In each case the tank on the low side must be filled before the tank on the high side.

For example, if the vessel has taken up a position of starboard angle of loll, the starboard DB tank
must be filled before the port one. This will initially result in a further increase in the angle of loll to
starboard, but the increase will be gradual and controlled. Thereafter when the port tank is filling, the
angle of loll will diminish and eventually disappear, provided that the filling of the first pair of tank is
sufficient to eliminate the negative stability.

If these rules are ignored, and the DB tank on the high side is mistakenly filled first, the ship will, at
some time during the process, rollover violently from her angle of loll from starboard to take up a
similar angle of loll to port. The sudden roll may cause the vessel to list to a far greater angle and
subsequently capsize.

The disadvantage of the method describe above is when the vessel has only a small reserve stability,
additional list may cause cargo to shift.

191
Definitions

1. Stability of a vessel for a small angles of inclination Initial Stability


(up to 15 degrees).

2. Means absence of movement, in the case of ships, an Statical


absence of movement of the water in which the vessel
is assumed to be floating.

3. Is the stability of a vessel at statical condition. Statical Stability

4. Are graphical presentations showing the value of the Statical Stability


righting arms at various angles of inclinations and at Curves
various displacements.

5. A ship is said to be heeled when she is inclined by an Heel


external force.

6. A ship is said to be listed when she is inclined by List


forces within the ship.
7. Traditionally defined as any push or pull on a body. Force
Points to consider regarding the force.
a. The magnitude of the force
b. The direction in which the force is applied,
and
c. The point at which the force is applied

8. The force representing the combined effect of two or Resultant Force


more forces acting at a point.
9. It is formed by the two equal forces acting on a body in Couple
opposite directions and along parallel lines. All couples
are expressed as a moment.
10. Is a measure of the turning effect of the force about a Moment of a force
point.
The turning effect will depend upon the following:
a. The magnitude of the force, and
b. The length of the lever upon which the force
acts, the lever being the perpendicular
distance between the line of action of the
force and the point about which the moment
is being taken.
The magnitude of the moment is the product of the
force and the length of the lever.
Force can be expressed as weight
Lever can be expressed as distance
11. Imaginary moment representing the combined effect of Resultant
two or more forces acting about a point. Moment

Density And Specific Gravity:

12. Is defined as 'mass per unit volume'. Density


13. Is defined as the ratio of the weight of the substance to Specific
the density of fresh water. gravity(SG); relative density

192
Mass And Weight According To S.I. Units.

14. Is the fundamental measure of the quantity of matter in a Mass


body and is expressed in terms of the kilogram and the
tonne.

15. Is the force exerted on the body by the earth's Weight


gravitational force and is measured in the terms of
Newton(N) and kilo-Newton (kN).

Weight and mass are connected by the formula: -


Weight = mass x acceleration
Mass = weight facceleration

Buoyancy And Gravity

16. The force acting vertically upwards tending to support a Buoyancy


body which is wholly or partly immersed in a fluid.

17. Is that point at which all the vertically upward forces of Center of
buoyancy can be considered to act; or it is the center of Buoyancy (B)
the volume of the immersed portion of the vessel.

18. Is that point at which all the vertically downward forces Center of
of the weight of the vessel can be considered to act; or it Gravity (G)
is the center of the mass of the vessel.

Distances and Points

19. Is a point to through which the center of buoyancy acts META CENTER
vertically upwards as the vessel is inclined and B shifts to (M)
low side.
It is also the highest point at which the center of
gravity (G) may rise and still permit the vessel to have
positive stability.
The metacenter is located at the intersection of
the line of force of the center of buoyancy and when the
vessel is erect and the line of force of center of buoyancy
when the vessel is inclined a small angle.
The metacenter remains in the same position for
angles of inclination (or heel) up to about 15 degrees
from the vertical. Because the height of metacenter (KM)
above the keel (K) depends upon the ship's underwater
form. Over 15 degrees the metacenter no longer remains
in the same position.
20. Is the perpendicular distance from center of gravity (G) RIGHTING ARM
to the line of force of center of buoyancy (6). The or RIGHTING
distance is formed when the vessel is inclined or heeled LEVER (GZ)
and the 6 shifts to other location (B 1 ).
21. Is a couple formed by the parallel and opposing forces of moment of
(G) and (6). The magnitude of the couple is expressed statical stability
as a moment. The moment is equal to the product of GZ
and the displacement of the vessel.

193
22. Is the vertical distance from the center of gravity (G) to META CENTRIC
the transverse metacenter (M). The length of metcentric HEIGHT (GM)
height (GM) is directly related to the length of the
righting or upsetting arms and therefore will directly
indicate the measure of ship's initial stability.

23. Is the vertical height of G above the keel. For transverse KG


stability calculations it is assumed that G will be on the
centerline when KG is used.

24. Is the distance of the metacenter above the keel. KM

25. The height of the center of buoyancy above the keel. KB

26. Distance from 6 to M. Also known as metacentric radius. BM

194
GRAIN LOADING CALCULATIONS

Grain Loading

Definitions

Angle of repose -When grain is freely poured into a


compartment it arranges itself into a pile of conical
shape. The angle of the surface of the grain with
horizontal is termed angle of repose. If the pile is not
moving as in at sea, the surfaces of the grain remain
undisturbed.

If the grain is in a ship, then the grain is subjected to the motions of the ship at sea. The surface of the grain
will move in response to this motion because the grain will maintain its angle of repose. This movement of
grain is called the grain shift.

Grain shift -The movement of grain due to the motion of a ship at sea and consequently the movement of
the center of gravity of the grain mass.

The center of gravity of the grain after shifting will no longer lie on the centerline and will cause the ship to
heel. By how much the ship heels can be indicated by its grain heeling moment.

The magnitude of grain shift depends upon the amount of open space above the grain into which it can
move. Thus when a compartment is filled to the maximum extent possible, the adverse effect of grain shift,
that is the grain heeling moment, will be less.

195
To minimize the possibility that bulk grain will shift at sea, the IMO Grain Rules require that the grain be
trimmed, that is leveled to a zero degree angle with horizontal; after it has been loaded. In such a case the
ship will have to roll in excess of the angle of repose, before the grain would shift.
Grain heeling moment -is the off center eight
condition which causes the ship to heel. The amount
at which the ship would heel after the grain shift can
be indicated by its heeling moment. The grain
heeling moment is the distance the center of gravity
of grain has moved multiplied by its weight.

Calculating the grain heeling moment is tedious and


because of the lengthy arithmetic involved it is not
practical for the ship officer to do the calculation
himself for specific loading conditions.

What is provided to the ship officers by the naval architects is the pre-calculated volumetric heeling moment
in form of graphs.

Volumetric heeling moment -given the angle of shift of grain and the internal geometry of the ship the
naval architects can pre-calculate the volume of space the will be occupied by grain after shifting. The
volume times a distance does not constitute moment. But volume of space divided by stowage factor of the
commodity which fills the volume equals a weight.

From volumetric heeling moment the grain heeling can be calculated as:

Grain Heeling Moment = Volumetric heeling moment


3
Stowage Factor ( m /MT)
Or by;

Grain Heeling Moment = Volumetric heeling moment x 35.88


3
Stowage Factor ( ft /L T)

One of the assumptions in pre-calculating the volumetric heeling moment is the angle of shift. Under the IMO
Grain Rules:

For filled compartment -the resulting grain surface after shifting shall be assumed to be at 150 to the
horizontal.

For partly filled compartment -the grain surface after shifting shall be 250 to the horizontal.

The volumetric heeling moments for each cargo space are obtained from tables or graphs in the loading
manual. The correct tables or graphs must be used and corrections applied to the values obtained as
specified in the loading manual or as necessary.

196
Corrections for the feeling moments so obtained may be governed by the following factors:

1. Grain stowed with trimmed and untrimmed ends


2. Assumed center of gravity
3. Whether the compartment is full or not full
4. Method of assessing the amount of grain in the compartment
5. Any other corrections included in the table

Trimmed And Untrimmed Ends

The grain shift depends upon the amount of open space above the grain into which it can move. The grain
heeling moment will be less for a compartment filled to the maximum extent possible, than if the
compartment is partially filled. As defined in the International Grain Code;

The term filled compartment, trimmed, refers to any cargo space in which, after loading and trimming as
required under A 10.2 (the bulk grain shall the trimmed so as to fill all spaces, (under the decks and
hatch covers to the maximum extent possible) that the bulk grain is at its highest possible level.

The term filled compartment, untrimmed, refers to a cargo space which is filled to the maximum extent
possible in way of the hatch opening but which has not been trimmed outside the periphery of the
hatch opening either by the provision of A 10.3.2 for all ships or A 10.3.2 for specially suitable ships.

The term partly filled compartment refers to any cargo space wherein the bulk grain is not loaded in the
manner prescribed in A 2.2 or A 2.3.

Assumed Center Gravity Of Grain

The volumetric center of gravity is


normally higher than the true center of
gravity of cargo.

The volumetric center is the center of the volume of the cargo, including broken stowage. For a full
compartment the volumetric center is at the c3enter of the volume of the compartment.

GRAIN LOADING STABILITY REQUIREMENTS

Once the grain heeling moment has been determined, the response of the ship to this moment must meet
the following requirements as prescribed in the International Code for the Safe Carriage of Grain Bulk which
states that;

7.1 The intact stability characteristics of any ship carrying bulk grain shall be shown to meet, throughout the
voyage, at least the following criteria after taking into account in the manner described in Part B of this Code
and, in figure A7, the heeling moments due to grain shift:

197
1. the angle of heel due to the shift of grain shall not be greater than 12 or in the case of ships
constructed on or after 1 January 1994 the angle at which the deck edge is immersed,
whichever is the lesser;

2. in the statical stability diagram, the net or residual area between the heeling arm curve and the
righting arm curve up to the angle of heel of maximum difference between the ordinates of the
two curves, or 40 or the angle of flooding (S 1 ), whichever is the least, shall in all conditions of
loading be not less than .075 meter-radians; and

3. the initial metacentric height, after correction for the free surface effects of liquids in tanks, shall
be not less than 0.30 m.

(1) Where:

AO = assumed vol. heeling moment due to transverse shift


stowage factor x displacement
A40 = 0.8 x AD

Displacement = weight of ship, fuel, fresh water, stores etc. and cargo

(2) The righting arm curve shall be derived from cross-curves which are sufficient in number to accurately
define the curve for the purpose of these requirements and shall include cross-curves at 12 and 40.

ITEMS TO CONSIDER WHEN LOADING GRAINS

1. Quantity and type of grain to be loaded


2. An accurate estimate of the stowage factor
3. The quantities of fuel and water on hand at departure, daily consumption and amounts
4. The seasonal zones to be traversed during the voyage
5. The quantities and stowage of other cargo to be carried in the ship at the same time as the grain.
6. The distance and steaming time required to the port or ports of discharge
7. Any other restrictions, which may be encountered during the voyage.

198
METHODS OF GRAIN LOADING CALCULATIONS

There are two methods for calculating the intact stability of the ship when loading grain in accordance with
the requirements of International Grain Code.

1. The first one is a method using the Table of Allowable Heeling Moment.
2. The second method is to obtain directly the heeling angle and residual dynamical stability from the
statical stability curve of the loading condition under consideration.

GRAIN LOADING CALCULATION USING THE TABLE OF ALLOWABLE GRAIN HEELING MOMENT

To facilitate the grain heeling moment calculation, the data can be tabulated as shown below.

Taking the Case 4 -as an example;

Horizontal Volumetric Grain


No. of Weight Volume
Stowage 3) Heeling Multiplier Heeling S.F. Heeling
Hold (T) (m
moment moment moment

Step 1.0 - Calculate the actual volume occupied by the cargo. Volume of cargo equals weight times stowage
factor.
Step 2.0 - Calculate the grain heeling moment for each hold loaded with grain by completing the above
table.

For holds with ends trimmed -read and copy the values of volumetric heeling moments from the
tables of information pertaining to filled compartment end trimmed.

Note: Look up the table in pages of your loading manual

Ex.

from graphical data;

if the values found in tables are for horizontal volumetric heeling moment, multiply these with 1.0 to get
volumetric heeling moment.
Divide the volumetric heeling moment by stowage factor (in metric units) to get grain heeling moment.
Repeat the process for other holds filled with ends trimmed.

Step 3.0

199
3.1 For holds full but ends untrimmed -read values from tabulated volumetric heeling moments for
filled compartments ends untrimmed.

Information pertaining to cargo holds filled with untrimmed ends.


3
Compartment Grain Capacity (m ) V.H.M
No.2 Cargo Hold 12.667.2 2684.9

3.2 Divide the volumetric heeling moments by stowage factor to get grain heeling moment.

Step 4.0

4.1. For holds partially filled. -use the volume of cargo in partly filled compartments as argument to
enter the graph of volumetric moments.
4.2. to get volume:
Volume = cargo weight x SF
Volume = 8348 mt x 1.3 m3fmt = 10868 m3
4.3. Locate volume at the bottom scale mark "Capacity" and draw up a vertical line to the curve
labeled "CAPACITY".d
4.4. At the intersection of the projected line and the capacity curve, draw a horizontal line (parallel to
the base).
4.5. Mark intersection at horizontal heeling curve, KG curve, depth of hold etc.
4.6. Drop a line from this intersection and read at the bottom scale the value for horizontal heeling
moment, and KG.

CURVES OF VOLUMETRIC HEELING MOMENT

200
Apply the multiplier factor for vertical shift of grain as appropriate. The multiplier has three possible
values (1.0, 1.06, 1.12). These corrections are normally applied as follows or as specified in the
loading manuals.
multiplier 1.0

- hold is full and KG is the volumetric center.


- The table or curve of heeling moment has been adjusted for vertical shift of grain by the use of
multiplier 1.12.

multiplier 1.06

- hold is full and the KG is true center of cargo.

multiplier 1.12

- hold is partially filled, and the KG is true cargo center or the graph was not corrected for vertical shift
of grain.
4.8 Convert the volumetric heeling moment to the grain heeling moment.

Grain Heeling Moment = Volumetric heeling moment


3
Stowage factor (m /MT)

Or by;

Grain Heeling Moment = Volumetric heeling moment x 35.88


3
Stowage Factor ( ft /LT)

Step 5.0

5.1 Take the sum of actual grain heeling moment and compare it with the Allowable Heeling
Moment.

5.2 Should the actual heeling moment exceed the allowable heeling moment for any voyage
condition, the loading arrangement must be amended to produce less heeling moment.

Units of all tonnages used in calculation should be shown in the same units as are used in the
approved grain loading booklets. Tonnages and moments may be rounded off to the nearest
unit.

201
202
203
204
205

Potrebbero piacerti anche